You are on page 1of 104

1.

Introduction
Corporate Personality - Corporate Governance - Majority rule - Minority protection

1.1 What is a Company?

A distinct legal entity, separate from those who created it, and its members, to which rights,
liabilities, duties and privileges may be accrued. Sutton's Hospital (1612)
Companies: are formed by registration under the Corporations Act as either public or proprietary
companies, CA s 9.

 Originally companies were always multi-member corporations, but it is now possible for a
company to be formed with only one member.
 Other corporations (e.g. incorporated associations and statutory corporations) also fall under
the definition of ‘company’.

1.1.1 Historical development of the corporate form

UK Development 1961-1963: Uniform Companies Act 1961

 Early boroughs and guilds 1989: Corp’ns Act 1989


 Incorp’n by Royal Charter
NSW v Commonwealth (1990) challenged
 Incorp’n by Private Act of Parliament
Corp’ns Act 1989; held invalid: Cth has no
 The rise of Joint Stock Companies: de power over co incorp’n.
facto incorp’n; mimicked benefits of
regulated and statutory companies (Joint 1991: Corp’ns Law 1991
stock co: stock is owned jointly by the
1999-2000: Constitutional challenges to the
shareholders)
Corp’ns Law 1991.
 Bubble Act 1720: designed to outlaw
joint stock companies. R v Hughes (2000): Affirmed NSW v Cth
 Repeal of the Bubble Act in 1825 (1990) that corp’ns power is restricted to
 Joint Stock Companies Registration and corp’ns formed under some independent
Regulation Act 1844: permitted statutory source of power and excludes power to make
incorp’n of joint stock companies as a laws regarding the process of incorp’n itself.
right
Re Wakim; ex parte McNally (1999): The
 Limited Liability Act 1855: first allowed
Constitution exhaustively states the manner in
limited liability for corp’ns established by
which jurisdiction can be invested in federal
general public.
courts. Legislation that purported to best State
 Companies Act 1862
judicial power in the Federal Court was
Australian Development invalid: no mirroring provision existed to vest
state courts with federal jurisdiction.
Limited Cth powers in the Constitution, most
notably: s 51(i) T&C and s 51(xx) corp’ns 2001: Corp’ns Act 2001: Current arrangement
based on referral of state power.
Cth powers did not extend to formation and
incorp’n of companies, power to legislate wrt 2006: Australian Stock Exchange merged
reg’n of corp’ns belonged to the states. But with Sydney Futures Exchange to be the
states could refer their power to Cth. Australian Securities Exchange (the “ASX‟)
regulated by ASIC.
1901-1950: Separate State Acts

1
1.1.2 Theories and conceptions of the corporation

Managerialist theory Contractual theory

Managerialist Theory Contract Theory


What is a Hierarchy based entity, Separation Nexus of contracts- made by rational,
company? of Ownership and Control [Berle & self interested agents seeking to
Means(1932)] maximise own economic well being
What is the key To ensure the accountability of To reduce transaction costs of
role of company management to shareholders via contracting and prevent market failure
law? mandatory legal duties
What outside Shareholder interest paramount. Minimal.
interests can be Company law is concerned with Main focus is to reduce costs of
taken into account internal relations between ownership voluntary contract and uphold
in determining and control freedom of contract.
internal company
decisions?

Concession theory

Corp as an artificial entity The separate legal status of the company is treated as a concession or
privilege granted by the state

Shareholder approach

Prioritise interests of shareholders.

Stakeholder approach

Example is communitarian theory, where the corporation is viewed as a community to address the
threat of harm to non-shareholders from exclusive management focus on shareholder interests. Similar
themes underpin regulation imposed by the state under concession theory, but concession theory also
protects rights of shareholders.

Other

Team Production Model: The Corporation ultimately works not for shareholder owners but for the
team members of the firm.
Communitarian Theory: Views the corporation as a community to address the threat of harm to non-
shareholders from exclusive management focus on shareholder interests.

2
1.1.3 Policy concerns

1.1.4 Types of companies

Proprietary Limited by shares


Companies Unlimited with share capital
Public Limited by shares
Companies Limited by guarantee
Unlimited with share capital
No- liability company

A. Classification according to liability

Company limited by shares (s 9, public or proprietary)


Liability of its members limited to the amount (if any) unpaid capital on the shares respectively
held by them
Company limited by guarantee (s 9, public)
Liability of its members limited to the respective amounts that the members undertake to
contribute to the property of the company if it is wound up
Unlimited liability company (s 9, public or proprietary)
No identifiers for unlimited companies. Just have to write ‘Pty’ if they are proprietary.
No limit on members’ liability
Fails to insulate members from trading losses of the enterprise, but it does have one advantage:

 Under the law on maintenance of a company’s share capital, a company may only reduce its
share capital pursuant to a formal approval mechanism. But because members of unlimited
companies bear full responsibility for the company’s liabilities. this class of company is
exempted from the prohibition upon unsanctioned capital reduction (s 258A).
 The power to reduce capital is valuable to companies which attract the funds of the investing
public and invest them in corporate and government securities. They are structured so that
investors may redeem their investments, which take the form of shares in the mutual fund
company, simply by having the company buy back their shares.
No liability company (s 112, public)
Means a company that is registered as… a no liability company under this Act (s 112(2)-(4))

3
Identified by NL or No Liability as part of and at the end of its name (s 148(4))
May be registered as a no liability company only if the company:
1. Has a share capital (all members must hold shares); AND
2. Company’s constitution states that its sole objects are mining purposes (viz, prospecting for,
extracting or selling ores, metals or minerals: s 9); AND
3. Company has no contractual right under its constitution to recover calls made on its shares
from a shareholder who fails to fully pay them: i.e. if director makes a call on the shares,
members can choose whether or not to pay them (s 112(2))
NL companies have a number of special provisions.
POLICY ISSUES: LTD LIABILITY
Pros:

 Encourages investment (for those with no interest in or capacity for mgmt participation)
 Relieves s/h from burden of monitoring other s/h capacity to contribute
 Encourages free liquidity of share capital: ↓ cost of capital; ↑ accountability (sell if under-
performing)
 Market price mechanism for shares only works if the price reflects the value of the share (and
the business) itself, not the ability of a s/h to pay for company’s deficiency.
 Encourages entrepreneurial risk – invest in projects with possibility of +ve returns with risk
exposure, while still limiting the risk exposure
Cons:

 Shifts risk from owners to creditors


 Protects company assets but limits exposure to claimants in tort (e.g. James Hardie asbestos)
 Logic of separate legal personality and limited liability doctrines favour externalisation of the
social costs of corporate behaviour:
o Shifting risk of enterprise operations away from shareholders and onto stakeholders
or wider society (sometimes employees too

B. Classification according to size

Public companies
s 9 Public company means a company other than a proprietary company.
Requirement to hold an AGM applies only to public companies (s 250N). Public companies can be
listed or unlisted.
Proprietary Companies
s 45A(1): A proprietary company is a company that is registered as, or converts to, a proprietary
company

 To be a proprietary company, a company must:


o s 113(1): have no more than 50-non employee shareholders
o s 113(3): not engage in any activity that would require the lodgement of a prospectus,
although it can offer shares to existing shareholders and to employees of the company
or its subsidiaries
 Also: pty company must be limited by shares or be an unlimited company with a share
capital: s 112(1)

4
Privileges applicable to proprietary companies?

 May incorporate with a single shareholder(s114) and trade with single director(s 201A)
 No requirement to hold an annual general meeting (cf 250N)
 May pass shareholder resolutions without holding a general meeting(s 249A)
 Not affected by related party transactions regime under Ch 2E
 No entrenched statutory right of members to remove directors (cf 203D)
 Directors may be removable by other directors (cf 203E)
Small and large proprietary companies
s 45A(2): proprietary company is a small proprietary company for the financial year if it satisfies at
least 2 of the 3 following conditions:
(i) consolidated revenue for company and entities it controls (if any) for the financial year is
< $ 25 million;
(ii) value of the consolidated gross assets at the end of the financial year of company and
entities it controls (if any) < $12.5 million;
(iii) company and entities it controls (if any) < 50 employees at the end of the financial year;
Otherwise: it will be a large proprietary company [s 45A(3)]
Question of Control? Accounting standards [s 45A(4)]
Privileges applicable to small proprietary companies?

 All companies must keep written financial records explaining transactions and their financial
position (s 286)
 BUT small pty co.s do not have to:
o Have their accounts audited (cf large pty co’s, and public co’s)
o Lodge their accounts with ASIC. ie their accounts remain private (s 295)
o Obligations to audit, & publish accounts may arise in certain circumstances (s 293-
s/h direction, s 294 ASIC direction)

Proprietary (Pty Ltd) Public (Ltd)


Shareholders Non-employee shareholders: Min: 1
(s 45A, note 2; s 113) Min: 1 Max: 50 No maximum
Members Min: 1 Min: 1
s 114
Directors 1 director 3 directors
s 201A (1 resident in Australia) (2 resident in Australia)
Finance Difficult to get sufficient funds Can get funds from the public
(s 45A, note 2; s 113) from the public (with disclosure document)
Listing Can’t be listed Can be listed or unlisted
(s 45A, note 2; s 113)
Type  Limited by shares  Limited by shares
(s 45A, note 2; s 113)  Unlimited  Limited by guarantee
 Unlimited
 No liability

C. Classification according to listing

CA ss 45A, 112–116, Part 1.5 “Small Business Guide”

5
1.1.5 Corporate constitution and organs

Corporate Constitutions

 In 1998 the requirement to have a corporate constitution in Australia was abolished.


 In place of the memorandum and articles of association, the Corporations Act introduced a
series of provisions which any company may use to regulate its internal proceedings and
management.
s 135: some are ‘replaceable rules for all companies’ and some are ‘replaceable rule for proprietary
companies and a mandatory rule for public companies’
If a company chooses to adopt its own constitution, that will displace the application to the company
of any inconsistent rule except, in the case of a public company, a rule which is expressed to be
mandatory and which operates therefore as an ordinary provision of the CA for the company.
Companies have three options in relation to the choice of rules to govern their internal
management:
1. Elect to function without a constitution, relying solely upon the replaceable rules
2. Adopt its own constitution to displace or modify the RR wholly or in part (s 136)
a. A public company which adopts a constitution must lodge a copy with ASIC together
with a copy of any special resolution altering its provisions: ss 117(3), 136(5).
3. If the company was incorporated before 1 July 1998, it may retain its memorandum and
articles of association as the constitution of the company to the exclusion of inconsistent
replaceable rules; if the company is a public company, those provisions of the Act which
contain a mandatory rule for public companies also apply to them s 135
Varying the terms of a constitution

 A company may adopt or vary the terms of a constitution by special resolution: s 136(1)-(2).
 Special resolution is one passed w/o the support of at least 75% of the votes cast by
members entitled to vote on the resolution who have notice of intention to propose the
special resolution and of its terms s 9
 There is scope for entrenchment of a constitutional provision against alteration by a special
resolution, by specifying in the constitution a further requirement for its alteration which
provision may not itself be repealed unless the further requirement is satisfied s 136(3), (4).
 For example:
o Special resolution be passed with a greater majority than 75% of the votes cast
o The consent of a particular person
o A particular condition fulfilled
Effect of constitution
Company’s constitution and any replaceable rules that apply to it have effect as a contract between:

 The company and each member


 The company and each director and secretary and
 The members themselves
…under which each person agrees to observe and perform the constitution and rules so far as
they apply to that person s 140

6
RR do not apply to a proprietary company while the same person is both its sole shareholder and sole
director. This is justified on the ground that such a company has little need for a formal set of rules
governing its internal relationships.

 Special rules applying specifically to such companies: ss 198E, 201F and 202C.
 Effect is to allow the company to function flexibly in its corporate character through the sole
individual who constitutes and acts for it.
Breach of constitution
Failure to comply with a replaceable rule is not of itself a contravention of the CA (s 135(3)) and the
intention is clear that the rules and constitution are on an equal footing in terms of their status and
enforceability.

Corporate Organs

Who can act as an organ for the corporation?


The corporation is an abstraction: it has no physical existence and exists only in
contemplation of law, so it requires the mediation of natural persons for the expression and
execution of its will.
Historically, two groups of individuals have been recognised as having authority to act for
the corporation. Its members assembled in general meeting and its board of directors or
governing committee.
Management powers are all but invariably vested in the board s 198A (RR).

 In a smaller company, the board may function as an organ to take the active direction of
company affairs.
 However, in most companies of any significant size the day-to-day management and
operations of the company are conducted by senior managers appointed by the board and
employees appointed by those managers.
 Company constitutions commonly provide for the appointment of managing (or executive)
directors and for the delegation to them of the powers of the board (ss 201J, 198C). Act also
requires company to appoint secretary (s 204A)

1.1.6 Corporate groups

s 46: Subco is a subsidiary of Holdco iff:


(a) Holdco
i. controls the composition of Subco’s board OR
ii. is in a position to cast more than half the votes in Subco’s GM
OR
iii. holds over half of Subco’s share capital OR
(b) Subco is a subsidiary of Medco, where Medco is a subsidiary of Holdco
s 9 (definition of “holding company”): in relation to a body corporate, means a body corporate of
which the first body corporate is a subsidiary.
Note: ss 50, 50AA

7
s 50AA: Definition of control: an entity controls a second entity if the first entity has the capacity to
determine the outcome of decisions about the second entity's financial and operating policies
Capacity is marked by:

 The practical influence that the putative controller has rather than the rights it can enforce;
and
 any practice or pattern of behaviour affecting those policies (s 259E)
s 9: A wholly owned subsidiary is where the holding company owns the whole of the share capital of
the subsidiary (either directly or through nominees or subsidiaries with no outside interests)
s 9 Group of companies is common term for the ultimate holding company and its subsidiary
companies.
s 50 Each member of the group is said to be related to the others.

1.2 Administration of Australian companies

1.3 Why form a company?

 Limited liability
 Perpetual succession
 Taxation benefits for corporations
 Finance: Companies can raise share capital; obtain debt. Sole traders must provide their own
capital or borrow.

1.4 Other forms of Business Association

1.5 How do you form a company?

 Application for registration lodged with ASIC: s 117 (including company type, name,
member details, director details etc.)
 Pursuant to s 350(1)(b), ASIC approved Form 201 as the form to be lodged under: s 117(1)
 ASIC may give ACN, register and issue certificate of company details: s 118
 A company comes into existence as a body corporate at the beginning of the day on which it
is registered and derives powers and capacity only upon incorporation: s 119
o See further: ss 117-123

8
2. Corporate Personality and Limited Liability
Important terms:
Debenture: a debt instrument not secured by assets or collateral, just creditworthiness, e.g. a loan.
Floating Charge: a security that has an underlying asset (or group of assets) subject to change in
quantity and value. If company fails to repay, it becomes ‘fixed’. e.g. a mortgage or lien.
Fixed Charge: a security over a specific underlying asset to secure payment of debt eg parcel of land.
Unsecured creditor: individual or institution that loans money without obtaining specific assets as
collateral, e.g. an unsecured debenture.
Liquidator: person appointed to wind up the company.
Winding up: process of selling all assets, paying off creditors, distributing any remaining assets to
S/H and dissolving the business.

2.1 The doctrine of corporate personality

Once incorporated, a company is a separate legal entity from its incorporators (members, owners,
directors, etc) (Salomon’s Case)
Reflected in s 124: “a company has the legal capacity and powers of an individual…. A company also
has all the powers of a body corporate”.

2.2 Concepts of separate corporate personality and limited liability

A company exists as a separate legal entity. It is separate to its members and to its directors
(Salomon’s)
A person can operate in a dual capacity in a company eg as D and as employee. Company is separate
from its controllers (Lee v Lee’s Air Farming)
* Salomon v Salomon and Co [1897]

Facts: Leather and boot company with 20,007 shares (Act required seven subscribers, so Mr Salomon held 20,001 and his wife and 5
children had one each). Sold business to his company for shares and cash. £10,000 remained payable to Mr Salomon. £10,000 debt was
secured through issue of debentures secured by a floating charge covering all assets. He was majority shareholder and primary creditor
(owed £10,000) when the company was wound up. Asset sales were less than £10,000 and unsecured creditors received nothing

Held: House of Lords held the company was a separate legal entity to its members and directors. The assets of the company are not the
assets of the members. Contracts entered into by the company do not bind the members. They create rights and liabilities that vest in the
company, not its members.

Reasoning: The business might be exactly the same as it was before incorporation, with the same controllers and the same hands receiving
the profits. But the company is not in law the agent or trustee for the subscribers. Mr Salomon was not liable except to the extent and in the
manner provided by the Act, so he could not be held liable for the company’s debt over and above the capital contribution so as to expose
him to unlimited personal liability.

* Lee v Lee's Air Farming Ltd [1961]

Facts: Incorporated partnership with two shareholders: Mr Lee and his solicitor. Mr Lee was governing director and had full control. He
was also employed by the company as its chief pilot. He was killed and his wife sued for compensation entitlement to widows. Rejected
because he couldn’t be employee and governing director.

Held: Privy Council held that a person can operate in a dual capacity in a company. Company is separate from its controllers.

9
THEORY: Concepts of separate corp personality and limited liability
This doctrine sets the registered company and other corporations apart from unincorporated forms
of business association (e.g. partnership).

 Recognition of the corporation as an entity whose rights and duties are distinct from those
of its members and directors is a precondition to the limited liability that members of most
types of companies enjoy.
Arguably also has the consequence of distancing shareholders from the enterprise and weakening
their responsibility for its affairs.

 Risk of business failure falls on creditors. Hence importance of financial reporting and the
duties upon directors to pay proper regard to creditor interests and prevent company incurring
debts while insolvent.
Nexus between corporate personality and limited liability

 Corporate personality function: marking out an asset pool against which creditors of the
enterprise have prior claims.
 Entity status partitions this asset pool from the personal assets of shareholders and directors of
the company to the claims of the firm’s creditors.
Doctrines of separate legal personality and limited liability seen in Corporations Act:

 s 112: Types of companies


 s 114: Minimum of 1 member (agency – corporation is a concept, needs an organ)
 s 119: Company comes into existence on registration
 s 124: Legal capacity and powers of a company
 s 125: Constitution may limit powers and set out objects
 s 201A: Minimum number of directors
Limits of corporate personality

 Corporate personality does not determine the other privileges and treatment of the corporate
entity in the wider dimensions of the legal attribution of personhood. This varies based on
context (e.g. a real person would die at the end of their life, but a corporation enjoys perpetual
succession).
 Separate personality doctrine and its consequences rest fundamentally upon judicial decisions.

10
2.3 Piercing the veil of incorporation

Rigid formalistic insistence upon the distinction between group and individual personalities may be at
the expense of other policies and values cherished by the legal system.

 In particular situations the courts and legislature pierce the corporate veil, ignoring the
corporate entity and looking instead to those who control it or its affairs to vindicate those
policies or values.
 Although in some cases the veil is also pierced to relieve the company of the legal
consequences of its separate entity status.
The ‘corporate veil’ exists once a company is registered and it separates the company from the
people who formed it (and from those who become its members).
Company Members
Separate legal entity with own Own shares but not a proprietary
assets, liabilities, contracts VEIL interest in the co’s assets
May also be creditor, debtor,
director of co

2.3.1 At common law

Categories of piercing the corporate veil are not:

 Exclusive (may belong to more than one)


 Exhaustive (still undeveloped area of law)
There is no common unifying principle which underlies the occasional decision of courts to pierce the
corporate veil. (Rogers AJA in Briggs v James Hardie)

A. Fraud or improper conduct

To satisfy this category, the whole purpose of incorporation must be to evade existing legal
obligations. The key is the intention of forming the company. (Gilford Motor Co Ltd v Horne)
* Gilford Motor Co Ltd v Horne [1933]

Facts: Horne had a restrictive covenant in his employment K. Horne’s contract was terminated by agreement and he set up a company
competing against Gilford. Ds were Horne’s wife, son and an employee of the business. Horne was the effective manager and controller.
Gilford applied for an injunction

Held: Horne’s attempt to use the co to avoid his existing legal obligation not to compete with Gilford justified lifting the corporate veil.

Reasoning: The company was formed and was carrying on business merely as cloak or sham for the purpose of enabling Horne to commit
the breach of the covenant that he entered into deliberately with the plaintiffs on the occasion of and as consideration for his employment as
managing director.

* Jones v Lipman [1962]

Facts: Lipman entered into a contract to sell his house to Mr and Mrs Jones. Lipman decided that he did not want to proceed with the sale.
He incorporated a new company which he owned and controlled and sold the house to the company. Lipman admitted that this constituted a
breach of contract and offered to pay damages. Mr and Mrs Jones sought specific performance.

Held: The company was a sham. The corporate veil was not pierced here (see Prest v Petrodel)

Reasoning: Lipman sought to rely on Salomon, arguing that the company was now owner of the property. The Joneses relied upon Gilford
and this was accepted by the court. The court described the company as: ‘a device, a sham, a mask which he holds before his face in an
attempt to avoid recognition by the eye of equity’.

11
B. Agency

Degree of control and dominance that SSK had over subsidiary established an agency agreement.
(SSK)
Reasoning: Six questions for determining the existence of an agency relationship (all questions
must be answered in the affirmative) (SSK)
1. Profits treated as the profits of the parent?
2. Were the persons conducting the business appointed by the parent?
3. Was the parent the head and the brain of the trading venture?
4. Did the parent govern the adventure, decide what should be done and what capital should be
embarked on the venture?
5. Did the parent make the profits by its skill and direction?
6. Parent in effectual and constant control?
* Smith Stone and Knight Ltd v Birmingham Corporation [1939] 4 All ER 116

Facts: SSK bought a partnership which it then had incorporated. Subsidiary remained under the full control of SSK: Directors appointed by
SSK; all profits went to SSK (without declaration of a dividend); and o all shares effectively owned by SSK. Subsidiary operated on land
owned by SSK. Local council (Birmingham Corp) compulsorily acquired the land. SSK applied for compensation for the cost of relocating
subsidiary. Tenants without a long-term lease were not eligible for compensation. Subsidiary only had a temporary lease.

Held: The degree of control and dominance that SSK had over the subsidiary established an agency relationship. SSK was entitled to
compensation for disturbance to the business operated by the subsidiary.

However, Australian courts have indicated an anomaly or injustice will not always induce the court to
depart from the strict rule of separate corporate personality.
Rogers AJA in Briggs v James Hardie “Court is more willing to [imply agency] when he strict
application of the principle of separate personality would result in an anomaly or an injustice, but far
from true to say that an anomaly or injustice will always induce the court to depart from strict rule”
However, Australian courts have indicated mere dominance by a parent company is not enough to
justify piercing the corporate veil (see Briggs v James Hardie)

C. Corporate Groups

UK Jurisprudence

Very liberal departure from general principle. Very rare exceptions  single economic entity, thus
DHN had owned the land itself, it was entitled to compensation for loss of business.
DHN Food Distributors Ltd v London Borough of Tower Hamlets [1976]

Facts: Land and operating vehicles used in a single business operation were divided between three companies in a wholly owned corporate
group. Tower Hamlets compulsorily acquired land of subsidiary A to build houses. DHN had to close down. Compensation was already
paid to subsidiary A. DHN could only get compensation too for ‘disturbance of the business’ if it had more than a license interest.

Held: English Court of Appeal held that the separate group structure should not defeat the parent company’s claim for compensation for
disturbance of the business that would otherwise be available if they were conducted under a unitary rather than a group structure.

Reasoning: DHN and Subsidiary A were part of single economic entity. Therefore as if DHN had owned the land itself, it was entitled to
compensation for the loss of business.

Australian Jurisprudence:
Walker v Winborne: (Mason J) subsidiary is its own separate legal person. Enterprise is not for
Australia.
All companies within a corporate group are separate legal entities (IEL v Blackburn)

12
Industrial Equity v Blackburn (1977)

Facts: IE declared a special dividend more than the company’s current year profits. Constitution specified that dividends could be paid if
total profits for the corporate group to which IE was the parent were used.

Held: Utilisation of consolidated accounts does not result in all companies in the corporate group being treated as one company. A company
that seeks to pay a dividend must draw only on its own profits and cannot use the profit of other related companies because all companies
are separate legal entities.

Reasoning: It can scarcely be contended that the consolidation provisions of the Companies Act operate to deny the separate legal
personality of each company in the group. Thus in the absence of a contract creating some additional right, the creditors of company A, a
subsidiary within a group, can look only to that company for payment of their debts.

Creditors must look to the specific company within a group with whom they made a K for its
enforcement (Qintex)
Qintex Australia Finance Ltd v Schroders Australia Ltd (1990)

Facts: Schroders sold currency on behalf of Qintex group and made $11 million profit. Drew a cheque for Qintex Television but was paid
into account of Qintex Finance. Schroders bought the same amount of currency that it had just sold, resulting in $1 million loss which it
appropriated against Qintex Finance’s account.

Held: Schroders could only seek payment for the currency trading debt from Qintex TV. Creditors must look to the specific company within
a group with whom they made a contract for its enforcement.

Consolidated accounts are legal under CA, but that doesn’t mean the corporate veil can be pierced and allow party to pursue another
company in the group for a debt.

Reasoning: Chief Justice Rogers commented on commercial practice versus Salomon and Industrial Equity and said [269]: It may be
desirable for parliament to consider whether this distinction between the law and commercial practice should be maintained.

Cross reference: See also Part 6 (Directors’ Duties), 6.2.2 E

D. Other categories

Tort victims – there is a policy rationale for treating tortious claimants and contractual claimants
differently b/c they can’t choose their tortfeasor based on assessment of risk.
* Briggs v James Hardie (1989)

Facts: Plaintiff was a 71year former mineworker who developed asbestosis. Tried to sue Asbestos Mines Pty Ltd, James Hardie and
Wunderlich (corporate group) for damages. Argued no room left for a truly independent, separately functioning, corporate entity.

Held: The mere exercise of control over a subsidiary by a parent company is an insufficient reason to pierce the corporate veil in the group
situation in Australia. This is obiter, however. Flagged for development.

Actual Decision: NWSSC held there it was arguable that Briggs could make a claim against the parent companies and ∴ he should be given
an extension of time to bring a claim against them, remitted back to District Court to decide according to law. No authority regarding
piercing the corporate veil: it was ultimately rejected under statute of limitations.

Reasoning: Rogers AJA: In everything but name, the holding company and its fully owned subsidiary are one. Holding company
customarily exercises complete dominion and control over the subsidiary. If conclusion that Hardies was principal and Asbestos the agent,
then that conclusion could apply in relation to just about every holding company and fully-owned subsidiary and the principle of limited
liability in relation to the activities of the subsidiaries would be left in tatters

As the law presently stands, in my view the proposition advanced by the plaintiff that the corporate veil may be pierced where one company
exercises complete dominion and control over another is entirely too simplistic. Law pays scant regard to the commercial reality that every
holding company has the potential and, more often than not, in fact, does, exercise complete control over a subsidiary. If the test were as
absolute as the submission would suggest, then the corporate veil should have been pierced in other cases.

A duty of care is owed by parent directly to employees of the subsidiary, rather than having indirect
liability, by piercing the corporate veil. (CSR Ltd v Wren)
CSR Ltd v Wren (1997)

Facts: Mr W awarded damages for mesothelioma caused by inhaling asbestos fibres while employed by a wholly owned subsidiary of CSR.

Held: The subsidiary was part of CSR’s Building Materials Division; the full board and management of subsidiary were employees of
parent; the working conditions were determined according to the policies and directives of the parent. CV pierced.

13
E. General Principle?

“The threshold problem arises from the fact that there is no common unifying principle, which
underlies the occasional decision of the courts to pierce the corporate veil. Although an ad hoc
explanation may be offered by a court which so decides, there is no principled approach to be derived
from the authorities.” (Rogers AJA in Briggs v James Hardie & Co Pty Ltd (1989))

2.3.2 Under statute

s 588V(1): When Holding Company Liable


(a) H was the holding (‘parent’) company at the time when S incurs a debt; and
(b) S was insolvent at that time, or became insolvent by incurring that debt…or debts including
that debt; and
(c) at that time, there were reasonable grounds for suspecting that S was insolvent, or would so
become insolvent; and
(d) (i) H, or one or more of its Ds, was or are aware at the time that there are such grounds for so
suspecting; or (ii) having regard to the nature/extent of H’s control over S, or any other
relevant circumstances, it is reasonable to expect that a holding company in H’s
circumstances, or such a company’s Ds, would have been aware of such grounds
(e) that time is at or after the commencement of this Act
s 95A (1): A person is solvent if, and only if, the person is able to pay all the person’s debts, as and
when they become due and payable
s 95A (2): A person who is not solvent is insolvent
s 588G: Director’s duty to prevent insolvent trading by company. s 588G applies to impose
liability upon a person if:
(a) the person is a director of the company when the company incurs a debt; and
(b) the company is insolvent when it incurs the debt or becomes insolvent because it incurs the
debt; and
(c) when it incurs the debt there are reasonable grounds for suspecting that the company is
insolvent or would become insolvent because it incurs a debt; and
(d) the director is aware at the time the debt is incurred that there are reasonable grounds for
suspecting the company is insolvent or a reasonable person in a similar position in a company
in the company’s circumstances would be so aware
(e) that time is at or after the commencement of this Act
s 9: “director” of a company or other body means
(a) a person who:
i. is appointed to the position of a D; (“de jure directors”) or
ii. is appointed to the position of an alternate D and is acting in the capacity regardless
of the name that is given to their position (“directors under another name”)
(b) unless the contrary intention appears; a person who is not validly appointed as a director if:
i. they act in the position of a D (“de facto directors”); or
ii. the Ds of the company or body are accustomed to act in accordance with the person’s
instructions or wishes (“shadow directors”)
s 9(b)(ii) interpreted as a ‘willingness and ability of parent to exercise control and an actuality of
control, over the mgmt. and financial affairs of subsidiary’. Mere control of how board of subsidiaries
are constituted is not enough (Antico)

14
* Standard Chartered Bank of Australia Ltd v Antico (1995)

Facts: Insolvent trading claim was made against Giant for approx. $30 million. Giant had several directors, including 3 who were appointed
by Pioneer (the biggest single shareholder in Giant at 42% and also a major lender to Giant). When Giant failed to repay a $30 million loan
facility the creditor sued Giant, its directors and Pioneer for insolvent trading.

Held: Pioneer was a shadow director of Giant.

Considered: Parent‘s % of shares in the subsidiary (but this is not determinative). Willingness and ability of the parent to exercise control,
and an actuality of control, over the management and financial affairs of the subsidiary.

Buzzle provides the dividing line btw what is a shadow D and what is not. For the definition to be
satisfied, the Ds of the co must be accustomed to act as Ds of the co in accordance with the person’s
instructions or wishes as to how they should so act.
Buzzle Operations Pty Ltd (in liq) v Apple Computer Australia Pty Ltd [2011]

Facts: Liquidator for Buzzle claimed Apple was shadow D b/c they were so important to B.

Held: B did not est Ds of B accustomed to act on A’s instructions when conducting business.

(1) Ds were not then acting as Ds of B


(2) a Aperosn / co is not w/in definition of s 9(b)(ii) merely b/c they impose conditions with which D feels they should comply eg lender,
mortgagee

Cross reference: See also Part 6 (Directors’ Duties), 6.5


Principles to determine de facto director:
Grimaldi v Chameleon Mining NL (No. 2) [2012]

Roles and functions performed will vary with the commercial context, operations and government structure (to extent it is operative) of the
co

There is no reason why a relationship may not evolve over time into that of a de facto director

Whether a person has acted in the position of a D is a question of substance and simply of how that person has been denominated in or by
the co

A rigid distinction btw a de facto and shadow director cannot be maintained

What is being asked for the making of a value judgment about proper characterisation of what in its context the person in question was
doing

Proof that a person exercises senior mgmt. functions, while ordinarily ‘a necessary condition of acting as a D’ will not necessarily be a
sufficient condition to qualify as a de facto director.

BCI Finances Pty Ltd (in liq) v Binetter (No 4) [2016]

Court applies an objective test in determining whether a person is a de facto director (Smithon v Naggar)

A person may still be de facto D even if co concerned has a properly constituted and functioning board (Grimaldi)

Affirms Buzzle. A person may alternatively / also be a shadow director  generally being accustomed to act in accordance with the wishes
of a person involved a ‘habitual compliance over a period of time’

2.3.3 The Position of Employees upon Insolvency

The position of employees upon insolvency


Summary: any person entering an agreement or transaction with the intention of avoiding paying
employee entitlements is liable to compensate for loss and can also face a penalty of 1,000 penalty
units or 10 years imprisonment or both.

15
s 596AA The object of this Part is to protect the entitlements of a company’s employees from
agreements and transactions that are entered into with the intention of defeating the recovery of those
entitlements.
‘Entitlements’ include:
(a) Wages;
(b) Superannuation contributions;
(c) Compensation for injury;
(d) Holiday pay; and
(e) Retrenchment payments for the employee.
s 596AB A person must not enter into a relevant agreement or transaction with the intention of, or
with intentions that include the intention of:
(a) preventing the recovery of the entitlements of employees of a company; or
(b) significantly reducing the amount of the entitlements of employees of a company that can be
recovered.
Penalty: 1,000 penalty units or imprisonment for 10 years, or both.

s 596C: A person who contravenes s 596AB is liable to compensate for loss.


Prest v Petrodel Resources Ltd [2013] (UK)

Facts: Mr Prest owned a network of offshore companies over which he exercised total management control. The companies owned
residential properties. During a divorce, Mrs Prest made a claim based in part on the value of the real estate. Mr Prest denied they were his
and claimed insolvency. Mrs Prest joined the companies as parties to the dispute

Held: Corporate veil may only be pierced to prevent deliberate abuse of corporate legal personality. There’s also a difference between
concealment and evasion

Concealment principle

- Legally banal and does not involve piercing the corporate veil

- Interposition of a company or perhaps several companies so as to conceal the identity of the real actors will not deter the courts from
identifying them, assuming their identity is legally relevant.

- Court is not disregarding the façade, but only looking behind it to discover the facts which the corporate structure is concealing. This
allows other types of law operate (trust, equity, etc).

Evasion principle

- Court may disregard the corporate veil if there is a legal right against the person in control of it which exists independently of the
company’s involvement

- Company is interposed so that the separate legal personality of the company will defeat the right or frustrate its enforcement.

16
OVERVIEW OF PIERCING THE CORPORATE VEIL EXCEPTIONS
Start: A company exists as a separate legal entity. It is separate to its members and to its directors
(Salomon’s)
Can the corporate veil be pierced?
At common law

 Fraud/improper conduct
o To satisfy this category, the whole purpose of incorporation must be to evade existing
legal obligations. The key is the intention of forming the company. (Gilford Motor
Co Ltd v Horne)
 Agency
o Degree of control and dominance that SSK had over subsidiary established an agency
agreement. (SSK)
o + 6 principles
 Corporate Groups (Aus)
o Walker v Winborne: (Mason J) subsidiary is its own separate legal person. Enterprise
is not for Australia.
o All companies within a corporate group are separate legal entities (IEL v Blackburn)
 A duty of care is owed by parent directly to employees of the subsidiary, rather than having
indirect liability, by piercing the corporate veil. (CSR Ltd v Wren)
General principle? no common unifying principle, which underlies the occasional decision of the
courts to pierce the corporate veil. Although an ad hoc explanation may be offered by a court which
so decides, there is no principled approach to be derived from the authorities.” (Rogers AJA in Briggs
v James Hardie & Co Pty Ltd)
Under statute

 Holding company may be liable under s 588V


 Director may be liable under s 588G
 Both require:
o H / D at time S incurs debt + S insolvent + reasonable grounds for suspecting S
insolvent + H / D aware of suspicion OR reasonably expected to be aware

17
18
3. Corporate Constitution
3.1 What is a corporate constitution?

3.1.1 Historical background

Memorandum and articles of association

English Companies Act 1856 introduced two distinct constitutional documents, generally bound
together in a single document.
1) Memorandum of Association: Governed the relationship between the company and the outside
world (external)

 Included name of company, registered office, limitation of liability, nominal share capital, etc.
 MA was originally unalterable
 Main purpose was to state unalterable ‘objects’ of the company
2) Articles of Association: Domestic regulations of the company that govern its internal
administration

 Companies statutes did not prescribe any particular content for Articles of Association, but
there was a template since 1856 called the Table A articles, which were widely adopted by
companies.
 Sometimes Table A articles were adopted with variations to effect a particular distribution of
control rights in a private company.
 Usually contains detailed provisions relating to the internal organisation of the company:
o Division of corporate powers between the board of directors and general meeting of
the shareholders
o Proceedings of the board and general meetings
o Appointment, removal and remuneration of directors
o Allotment, transfer and transmission of shares
o Declaration of dividends
o Winding up of the company

Replaceable Rules

Sections identified by headings in the Act, s 135:

 A “replaceable rule” or
 A “replaceable rule for proprietary companies and a mandatory rule for public companies”
Table of replaceable rules is contained within s 141.
Replaceable rules: ‘Default provisions’ that apply ‘in default’ of the members adopting a constitution
that provides for a different rule: s135(2)

 Whether a rule is replaceable often depends on whether it is public or proprietary.


 Some replaceable rules only apply to proprietary companies: e.g. discretion to refuse to
register transfer of shares s1072G or removal of directors by members s 203C.
 Some replaceable rules are mandatory for public companies: e.g. appointment of proxies
s249X.
Non-compliance with the replaceable rules is not itself a breach of the Act: s135(3)

19
s 135(3) A failure to comply with the replaceable rules as they apply to a company is not of itself a
contravention of this Act (i.e. provisions about criminal liability, civil liability and injunctions don’t
apply).
Inconsistency between replaceable rules and a company’s constitution?
If a company chooses to adopt its own constitution, that will displace the application to the company
of any inconsistent rule except, in the case of a public company, a rule which is expressed to be
mandatory and which operates therefore as an ordinary provision of the Act for the company.
Companies have three options in relation to the choice of rules to govern their internal
management:
1. Elect to function without a constitution, relying solely upon the replaceable rules
2. Adopt its own constitution to displace or modify the replaceable rules wholly or in part (s
136)
a. Public company which adopts a constitution must lodge a copy with ASIC together
with a copy of any special resolution altering its provisions: ss 117(3), 136(5).
3. If company was incorporated before 1 July 1998, it may retain its memorandum and articles
of association as the constitution of the company to the exclusion of inconsistent replaceable
rules; if the company is a public company, those provisions of the Act which contain a
mandatory rule for public companies apply to them s 135.

3.1.2 Contents of the corporate constitution

What is the constitution?


s 125: Constitution may limit powers and set out objects
(1) If a company has a constitution, it may contain an express restriction on, or a prohibition of,
the company's exercise of any of its powers. The exercise of a power by the company is not
invalid merely because it is contrary to an express restriction or prohibition in the company's
constitution.*
(2) If a company has a constitution, it may set out the company's objects. An act of the company
is not invalid merely because it is contrary to or beyond any objects in the company's
constitution.*
*Repeal of the common law ultra vires doctrine for completed contracts.
s 134: Internal Management of Companies: ‘A company’s internal management may be governed by
provisions of this Act that apply to the company as:

 replaceable rules;
 by a constitution; or
 by a combination of both.’
Company must have a constitution when:

 Companies want to be listed on the ASX


 No liability companies
 Companies limited by guarantee that want to remove the word ‘limited’ from their name
Why would you adopt one?

 Different classes of shares with different voting rights;

20
 Happy with the current version of the replaceable rules and wants to ensure they are not
subject to amendment;
 Convenience of one tangible document containing all internal rules.
Effect of the constitution
Constitution is a statutory contract.
s 140(1) Company’s constitution and any replaceable rules that apply to it have effect as a contract
between:
(a) The company and each member
(b) The company and each director and secretary and
(c) The members themselves under which each person agrees to observe and perform the
constitution and rules so far as they apply to that person.
Replaceable rules do not apply to a proprietary company while the same person is both its sole
shareholder and sole director. This is justified on the ground that such a company has little need for a
formal set of rules governing its internal relationships.
Special rules applying specifically to such companies: ss 198E, 201F and 202C.
Effect is to allow the company to function flexibly in its corporate character through the sole
individual who constitutes and acts for it.

3.1.3 Effect of mandatory rules

If a mandatory rule collides with an inconsistent provision in the company constitution, consider:

 Whether the mandatory rule represents an exclusive regime; or


 Whether the rule and the constitution are concurrent and alternative
Example: s203D Removal of directors by members
Empowers the members in a public company to pass an ‘ordinary resolution’ removing a director
s 203D (1): A public company may by resolution remove a director from office despite anything in:
(a) the company's constitution (if any); or
(b) an agreement between the company and the director; or
(c) an agreement between any or all members of the company and the director.
s 203D also imposes procedural requirements:

 s 203D (2) Must give company two months’ ‘special’ notice of the intention to move the
resolution; and
 s 203D (4) Directors must be afforded the right to put their case to members.
Similarly mandatory rule
s 203E Director cannot be removed by other directors--public companies
A resolution, request or notice of any or all of the directors of a public company is void to the extent
that it purports to:
(a) remove a director from their office; or
(b) require a director to vacate their office.

21
Case law regarding s 203D has found that it is not an exclusive or exhaustive regime, but rather is a
concurrent and alternative method of removing a director (see following cases).
MANDATORY RULE S 203D CASES
s 203D and any equivalent provision in the C creates concurrent and alternative procedures by which
the Ds removal may be effected. s 203D was not held to be an exclusive regime (Link Agriculture)
The words ‘a public co may remove a D from office despite anything in the co’s C’ meant ...
‘notwithstanding’ any provision in the C which would prevent such removal. This stops the
elimination of s 203D, does not stop an alternative means of removing a D. Open to members to use
either procedure (Boldbow).
Where C specifies basic procedure for removal of Ds but does not specify notice periods, the 2 month
period in s 203D(2) applies (Scottish Colonial).
Language and intention (to provide PF to Ds under challenge) is strong and unambiguous (Scottish
Colonial)
The use of the word may suggests it provides a mechanism, rather than the only mechanism, for
removing a D (State Street).
Words ‘despite anything’ operate to override a C to the extent of any inconsistency only. (State
Street)
Link Agricultural Pty Ltd v Shanahan (1998)

Facts: Conflict between statute and constitution. Statute permitted removal of a director by ordinary resolution. Constitution said the
company may by special resolution remove any director. Just under 75% of the shareholders voted in favour of the removal of a director.

Held: Constitutional provision creates a parallel process. 203D was not an exclusive regime, but rather a concurrent and alternative
procedure for removing a director.

Scottish & Colonial Ltd v Australian Power and Gas Co Ltd (2007)

Facts: Constitution provided for removal of a director ‘by resolution’, i.e. ordinary resolution, but no other procedural requirements. A
director called a general meeting proposing resolutions to remove other 4 directors of APG. Directors sought an injunction to restrain the
holding of the meeting on the basis that two months’ notice, as required by s203D, had not been given.

Held: Procedures required by s203D must be taken. Language of s203D(2) was strong, unambiguous. The purposes of the section were
clear: to prevent entrenchment of directors; and to afford procedural fairness to directors who are under challenge. Unlike s 203D(1), it does
not include words like ‘may’.

State Street Australia Ltd In Its Capacity As Custodian For Retail Employees

Discussion of s 203D(1) by Beach J: “[a] public company may ...”. The word “may” is empowering. Significantly, the phrase is not “may
only ...”. The text suggests that s203D(1) provides a mechanism rather than the mechanism” [at 17]

“The words “despite anything” operate to override a constitution to the extent of any inconsistency only.

Nothing more can be read into the words “despite anything ...”. [at 18].

“Nothing turns on the point that s203D is not a replaceable rule.” [at 19]

Essentially saying it may operate in effect like a replaceable rule, but that’s not relevant. Rather it is a mandatory rule that must always be
available, but can operate in parallel to any consistent constitutional provision.

Jervois Mining Ltd [2016]

s203D provides an avenue for directors of public companies to be held to account with safeguards open under s1322(4)’ [at 65]

22
203E: Removal of Directors by Directors
S 203E: a director cannot be removed by other directors in a public company.

 This only applies to public companies – in proprietary companies, it is quite common to have
such a provision.
 Example is the Coca-Cola Amatil Prenup In the future, all non-executive directors must sign a
contract prior to their appointment to the board.
 Coca-Cola would review the director’s performance every 2 years, and if a majority of the
board considered performance unsatisfactory and requested the director to resign, the director
agreed to do so.
 Arguable this is a breach of s203E: could be considered an indirect ‘request… by any or
all of the directors of a public company’ that requires a director to vacate his office.
 Accordingly, it could be void to that extent.

3.2 Alteration of the corporate constitution

3.2.1 Method of alteration

s 135 Replaceable rule can be displaced or modified by a company’s constitution.


s 136(2) A company may adopt or vary the terms of a constitution by special resolution
s 9 Definition: Special resolution is one passed without the support of at least 75% of the votes cast by
members entitled to vote on the resolution who have notice of intention to propose the special
resolution and of its terms.
s 137 Special resolution making an alteration to the constitution comes into effect on the date the
resolution is passed, or such later date as expressed in the resolution

3.2.2 The power to alter is mandatory

The constitution must be prima facie freely alterable. If there is a clause that completely prohibits the
alteration of the constitution, that clause is invalid.
Must always have capacity to alter constitution. Can never be unalterable. Can make it very difficult,
but theoretically must always be amenable to change.
Peters’ American Delicacy Co Ltd v Heath (1939)

Facts: Two types of shareholders: fully and partly paid. The company’s constitution provided that bonus shares could be issued to all
shareholders regardless whether their shares were fully or partly paid. Fully paid shareholders who were the majority shareholders altered
the constitution so that bonus shares were to be given only to fully paid shareholders.

Issue was whether the act was bona fide in the interests of the company as a whole (note: the ‘bona fide’ test has now been replaced by the
decision in Gambatto)

Held: Even though it affected the partly paid shareholders, there was nothing to show that it was oppression or unjust. In fact, the alteration
was bona fide in the company’s interests as a whole. The constitution was prima facie alterable.

23
3.2.3 Limitations on alteration

A Entrenchment by Further Requirements


B Entrenchment through Weighted Voting
C Statutory Entrenchment
D Class Rights
E Fraud on the Minority and Oppression
F Shareholder’s Agreements

A. Entrenchment

There is scope for entrenchment of a constitutional provision against alteration by a special resolution,
by specifying in the constitution a further requirement for its alteration s 136(3).
The further requirement may not be repealed unless the further requirement itself is satisfied s 136(4).
Examples of further requirements:

 Special resolution be passed with a greater majority than 75% of the votes cast
 The consent of a particular person
 A particular condition is fulfilled

B. Weighted voting

Even though weighted voting, in effect, gives D power to prevent his/her own dismissal, this does not
stop the alteration fo the C (Bushell).
Bushell v Faith [1970]

Facts: 3 shareholders, 2 were directors. Each shareholder held 100 shares. English legislation provided that, notwithstanding anything in the
articles, ‘a company may by ordinary resolution remove a director before the expiration of his period of office.’ Articles provided that in the
event of a resolution being proposed at a general meeting of the company for the removal of a director ‘any shares held by that director
should carry 3 votes per share.’ 2 shareholders voted to dismiss a director (200 votes), he voted against it. Should he have 100 or 300 votes?

Held: Article was valid. Parliament only made an ordinary resolution sufficient to remove a director. It had not sought to fetter a company’s
right to issue a share with such rights or restrictions as it thought fit, even if those rights will in effect give a director the power to prevent his
own dismissal

C. Statutory entrenchment

s 140(2) Members can’t be forced to agree to changes to a company’s constitution if it:

 Requires them to take up more shares


 Increases liability to contribute to SCF
 Otherwise pay money to the company
 Increase, impose or restrict transfer rights

D. Class rights

Commonality of interests enjoyed by a particular group of members, which differentiates them from
other members in the company.
Archetypal example: preference shareholders. Preferred right to a dividend stream, over and above
ordinary shareholders.
Class rights are usually minorities in the company. They are therefore particularly vulnerable to an
amendment or variation or abrogation of their rights by majority.

24
Varying class rights
If constitution provides a procedure for varying or cancelling class rights, must follow that s 246B(1)
If not, the default procedure is contained within:
s 246B(2): Varying or cancelling class rights requires an overall special majority and a special
majority of the affected class.
Challenging a variation of class rights
s 246D: members with at least 10% of the votes in the class can apply to court to have the variation
set aside, on the basis that it unfairly prejudices them.

E. Oppression

Statute:
s 232 statutory provision referred to as oppression

 Prevents people operating in a way that may be contrary to interests of members as a whole or
is unfairly discriminatory against members.
 Very powerful statutory remedy: easily pleaded as a cause of action, gives courts lots of
scope.
s 232: the court has jurisdiction to give relief if it is of the opinion that:
(a) the conduct of a company's affairs; or
(b) an actual or proposed act or omission by or on behalf of a company; or
(c) a resolution, or a proposed resolution, of members or a class of members of a company;
is either:
(d) (d) contrary to the interests of the members as a whole; or
(e) (e) ‘oppressive’ to, ‘unfairly prejudicial’ to, or ‘unfairly discriminatory’ against, a member or
members whether in that capacity or in any other capacity.
Common Law:
Two Gambotto tests

1) Alteration does involve expropriation of shares or value of proprietary interest attached to


shares, the majority shareholder would need to prove that it was both:
a) for a proper purpose; and
proper purpose would be to avoid significant detriment to the company.
b) was fair in all the circumstances
Requires substantive AND procedural fairness.
Onus is on the majority shareholder here.

2) Alteration does not result in any expropriation of shares or value of proprietary interest
attached to shares, the alteration is valid unless it was either:
a) beyond the purpose contemplated by the constitution; or
Technically this test is obiter so it’s unclear, but it’s unlikely ultra vires would
apply if contract has concluded. UV abolished by s 125(2). Case was pre-2001 CA.
b) it was oppressive (i.e. not fair in the circumstances or no reasonable board would make that
decision)
Onus is on the complainant for this test.

25
* Gambotto v WCP Ltd (1995)

Facts: IEL held 99.7% of shares. Gambotto held 0.09% and other minority shareholders had less. Alteration to the company’s constitution
that forced minority shareholders to sell their shares to majority shareholder (compulsory acquisition). Price of compulsory acquisition was
$1.80 per share, above market price of $1.37. Gambotto didn’t challenge fairness of the price. He argued WCP should not have the ability
to compulsorily acquire his shares. First instance and NSW Court of Appeal held the transaction was valid, not improper. Gambotto sacked
his lawyers and went to HCA unrepresented. Argued WCP could not by majority expropriate his shares: argued an equitable limitation on
voting rights of minority. And he won (some say this was at the peak of judicial activism in the 1990s Mason court).

Held: HCA majority held that proposed amendment was invalid because there was an equitable limitation on majority voting rights.

Cross reference: Part 7 (Members’ Rights & Remedies).

F. Shareholders’ agreements

Parties can get together outside of constitution and agree to private contract as to how they may or
may not take certain actions to regulate affairs as far as it relates to interests of a company.
Example:

 Company can never say they are never going to issue shares again, that would be against
public policy. Must be potentially amenable to change.
 However, shareholders can agree amongst themselves to never force company to issue new
shares and that’s enforceable by a contract.
Russell v Northern Bank Development Corp Ltd [1992] 3 All ER 161

Facts: Shareholders agreement made to stop the company from issuing any further share capital. Sloppy drafting. Company was named as a
party to the shareholder agreement.

Held: Court severed the company as a part and upheld the rest of the agreement.

Reasoning: Including the company as a party fetters the ability of company to alter its constitution. You can make it hard by entrenchment,
but not impossible.

UK Court of Appeal said that severing the offending part of the contract and upholding the rest was valid. By doing this, they recognised the
validity of collateral contracts (separate from the constitution).

Private parties when they get together as consenting members can agree this and the courts will enforce those agreements.

This is useful for advising minority shareholders before they make an investment – if they are concerned their interests won’t be protected.

Collateral contract can ensure they will not be at the mercy of majority who could abrogate or vary rights.

26
3.3 Effect of the corporate constitution

3.3.1 Statutory contract

Constitution is a statutory contract.


s 140(1) Company’s constitution and any replaceable rules that apply to it have effect as contract
between:
a) The company and each member
b) The company and each director and secretary and
c) The members themselves under which each person agrees to observe and perform the
constitution and rules so far as they apply to that person

3.3.2 Parties to the contract

3.3.3 Limitation on the obligations imposed or rights given

If you join a corporation you must abide by the constitution. In obiter, not every article is enforceable
by members, only if it effects them in their capacity as a member (Hickman).
* Hickman v Kent or Romney Marsh Sheep-Breeders' Association [1915]

Facts: Art 49 required that any dispute between the Association and its members should be referred to arbitration. Hickman attempted to
bring court proceedings against the Association over its refusal to register his sheep. Association argued it was a dispute which ought to be
resolved by arbitration.

Held: Had to go to arbitration first

3.3.4 Enforcement

By members

 s140(1)(c) makes it clear that those provisions of the constitution which create rights or
obligations as between the members will be contractually enforceable.
 Most of the rights and obligations in a constitution will relate to the relationship between
member and company rather than the relationship of one member to other members
* Hickman v Kent or Romney Marsh Sheep-Breeders' Association [1915]

No art can constitute a K btw the co and a 3P.

No right merely purported to be given by an art to a person whether a member or not in a capacity other than that of a member as for eg a
solicitor, promoter, D can be enforced against the co

Arts regulating the rights and obligations of the members as generally as such do create rights and obligations btw them and the co
responsibility.

27
Eley v Positive Government Security Life Assurance Co Ltd (1875)

Facts: Co wants to get rid of E. He sues relying on art 118 (appoints him solicitor). Court of Appeal said those rights are not enforceable
(outside right). Doesn’t affect him in his position as member.

Held: A member could not enforce an article which stated the terms upon which the co would employ the member as co’s solicitor. This
was b/c there was no r’ship btw the P acquiring shares and adopting the art. The art did not confer a membership right

Remedies

 Non-compliance with a company’s constitution or any applicable replaceable rules is not, in


itself, a contravention of the Corporations Act: s 135(3).
 However, each member has the right to seek appropriate declarations and injunctions relating
to the statutory contract
Rule in Foss v Harbottle
The standing at general law of a member of a corporation to sue on behalf of the corporation is subject
to the rule in Foss v Harbottle:
1) the corporation is prima facie the proper plaintiff in an action to redress an alleged wrong to it
or to recover money or damages allegedly owed to it; and
2) the court will not interfere with the internal management of corporations acting within their
powers

By Officers

The constitution of a company has effect as a contract between the company and each director and
company secretary under which the company and the relevant officers agree to observe and perform
the provisions of the constitution, and operative replaceable rules, so far as they are applicable to the
company and the relevant officers: s 140(1)(b)

 Gives officers standing to seek appropriate relief to enforce their rights under the constitution.
 s 140(1)(b) enacted to counter the decision in Beattie v E & F Beattie Ltd [1938] Ch 708.

By the company

3.4 Executive Directors’ Service Contracts

Appointment of managing directors s 201J (replaceable rule): Directors may appoint 1 or more of
themselves to the office of managing director of the company for a period, and on the terms
(including as to remuneration), as the directors see fit’
Powers of managing directors s 198C (replaceable rule):
(1) The directors of a company may confer on a managing director any of the powers that the
directors can exercise.
(2) The directors may revoke or vary a conferral of powers on the managing director
Examples of powers of the MD under a written service contract:

 The managing director must serve the company faithfully and carefully for a specified period;
 Define the managing director’s duties;
 Specify remuneration (including superannuation);

28
 Provide for summary termination by the company in certain circumstances;
 Protect confidential information of the company;
 Contain restrictive covenants after termination of the managing directors’ employment.
Removing managing directors
s 203C (RR): A proprietary company (a) may by resolution remove a director from office.
s 203D (MR for Public Companies): A public company may by resolution remove a director from
office despite anything in the company’s constitution or any agreement.
s 203F (RR): (1) a person ceases to be a managing director if they cease to be a director; (2) the
directors may revoke or vary an appointment of a managing director.
THOERY: Executive Directors Service Contracts
Early Cases: any contract entered into by the board was always subject to the inherent and
overarching authority of the shareholders in the general meeting to remove the person from office.
Modern Approach: the board and general meeting have separate constitutional powers. One is not
subservient to the other.
SUMMARY: EFFECT OF SPECIAL CONTRACTS

 Company always has power to alter constitution


 But, if the terms of an executive directors service contract are contained within the
constitution. Altering the constitution may, or may not, give rise to a breach of contract,
depending on a variety of factors.
If you can point to some kind of agreement that goes beyond the scope of a company’s constitution,
then they don’t have the legal right to break that special contract, even if they do have the right to alter
their constitution.
Where are Executive Service Contracts Found

 In the constitution itself;


 In a resolution incorporating the constitution;
 In a separate contract outside the constitution;
 Something that appears to be in the constitution but the court interprets it as a ‘special
contract’
Does a special contract exist?
No breach of contract cause of action available Breach of contract cause action available when
when the terms of the executive service contract the terms of the executive service contract
existed: existed:
Only in the constitution itself (Shuttleworth) In a separate contract outside the constitution
In a resolution incorporating the constitution (Shindler)
(Astoria) Terms are contained in the constitution but court
finds external facts that establish it in an
external contract (Bailey)

29
Shuttleworth v Cox Brothers

Facts: The co appointed a board of Ds for life, fixed under its articles. One of the Ds had failed to account for co money and property. Six
grounds to remove Ds from office, but they needed to add a seventh ground to remove this guy. The co then proposed to amend its articles
so that a D would lose his position if the other Ds requested in writing for him to resign. Common law required that any alteration to the C
must have been made ‘bona fide and be for the benefit of the co as a whole’ (Note: the test changed in Gambotto to a ‘proper purpose’ test).
Mr S, who was targeted by the changes, brought a claim alleging that the alteration of the articles was not bona fide for the benefit of the co
as a whole.

Held: Court held that action must fail because his fate was determined solely by the provisions of the co’s C. With regard to the alteration,
Court held it was bona fide for the benefit of the co and was therefore valid.

Reasoning: C was amenable to change. Co was allowed to add the seventh ground to remove him from his position. No evidence that Mr
Shuttleworth could establish to say there was an outside K: some kind of express or implied agreement that went beyond simply the terms of
the co’s C.

* Read v Astoria Garage (Streatham) Ltd [1952]

Facts: The lines of article 68 from the old Table A: stating: a D can be appointed as managing D but appointment ceases if he ceases to be a
regular D. No express K for the appointment as MD, only a board resolution passed at the first meeting of Ds of the co that Mr Read “be and
he is hereby appointed managing D of the co at a salary….”. Mr Read fell ill and co wasn’t doing well. The board resolved that Mr Read
should be removed and the decision was subsequently approved by the co in general meeting. Mr Read sued for damages for breach of K on
the basis his dismissal breached an implied term in the K of service that he would be given reasonable notice.

Held: Two Ks existed: special K for some terms of executive D’s service and statutory K. Statutory K allowed him to be removed as D, but
it was not inconsistent with the special K.

Reasoning: No cause for wrongful dismissal unless it can be shown that the terms of the K of employment were inconsistent with the
powers to determine a MD’s appointment at a GM. As the co had the power to dismiss, they could breach the K because every K entered
into by the co is subject to the power of the co. There is no cause for wrongful dismissal unless it can be shown that the terms of the K of
employment were inconsistent with the powers to determine a MD’s appointment at a GM. The C gave no right to receive any particular
notice of termination of employment in the event of a resolution at a GM.

* Shindler v Northern Raincoat Co Ltd [1960]

Facts: C adopted Table A, art 68: D can be appointed as MD but appointment ceases if he ceases to be a regular D. Mr Shindler sold the
share capital of Northern Raincoat to Lloyds. As part of the sale agreement, Mr Shindler entered into a separate written K with Northern
Raincoat to serve as its managing D for an agreed salary and for a term f 10 years. Lloyds sold Northern Raincoat equity to another co. The
new owner of Northern Raincoat did not wish to retain Mr Shindler’s services. At an EGM of Northern Raincoat, Mr Shindler was removed
from office as D. Mr Shindler sued for breach of K.

Held: The court adopted the modern approach, assuming that no injunction to prevent removal from office would apply, but that Mr
Shindler was entitled to damages.

3.4.1 Effect of alteration of the articles on the outside contract

If you have a special K incorporating terms of the C and those terms get amended, it could
automatically alter the terms of your special K as well, but it depends on intention of parties. (Bailey)
Bailey v NSW Medical Defence Union (1995)

Dr Bailey was a member of the NSWMDU, an organisation that provided insurance to its members against professional negligence lawsuits.
The C had an objects clause which provided that the purpose of the co was to ‘indemnify its members against professional negligence
claims.’ In 1982, the C was amended to give the co the discretion to refuse to provide assistance to its members at any time (blow out in
number of claims after electric shock and deep sleep therapy scandals, fears that member fees would go through the roof). In the mid-1980’s
Dr Bailey was sued by a patient he treated in the 1970’s and he received assistance from the NSWMDU. Dr Bailey died (suicide) before the
case finished and NSWMDU stopped assistance. Dr Bailey’s estate was ordered to pay the patient over $500,000. The executor, Mrs Bailey
sought to enforce the NSWMDU’s C to pay these costs.
Three issues:
1. Was the NSWMDU under an obligation to provide indemnity insurance for Dr Bailey’s estate?
2. If so, did this obligation form the basis of a ‘separate K’ between the NSWMDU and its members?
3. If so, how did the Cal amendment in 1982 alter that K?

Held: Special K existed. K appears to be in the C, but the Court interpreted it to be a ‘special K’.

30
4. Corporate Organs
Salomon’s Case [1897]: “The company is at law a different person altogether from the subscribers
to the memorandum… the company is not in law the agent of the subscribers or trustee for them...
Nor are the subscribers as members liable, in any shape or form, except to the extent and in the
manner provided by the Act. That is, I think, the declared intention of the enactment.” per Lord
Macnaghten

4.1 Division of powers between the corporate organs

Corporate personality is the starting point.


Original authority of the company to act derives from two organs, acting within the scope of their
powers (two organs):
1. General Meeting of Members
2. Board of Directors
Scope of powers defined through Corporations Act, and the corporate constitution (RR’s, adopted
constitution)

4.1.1 Division a matter of choice

 Division is a matter of choice

4.1.2 Usual division

198A(1) Powers of directors (RR): The business of a company is to be managed by or under the
direction of the directors.
Courts have interpreted s 198A to be construed very broadly to cover all sorts of powers, examples:

 Borrowing money on company’s behalf;


 Capacity to sell company’s assets;
 Capacity to enter into business ventures;
 Capacity to bring legal proceedings on company’s behalf; etc.
However, this is a replaceable rule and it is entirely possible (yet impractical) for members o say the
company should be managed by shareholders collectively at a general meeting.
Table A reg 66, former Corporations Law
(1) Subject to the Law and to any other provision of these regulations, the business of the company
shall be managed by the directors, who may pay all expenses incurred in promoting and forming the
company, and may exercise all such powers of the company as are not, by the Law or by these
regulations required to be exercised by the company in general meeting
(2) Without limiting the generality of sub-regulations (1), the directors may exercise all the powers of
the company to borrow money, to charge any property or business of the company or all or any of its
uncalled capital and to issue debentures or give any other security for a debt, liability, or obligation of
the company or of any other persons.

31
4.1.3 Powers exercised by the general meeting

Some powers can only be exercised by shareholders in a general meeting. These powers cannot fall on
directors, per se.

Powers conferred on the general meeting by the Corporations Act

 Modification or repeal of the constitution or a clause in it by special resolution s136(2)


 Capital decisions, e.g. subdividing or consolidating company’s shares: s 254H
 Reducing company’s share capital s 56B
 Non-binding shareholder vote s 250R(2)
 Altering class rights: default procedure in s246B, Pt 2F.2
 Altering status of company: power falls to general meeting
 Power to removing a director from office s 203D (can have alternative procedures for
removal to make it more onerous, but cannot exclude that procedure that involves
shareholders to prevent entrenchment)
 Director of a public company cannot be removed by other directors s 203E
 Replaceable rules
o Power to elect directors s201G
o Power to remove directors s203C
o Power to set director remuneration s202A
o Power to authorise a member to inspect the books of a company s247D (power held
jointly by the board)
 Gatekeeper functions of the General Meeting under the Corporations Act
o Related party transactions s 208
o Benefits for loss of office s 200B
o Financial assistance s 260B(1)
o Share buy-backs s 257D

Powers conferred on the general meeting by the constitution

In absence of a rule like s 198A, it is not uncommon to have certain powers given to general meeting
as dictated by the constitution. The provisions would reflect the company’s specific needs and is a
matter for agreement within the company.

Requirements of ASX Listing Rules

These are usually managerial decisions.


A s 198A division of power would allow board to do what it wants, but if a company is listed it needs
agreement from general meeting for these decisions:

 Certain new issues of shares [ASX LR 7.1]


 Changes to the nature or scale of the company’s activities [ASX LR 11.1]
 Disposal of major assets [ASX LR 11.2]
 Transactions with persons able to influence the company [ASX LR 10.1, 10.11]

32
Residual powers of the general meeting

Decisions which are not explicitly set out as being exercisable by the board of directors may trickle
down to be exercisable by the general meeting.
Traditionally, the GM has residual power, however s 198A reverses the traditional position.
s 198A is a catch-all provision, so it is hard to find any residual powers that fall outside that.
If you don’t have something as robust as s 198A in the constitution in terms of coverage of the field,
then the general meeting by default has powers.

Reserve powers of the general meeting

Reserve powers are those which must be exercisable by the general meeting in certain circumstances.
Exercisable by the general meeting if:
i. board is unable to act due to deadlock or lack of quorum from disqualification of
directors; or
ii. waiver by shareholders of directors’ breaches of fiduciary duties or ratification of
directors’ acts in excess of power.
Enlivened in instances where the board is unable to act.

4.1.4 Powers exercised by the board

Powers conferred on the board by the Corporations Act

Power of directors under s 198A is a broad, catch all provision, which grants wide power (it is a RR,
so subject to constitution).
Other replaceable rules:

 Power to appoint managing director s201J


 Power to call a general meeting s249C
 Power to appoint the chair of the general meeting in certain circumstances s249U(2)
 Power to set terms and conditions of office for company secretary s204F
 Power to determine dividends s254U(1)

Powers conferred on the board by the constitution

Same as for the shareholders. s 198A is a broad provision, but specific powers could be granted in the
absence of such a catch-all provision. It depends on the company’s specific needs and is a matter for
agreement within the company.

The board’s immunity from general meeting interference

Organic Theory: each organ is


autonomous in the fields where they have
been accorded decision making powers.
[see over page for cases]

33
Establishes board of directors’ immunity from interference by GM (Auto Self-Cleansing)

When a board of directors is given the power to manage the company’s business, that power should
interpreted broadly and given natural and ordinary meaning (Auto Self-Cleansing)

Directors are not agents of the GM: they are managing partners appointed to fill that post by a
mutual agreement btw all s/h (Auto Self-Cleansing)

GM cannot override managerial decisions of the board through resolution, where that power has
been vested in it (John Shaw)
* Automatic Self-Cleansing Filter Syndicate Co Ltd v Cuninghame [1906]

Facts: ASC’s C provided board with: (1) power to manage the business; and (2) power to sell the co’s assets to other companies with
similar objectives (court ultimately held these two powers combined to give them discretion as to who to sell assets to). The C also provided
that Ds could be removed by a special resolution. A major s/h established a co and requisitioned a GM. The purpose of the meeting was to
pass a resolution directing the co’s Ds to cause the co to sell its assets to the newly formed co set up by the major s/h. Resolution passed by a
majority of votes derived almost entirely by the major s/h and his associates. The Ds refused to follow the direction. The s/h sought court
orders forcing the Ds to execute the transfer.

Held: Ds were not bound by wishes of the majority of the members. Court said power to manage co’s business should be interpreted
broadly, should be given natural and ordinary meaning
- To allow the majority of members to overrule the Ds is analogous to removing the Ds from their positions.
- If the members wished to overturn the decision of the Ds, they had to remove them by passing a special resolution (an ordinary resolution
was insufficient under the C).
- Ds are not agents of the GM: they are managing partners appointed to fill that post by a mutual agreement between all s/hs.

Reasoning: C gave the board power to sell co’s assets on terms they saw fit. Court said this should be interpreted broadly, should be given
natural and ordinary meaning. Board did not honestly believe the sale was in the best interests of the co and were entitled to refuse res’n

* John Shaw & Sons (Salford) Ltd v Shaw [1935]

Facts: Three Shaw brothers were Ds and had been defrauding the co for many years. When it was recognised, they formed an agreement
with s/h’s. Action would not be brought against them for malfeasance if they agreed (1) new Ds be appointed so their power was diluted; (2)
accountant appointed to work out how much money had been stolen. Agreed verbally, but two of the three brothers refused to sign in
writing. At EGM, Shaw brothers secured an ordinary resolution that legal proceedings being brought by the board would be withdrawn.

Held: Court said that resolution was not effective.

Reasoning: Decision to bring legal proceedings was a managerial decision according to terms of the co’s constitution that fell to board, not
GM. GM could not override board’s decision to bring legal proceedings.

34
Do directors need to consider precatory (non-binding) resolutions (i.e. shareholder activism)?
If resolution falls w/in managerial powers exclusively vested in mgmt., then precatory resolutions
(non-binding) can be ignored (NRMA v Parker)
If beyond the exclusive powers of mgmt. and there is a proper ground to consider it, the court held
that to meet its DOC, the board must: (1) disseminate resolution for consideration by GM; AND (2)
provide counter-argument as to whether the board agrees with the proposal (NRMA v Parker)

ACCR v CBA [2016] (NRMA v Parker)

Facts: Members proposed to move res’ns that stated it was their intention Ds provide certain env reports in CBA’s Annual Report.

Held: If res’n falls w/in managerial powers exclusively vested in mgmt., then precatory res’ns (non-binding) can be ignored.

Reasoning: Members cannot make res’ns regarding the mgmt of the co simply on the basis of a legitimate interest in how the managerial
powers are exercised. It was not the function of the members to express an opinion as to how a power of the board of directors should be
exercised. Adopts principle from NRMA v Parker.

Non-binding advisory opinions about how the co should be run can be properly ignored by the board of Ds and do not need to be put to GM.
However, if res’n is broader and there is a proper grounding to consider res’n, board is req’d to disseminate for consideration by the GM.

Court said not only is appropriate for the mgmt to speak to the res’n, but also required to put the counter-argument and whether or not board
agrees with proposal to be consistent with DOC.

‘The starting point is the general principle that the shareholders in general meeting cannot interfere in the board’s exercise of powers
which are exclusively vested in the board … . This limitation on shareholder power means that if the company’s constitution gives to the
board the power to manage the company’s business, the directors are exclusively responsible for the management of the company and
shareholders cannot control the directors in the exercise of that power or direct the board by resolution to exercise that power in a
particular way (save for any matters that are within the power of the company in general meeting’. Davies J

‘In our view, Clifton is authority for the fundamental proposition that the shareholders in general meeting have no authority to speak or act
on behalf of the company except to the extent and in the manner authorised by the company’s constitution or any relevant statute, and to an
extent and in a manner consistent with the constitution or statute. This proposition extends to the expression of a “wish” on behalf of the
company, unless it is demonstrated that the meeting has legal authority to represent the company on the relevant subject matter’.

‘… In our view, [McLelland J in NRMA v Parker] was saying, correctly, that the shareholders in general meeting did not have a role to play
in the exercise of powers vested exclusively in the board by passing a resolution which would express an opinion on the exercise of those
powers. That general proposition may be affected by the particular constitution of a company, but it applies in this case’.

35
4.2 Shareholders Acting Collectively: The Company in General Meeting
Who are the members? Membership of a company can arise in a number of ways:

 Agreement in the company’s original application for registration;


 Share transfer;
 Receiving shares issued directly by the company
 Using non-share securities (such as options) to obtain shares in the company

4.2.1 Formal decisions by shareholders

A. Types of general meeting - Part 2G.2

Two types of meetings:


1. Annual General Meeting (AGM)
2. Anything else is an Extraordinary General Meeting (EGM)
Annual General Meetings
Public companies

 Must hold an AGM each calendar year, w/in 5 months after the end of its financial year 250N
 Public companies must provide ‘a reasonable opportunity to ask questions’:
o relating to management [s250S];
o auditor [s250T];
o remuneration report [s250SA] (listed companies)
 Publicly listed companies requirements:
o resolution remuneration report be adopted put to advisory vote [s250R(2), (3)];
o auditor must be present [s250RA];
o submission of written questions to auditor [s250PA]
Proprietary companies: Not required to hold an AGM by under the Act, but may be required under
its constitution.
Fundamental matters for an AGM s250R(1)

 annual financial report


 directors report,
 auditors report
 appointment of auditor
 auditor remuneration
 election of directors

36
B. Convening a general meeting

All AGM’s must be held at a reasonable time and place s249R


Who can convene a general meeting?
1. Board of directors under s 198A power
2. Individual director s 249C (RR)
a. Individual director in a listed company, ‘notwithstanding anything in the company’s
constitution’ s249CA
3. Director at the request of members s 249D
4. Members themselves s 249F
5. The court (in certain circumstances) s 249G eg Totex
3. Directors at the request of members s 249D
Directors must call and arrange to hold a GM on the request of members with at least 5% of the votes
who are entitled to vote at the GM.
Request must:

 Be in writing
 State proposed resolution
 Signed by members making the request
 Given to company
 Company then has 21 days to call meeting, 2 months to hold meeting
If directors fail to call a meeting within 21 days of a s249D request:

 Shareholder requisitionists (50% of the original 5%) who made s 249D request can call and
arrange to hold the meeting under s 249E.
 Directors are potentially liable for costs of the meeting under s 249E.
Requisitionists under s 249D will be constrained by the division of powers doctrine.

 They must state a legitimate object of the meeting (i.e. something that shareholders are
entitled to do) or the directors will be entitled to refuse to convene meeting NRMA v Parker
 s249Q reflects this NRMA v Parker principle, by requiring that ‘a meeting of a company’s
members must be held for a proper purpose.’
4. Members themselves s 249F

 Need support of at least 5% of the votes to be cast at the GM.


 At their expense: must pay for cost of calling and holding meeting
 Advantage? Temporal issue – get to work out the timing. Members can essentially hold threat
of meeting and specific resolution over board.
5. Court (in certain circumstances) s 249G

 Court can call a GM if it is impracticable to call it in any other way


o Court won’t find impracticability easily (high bar) but will where necessary.
 Court will consider this issue on application of any director or member entitled to vote

37
* Re Totex-Adon Pty Ltd [1980]

Facts: One of the two directors absolutely refused to co-operate with the other. Would not return calls or do anything. Needed both to call a
meeting.

Held: Court found it was impracticable to call a general meeting. Court varied quorum requirement and reduced to one director. Allowed
company to break its deadlock. Court won’t find impracticability easily, but will where necessary.

Woolworths v Get up

Facts: Woolworths is largest investor in poker machines in Australia. Get-Up took interest in the company. Caused a general meeting under
s 249D (210 members, before this 100-member rule was scrapped). Wanted company to consider whether it should continue to invest in
poker machines. Woolworths had just had an AGM, board asked to defer issue to next AGM.

Held: Court allowed issue to be deferred to next AGM based on the nature of the issue (i.e. it was actually a management power that had
been vested in the board, so they declined to make a s 249G order)

Reasoning: Court will consider nature of issue in deciding whether to grant a s 249G request: is it something that falls within management
powers exclusively vested in the board? (usually under s 198A)

ACCR v CBA

Members proposed to move resolutions that stated it was their intention that the directors provide certain environmental reports in the
CBA’s Annual Report.

This case recognised that there are limitations of the general meeting in trying to impose its will on the board in situation where the board
has exclusive management powers vested in them (usually under s 198A)

Held that the nature of issues will be considered when requests for general meetings are raised under s 249D or 249E or s 249F, or referred
to court under s 249G.

C. Notice, and conduct, of the meeting

Written notice given individually to:


1. Each member entitled to vote at the meeting and to each director [s249J] and
2. The company auditor [s249K] (needs to turn up to answer questions about accounts)
Amount of notice

 21 days for non-listed companies s 249H


o Constitution can specify longer minimum period of notice s 249H(1)
 Two exceptions:
o Members can consent to a shorter period of minimum notice if 95% of votes to be
cast agree s 249H(2)
o Cannot get shorter notice if public company removing a director from office under s
203D. Must be 2 months’ notice s 249H(3), (4).
 At least 28 days for public listed companies s 249HA
o Constitution may still specify a longer period of notice s 249H(1)
Notice is defective?

 Just because notice is defective does not necessarily invalidate any resolutions. Will depend
on who doesn’t get the notice (1 minor member? lots of members? majority shareholder?).
 Court will consider whether the defect is a procedural irregularity or an issue of substantive
injustice
 s 1322(1)(b): ‘procedural irregularity’ includes the ‘absence of a quorum at certain meetings,
and a defect, irregularity or deficiency of notice or time’

38
o s 1322(3): meeting is not invalid because of an accidental omission to give notice or
non-receiptof notice by any person, unless the court declares the proceedings at the
meeting to be void.
o s 1322(2): proceeding is not invalid because of any procedural irregularity unless the
court considers that the irregularity has caused or may cause the proceeding to be
invalid.
o s 1322(4): enables any interested person to apply to the court for a validation order
with respect to irregularities generally.
o s 1322(6) places conditions on the making of the order.
Content of notice of meeting
Must include (s249L(1)(b)):

 place, date and time;


 the general nature of the meeting’s business;
 any intention to propose a special resolution and the text of any special resolution
 proxy data
Notice must be in a clear, concise and effective manner s249L(3).
Read this requirement alongside common law disclosure obligations in relation to resolutions at
general meetings (next section): must be fair, reasonable and not misleading notice. It is the basis on
which members decide whether or not to attend the meeting.
There is a statutory requirement of s 249L, directors fiduciary duty to have free and fair notices and
common law obligation to have truly informed notice of meetings.
Essentially, notice must be fair and reasonable, sufficient to fully and fairly inform. Standard is not a
commercial lawyer with lots of experience (Fraser v NRMA)
Conduct of the meeting
Quorum: 2 members [s249T (RR)]
Chair: directors may elect an individual to chair meetings [s249J (RR)]
Resolutions:

 ordinary resolutions (can be passed by the majority of those present and voting at the
meeting);
 special resolutions (notice as set out in s249L(1)(c) – i.e. state resolution and intention that it
will be a special resolution - is given and passed by at least 75% of votes cast by members
entitled to vote);
Voting: 1 share, 1 vote [s250E (RR)];

 Questions decided by a show of hands [s250J].


 Members can demand a poll [s250K].
 A poll can be demanded by at least 5 members entitled to vote on the resolution or members
with at least 5% of the votes that may be case, or the chair [s250L(2)]. Proxies are included
Minutes: must be entered within1 month after the relevant meeting in the minute book and signed by
the chair [s251A]
Proxies: members of a public company have the right to appoint a proxy [s249X; RR for Pty]

39
Appointing a proxy
Proxy: person with authority to vote in the interests of the person who appointed them. Appoint them
to attend a GM on your behalf if you can’t make it.
s 250A: Appointment of proxy is valid if it is:

 Signed, or otherwise authenticated, by the member


 Contains: member’s name, address, company’s name, proxy’s name or name of the office
held by the proxy, meetings at which the appointment may be used.
s 249X: Right to appoint a proxy is mandatory in public companies and RR for proprietary.
Both actual people and bodies corporate can be proxies. Once appointed, they have the same rights as
members to speak, vote or demand a poll.
Two types of proxies:

 Directed proxies: specify the way the proxy needs to vote on resolutions
 Undirected proxies: leave the way the proxy needs to vote up to their discretion
Proxy docs are not binding on undirected proxies wrt resolutions (Re Jervois Mining)
Where a chair is also a D – a D will not be liable for breach of fiduciary duties if they fail to vote
directed proxies in their capacity as chair (but note statutory requirement imposed after Whitlam)
*Whitlam v ASIC (2003)

Facts: Nicholas Whitlam (son of Gough) was president and chairman of NRMA’s general meeting. He failed to sign and vote all the
proxies given to him by shareholders who directed him to vote against a resolution to increase directors’ remuneration. Under former s
250A(4) chair must vote in the way directed by proxies. Contravention can expose contravener to civil penalties under the CA.

First instance: Failure to sign the poll paper constituted deliberate attempt to disenfranchise those people who appointed him as a proxy
holder and amounted to a breach of fiduciary duties. 5 year disqualification order.

Court of Appeal: Overturned this. Held that when Whitlam failed to sign proxy votes, he was not exercising his powers as a director, so he
was not in breach of directors’ duties, because he was not acting as a director. He was failing in his position as a proxy holder.

Reasoning: Chair of a meeting who was also chair of the board failed to vote directed proxies. Court held he was not acting in his capacity
as a director when he failed to vote, so he was under no fiduciary obligations to the company. He was only under an obligation to the proxy
donor. Therefore, he did not breach his duties to the company as its director.

Extension of general principle that directors voting their own shares can vote in their own interests, and are not bound by their duty as
directors to act in the interests of the company as a whole.

Statutory requirement imposed after Whitlam v ASIC: a proxy holder is under an obligation to vote directed proxies only when the proxy is
given to the chair of the meeting s 250BB(1)(c)(d).

Re Jervois Mining Ltd; Campbell v Jervois Mining Ltd [2009]

Facts: Proxy document which said that if the member gave proxy to chairman and left it undirected, he intended to vote in favour of every
resolution at the upcoming AGM. He voted against every resolution.

Held: Court held proxy documents were not binding on undirected proxies with regard to resolutions.

Reasoning: If the proxy promised to vote one way, so the member gave them undirected proxy status on that basis, but the proxy decided to
change their mind and vote the other way, the initial promise would not be binding. If you really wanted someone to vote in a particular
way, you would direct them to.

40
D. Disclosure obligations in relation to resolutions at general meetings

Historically, the common law was that a resolution would be invalid if it was based on a “tricky
notice or circular”.
Modern standard: fair and reasonable, sufficient to fully and fairly inform. It must not be misleading
to someone who “looks at it in a quick and cursory way” (Re Marra Developments). This is a high
standard.
Must include (s249L(1)(b)):

 place, date and time;


 the general nature of the meeting’s business;
 any intention to propose a special resolution and the text of any special resolution
 proxy data
Notice must be in a clear, concise and effective manner s249L(3).
Standard is not an experienced commercial lawyer, essentially a lay person (Fraser v NRMA)
The notice must ‘fully and fairly inform members of what is to be considered at the meeting and for
which proxy is sought’. (Fraser v NRMA)
The need to make ‘full and fair disclosure’ has to be tempered by the need to present a document that
would be intelligible to reasonable members of the class to whom it was directed, and that it is likely
to assist rather than confuse. (Fraser v NRMA)
May require directors ‘to take reasonable steps to ascertain relevant information for communication to
members if that information is not known to the board’ (Fraser v NRMA)
Re Marra Developments (1976)

Facts: Notice was sent with 9 proposed res’ns for removal of Ds and a vote of no confidence. Immediately below on the notice, were 2 pro-
management res’ns. Notice was so confusing that members voted for all res’ns, even though some contradicted each other. Clearly no-one
had any idea what they were voting for. Argued the notice was misleading.

Held: The notice was invalid and the res’ns passed were invalid.

Reasoning: Test for contents of notice is that it must be a fair and reasonable representation of what is proposed and that it must not be
misleading to someone who “looks at it in a quick and cursory way”. This is a high standard

* Fraser v NRMA Holdings Ltd (1995)

Facts: Concerned a proposal to restructure NRMA Ltd and NRMA Insurance by ‘demutualisation’, where members of each entity would
exchange membership for shares in a new co, NRMA Holdings. Here, demutualisation means moving away from co limited by guarantee to
one limited by shares. Members were sent a prospectus for holdings which contained notices of meetings convened to consider res’ns to
adopt the reconstruction. Notice was challenged for being insufficient: 2 members of NRMA obtained declarations under s52 of the TPA
that members of both companies were not fully and adequately informed of the proposals to be put before them. No explanation of
disadvantages to demutualisation or changes proposed.

Held: Court held Ds had not satisfied their fiduciary duty to fully and fairly inform the members, so the res’n was invalid

ENT Pty Ltd v Sunraysia Television Ltd (2007)

Facts: PBL buying Swan TV (owned by Sunraysia). Minority s/h objected to quality of info (ENT – 27% stake in Sunraysia but held 44.6%
in Swan’s valued power). Clause: ‘An entity disposing of its undertaking must obtain s/h approval’.

Held: Failed to provide info necessary for disclosure.

Principle: standard of disclosure is one of a normal person who reads on the run.
The adequacy of the info disclosed is to be assessed in a practical, realistic way having regard to complexity of the proposal.
The range of info to be provided is limited by the need to present a document that is understandable to ordinary shareholders.
Ds may themselves have to consider alternatives before committing the co to a major transaction, but they do not necessarily have to
disclose the alternatives to the shareholders. Also do not need to disclose historical trade info they’ve considered.

41
4.3 Informal decisions by shareholders

4.3.1 Unanimous assent

“A company is bound in a matter intra vires by the unanimous agreement of its members” (Lord
Davey in Salomon’s)

 Most instances relate to small proprietary companies


 If s/h attend and vote in GM in full knowledge and consent to some matter that could be
carried out by the GM, assent is binding even though all formalities may not have been
satisfied.
Even if formalities are not complied with, unanimous assent of shareholders is binding.
Broad view: Shareholders expressing Narrow view: Scope of doctrine of unanimous
unanimous assent can also bind the board of assent is limited to powers of general meeting,
directors, even if it’s an exclusive management because it’s based in estoppel. You need to
power of the board. This views the doctrine of show informed assent and it can only be used
unanimous assent as an instance of piercing to cure procedural defects, which does not
the corporate veil and viewing the include failure to give notice of general
shareholders as the company, so they can meeting. This says that separate legal
assent to anything. personality is the more important doctrine over
instances where corporate veil needs to be
I.e. shareholders at general meeting can usurp
pierced. This relies on Auto Self-Cleansing
powers that are given exclusively to board.
which held that each organ is free from
interference from the other organs.
Narrow view is mainly adopted by modern courts in Australia.

A. Under general law

* Re Express Engineering Works [1920]

Facts: There was a D’s meeting where they made a decision to purchase property. The five Ds were also the only s/h. The BOD couldn’t act
b/c they were conflicted out, s/h were only ones who could remedy. Formality of calling GM wasn’t satisfied.

Held: Since Ds were the only s/h, those s/h had in effected assented to the transaction. Decision was binding because even if formalities
were not complied with, unanimous assent of all the shareholders was sufficient to validate the resolutions.

Reasoning: Even though it was purported to be a board meeting, it was sufficient to show the unanimous assent of the shareholders. If all
s/h are in agreement, even if they thought it was a different kind of meeting, it is sufficient to satisfy the doctrine of unanimous assent.

This case is saying that a constitution can give exclusive power of management to the board of directors, but shareholders by unanimous
assent can take usurp that power. Clearly a broad view of the unanimous assent doctrine. Inconsistent with Auto Self Cleansing.

* Re Duomatic Ltd [1969]

Facts: D’s were meant to be remunerated to GM. They just took money from company account from time to time. D’s were guilty of
misfeasance b/c they didn’t go through relevant process i.e. GM.

Held: Dismissed claims, s/h had no rights and so couldn’t effect votes. Applied doctrine of unanimous assent, decision to pay directors was
valid so did not need to repay their salaries.

Reasoning: “If it can be shown that all shareholders who have a right to attend and vote at a general meeting of the company, assented to
some matter which a general meeting of the company could carry into effect (i.e. not any matter, only matters which a general meeting can
carry into effect), then that assent is as binding as a resolution of the general meeting would be.”

Doctrine of unanimous assent only cures resolutions if the power to pass the resolutions in the first place sat with the shareholders in general
meeting. Not if they sat with the board of directors.

42
* Re Compaction Systems Pty Ltd [1976]

Doctrine further developed.

Company issued 20 shares for $1 each. 19 shares were held by its parent company and 1 was held by the solicitor. Parent company was
liquidated. Erroneously thought it was the only shareholder. Purported to have an EGM by itself and resolved to wind up Compaction
Systems. Solicitor held a different class of shares to the parent company. He did not have voting rights. He only had the right to address the
general meeting. Solicitor objected doctrine of unanimous assent was not satisfied, even though he could not vote, so parent company held
100% of votes.

Held: Resolution to wind up the company was invalid. Doctrine of unanimous assent involves more than just 100% assent of those with
voting rights. Other meeting rights need to be satisfied.

Reasoning: NSW SC said right to attend and speak at a general meeting is not an insubstantial right. Gives opportunity to advance
arguments and influence course of discussions. Solicitor was deprived of that right, so there was no unanimous assent.

* Herrman v Simon (1990)

Doctrine further developed again

Facts: Mr H proposed C be altered which had effect of putting s/h in a disadvantaged position. Shareholders tried to change the constitution,
but there were several requirements to do this. Needed to (1) hold three separate meetings; (2) notice of general meeting to all shareholders.
Company went ahead and held a general meeting at which a resolution to change the constitution was unanimously passed. In art because
majority shareholder did not understand the effect of resolution. The majority shareholder is now challenging the resolution, arguing the
notice was insufficient and seeking declaration it was invalid. Other shareholder argued doctrine of unanimous assent. Requirements weren’t
met, but everyone voiced assent.

Held: Informal acts doctrine cannot be relied on to destroy substantive rights. Doctrine of unanimous assent can only cure procedural
defects, not substantive. Principle that should be applied in cases where there is a defect in the passing of a shareholder resolution:

If shareholders who have a right to attend and vote at a general meeting, assent with full knowledge and assent, then that assent is binding.
But it can only go to formality, not substance. That is, doctrine of unanimous assent only cures procedural defects, not substantive defects.

Here, doctrine of unanimous assent didn’t apply for a number of reasons:

1. The shareholders did not have full knowledge or give full assent. It’s a waiver of rights and you can’t waive rights without knowing what
you’re waiving.

2. The right to obtain notice and consider something is an issue of substance. It’s not a procedural defect.

B. Under statute

Informal decision making process under s249A.

 Allows passage of resolutions by shareholders by circulating resolutions, rather than holding a


formal meeting. Convenience purposes.
 Instead of having general meeting, all members who are entitled to vote can sign a written
document on which the resolution is written.
Comes into force when last member signs.
s 249B: Corresponding section for informal decision making for proprietary companies with one
shareholder: just need to decide the resolution and make a note of record that it has been passed.

43
4.3.2 Court's jurisdiction to cure irregularities

It is a precondition to the operation of s 1322 that the irregularity must be procedural irregularity; not
a substantive irregularity. (Cordiant Communications)
s 1322 of the Corporations Act:
s 1322(1)(b): ‘procedural irregularity’ includes the ‘absence of a quorum at certain meetings, and a
defect, irregularity or deficiency of notice or time’
s 1322(3): meeting is not invalid because of an accidental omission to give notice or non-receipt of
notice by any person, unless the court declares the proceedings at the meeting to be void.
s 1322(2): proceeding is not invalid because of any procedural irregularity unless the court considers
that the irregularity has caused or may cause the proceeding to be invalid.
s 1322(4): enables any interested person to apply to the court for a validation order with respect to
irregularities generally. s 1322(6) places conditions on the making of the order.
Irregularities that give rise to substantive injustice vs those that don’t. Substantive injustice requires
the court to make a decision.
* Cordiant Communications (Australia) Pty Ltd v Communication Group Holdings Pty Ltd (2005)

Facts: The s/h who had statutory right to vote at meeting was denied that substantive right and that wrongful denial was not something that
could be considered a procedural irregularity. Special resolution was passed after a chairman's ruling wrongly denied Cordiant its right to
vote. Company asked court to cure irregularity under s 1322 and find resolution was properly passed.

Held: Courts found substantial injustice. It is a precondition to the operation of s1322 that the irregularity must be a procedural irregularity;
not a substantive irregularity

Reasoning: Court held that the ‘best interests of the company’ was not the relevant factor to consider with regard to s 1322. Rather, the
court must consider whether the irregularity was substantive or procedural. Substantive irregularities are those affecting the existence,
extent or enforceability of rights or duties Procedural irregularities merely depart from the proscribed manner, without changing the
substance

4.4 Shareholder access to corporate information


How can shareholders access corporate information?

 Ask questions at the AGM [ss 250S; 250SA; 250T]


o Shareholders to be given reasonable opportunity to ask questions of management of
the company and of the remuneration report.
 Through ASIC
 Statutory right to obtain copies of the company’s constitution [s139] and to inspect minutes of
the GM [s251B]
 Replaceable rules or the constitution [s247D]
 Statutory right of inspection [s247A]
o application to court for right to inspect documents/books.
o court may grant if satisfied applicant acting in good faith and for proper purpose
o information must be kept confidential s247C
 S/h have no general right to access minutes of Director’s minutes  it would interfere with
managerial business
 Access to company registers s 173
o can’t send advertising materials to those members, must be for legitimate company
purpose

44
4.5 Functioning of the board of directors

Types of directors

 Managing director: s 198C


 Alternate director: s 201K
 Executive vs. non-executive directors
 Nominee directors eg representor of interests of employees / creditors / preference s/h

Directors’ meetings

 Convening: s 248C – convene meeting on reasonable notice given to other directors


 Conduct
 Minutes
 Quorum requirements: s 248F
 Passing of D resolutions: s 248G (RR) (1) must be passed by majority; (2) deadlock; chair
has casting vote

Role of the chair

 Chairperson has casting vote; s 248E


ASIC v Rich (2003) 44 ACSR 34; 21 ACLC 450

4.6 Directors’ access to information

Statutory Right: a current director has a right of access to the financial records at all reasonable times
[s290]
Common Law Right: an incident of the director’s fiduciary duty, however the scope of this right has
been subject to debate: Edman v Ross (1922)
Right of Access to Company Books s 198F
A director or former director may inspect the books of the company for the purposes of a legal
proceeding:

 to which they are a party;


 which they propose in good faith to bring; or
 that they have reason to believe will be brought against them.
The right continues for 7 years after ceasing to be a director.

45
46
5. Corporate Contracting
Capacity, Authority and Breach of Duty
[Fady likes this part of the course for conceptual reasons]

5.1 Capacity of the corporation

Common law Statutory law


Doctrine of ultra vires  s 124 legal capacity and powers of both
an individual and body corporate
 s 125 abrogates the intra vires doctrine

5.2 Authority of organs and agents

5.2.1 Contracting by corporate organs and agents

[This WILL be in the exam]


How does a company enter into a contract?
1. Contracting ‘directly’ by organs on its own behalf (seal or not) and witnessed by two
directors (director and company secretary; sole director/secretary): s 127
a. (see also s 129(6) – cure defects in formal authority, won’t give substantive authority)

2. Contracting ‘indirectly’ through agents acting on behalf of the company: s 126


a. Actual Authority – Express or Implied
b. Ostensible or apparent authority – At common law, a company is estopped, as against
a person (being a contractor) acting in good faith, from denying that it had given a
person authority to bind it to a contract in certain circumstances.

DIRECTLY INDIRECTLY
s 127(1) w/o a common seal s 126(1) A company’s power to make, vary,
s 127(2) w a common seal ratify or discharge a K may be exercised by an
Company may execute a document in both individual acting with the company’s express
circumstances if it is signed by: or implied authority and on behalf of the
i. 2 directors; company. The power may be exercised w/o
ii. 1 D and company secretary using a common seal.
iii. 1 D in single s/h / director pty company (2) this section does not affect the operation of a
law that requires a particular procedure to be
complied with in relation to the K

47
5.2.2 Authority at common law

Agency  “… a word used in law to connote an authority or capacity in one person to create legal
relations between a person occupying the position of principal and third parties.” International
Harvester Co of Australia Pty Ltd v Carrigan’s Hazeldene Pastoral Co (1958)
(1) Is that person really an agent? (2) What is the scope of their authority?

A. Actual authority

Actual authority (conferral of authority between principal and agent) may be express or implied
(Hely-Hutchinson v Brayhead Ltd):

Express Given by express words/writing eg when BOD pass resolution authorising two D’s to sign
a cheque eg s 198A
Implied Inferred from conduct of parties and circumstances of the case eg when BOD appoint
MD, impliedly authorising him/her to do MD tasks within scope of office
Incidental authority – suggests once can infer P has given A authority to do whatever
necessary or normally incidental to performance of that particular act
Customary authority – has authority to act within reasonable business practice
Usual authority – authority that attaches to a particular office w/in a corporation or a
position w/in a corporation eg a person appointed to MD carries w it the inference to do
such things falling w/in usual scope of that office (Hely-Hutchinson)
Acquiescence – established through and established course of dealings between the P and
A; authority can be implied from the conduct of the parties in the circumstances of the
case
Conceptual Question: What is the dividing line between implied actual authority through
acquiescence and ostensible authority?

*Hely-Hutchinson v Brayhead Ltd [1968] (UK)

Facts: Suirdale was chair and MD of P co. B co gave P co financial assistance. S joined board of B co. S injected more funds into P co at
request of Richards, B co chair, on two conditions (1) B co indemnify him (2) B co guarantee repayment. Letters signed by R but not given
pursuant to board resolution. P co goes into liquidation and S claims on indemnity and guarantee. B co denies liability saying R had not
authority and S, as D of B co, had notice of that want of authority.

Held: R had apparent authority to make K but also had actual implied authority (by acquiescence)
(R acted as de facto MD, CE who made final decision on matters concerning finance, committed to Ks w/o BOD’s knowledge –
communicated acquiescence, conduct had occurred over many months)

48
B. Ostensible or Apparent authority

[Agency by estoppel]
Ostensible authority is a legal relationship between the principal and contractor that will be binding
where three elements exist (Freeman & Lockyer v Buckhurst):
1. There must exist a representation made to the 3P that the office had authority to enter a K
of the kind which the 3P seeks to enforce on behalf of the company;
2. The representation as to the authority of the office must be made to the third party by a person
or persons who have actual authority to manage the business of the company; and
3. The third party must have been induced by the representation to enter into the contract i.e they
in fact relied on it
A person with only ostensible authority to do an act cannot make a representation which may be relied
on as giving another agent ostensible authority to do that act (Crabtree-Vickers)

* Freeman & Lockyer v Blackhurst

Facts: formed co to develop / property to Kapoor. Hun provided finance. Both equal s/h, D. Co’s constitution allowed them to appoint MD
but never done. Running of company left to Kapoor. Kapoor hired fellow architect (F&L) to draw up laws, no BOD resolution to approve
this K. Co refused to pay F&L fees later on, so they sue. Was the company bound by the K signed by Kapoor?

Held: Ostensible authority but no acquiescence  representation of K to F&L (3P), K has actual authority; F&L entered into K.

While Kapoor did not have actual authority to sign the K, the company was aware of, and had acquiesced in, Kapoor acting as managing
director for the purpose of the development and sale of the company’s property.

Accordingly, BOD represented Kapoor as someone who could enter into a K with F&L, and the company was bound by the K.

* Crabtree-Vickers Pty Ltd

Facts: Bruce Snr, Bruce Jr (who was MD with restricted powers) and their wives form BOD. Snr is founding D and chair, no long FT D.
Involved in exec decisions. Thought of as gov director. Peter (other son), no longer D b/c he was a bankrupt, but still FTE.

Unusual mgmt. agreement – co could be run by BOD proper (actual authority) or a three person sub committee comprised on at least 3 of
these men (importantly this had to include Bruce Snr).

Peter tasked with getting printing machine quotes but had no authority to purchase machine. Bruce Jr (name = MD, effect = not full scope of
authority), gives Peter blank order form. Peter uses it to buy printing machine from C-V. C-V import machine. Bruce Snr gets angry when
he finds out, says no K, Peter had no authority. C-V seek to enforce K.

Held: Against 3P contractor (C-V). Endorses Diplock LJ reasoning in F&L

“… a person (Bruce Jr) with no actual, but only ostensible, authority to do an act or make a representation cannot make a representation
which may be relied on as giving a further agent an ostensible authority. Hence the stress by Diplock LJ [in Freeman & Lockyer] on the
need that the person or persons making the representation must have actual authority to make the representation.”

A  makes representation  B

A must have actual authority for any act of B as agent to be binding

Pacific Carriers Ltd v BNP Paribas (2004)

Facts: Ms Dhiri, manager of the Documentary Credit Department, signed 2 ‘letters of indemnity’, addressed to charterer of a cargo ship,
fixed a special stamp to the letters. Under these letters of indemnity, the bank agreed to indemnify the ship charterer in respect of any loss
or damage the ship charterer might sustain as a result of delivering the cargo without proper documentation being produced by the receiver.

When the cargo was delivered without the proper documents being produced, the ship charterer sued the bank, seeking to be indemnified.
The bank argued that it was not liable on the letters of indemnity because Ms Dhiri had no authority to bind the bank to an indemnity. All

49
parties accepted that Ms Dhiri lacked express actual authority to bind the bank to an indemnity, as it was a function of a different department
of the bank. Required two signatures of A-signatories. She was not even one of those.

Bank argued that the representation must be one made to the ship charterer by the bank about Ms Dhiri, not merely one that Ms Dhiri made
about herself.

High Court held: Ms Dhiri could not hold out herself, but that was an over-simplification. Representation does not need to be positive.
Bank equipped Ms Dhiri with the title, status and facilities and did not implement appropriate safeguards – this is a representation of actual
authority.

5.2.3 Statutory assumptions as to authority

What may 3P/outsiders rely on?

 The doctrine of actual authority (express or implied)


 The doctrine of ostensible (or apparent) authority
 The indoor management rule
 ss128–129 Corporations Act (the “statutory assumptions”)
When is a 3PC entitled to rely on s 129 assumptions?
s 128(1) A person is entitled to make s 129 assumptions in relation to dealings with a company.
The company is not entitled to assert in proceedings in relation to the dealings that any of the
assumptions are incorrect.

 Dealings has been interpreted very broadly: includes “purported dealings”, not just dealings
that are actually authorised: Story v Advance Bank
 Even when you have questionable agency, the substantive law underneath is still enough to
get you within the statutory regime for ‘dealings’
Indoor management rule

 Historically a narrow device relied on by the courts to make good in effect on procedural
irregularity within a company - assume all procedural requirements satisfied
 Two exceptions – actual knowledge, put on inquiry
Northside Developments Pty Ltd v Registrar-General of NSW (1990)

“On the one hand, the rule has been developed to protect and promote business convenience which would be at hazard if persons dealing
with companies were under the necessity of investigating their internal proceedings in order to satisfy themselves about the actual authority
of officers and the validity of instruments.

On the other hand, an overextensive application of the rule may facilitate the commission of fraud and unjustly favour those who deal with
companies at the expense of innocent creditors and shareholders who are the victims of unscrupulous persons acting or purporting to act on
behalf of companies.” per Mason CJ

Statutory Assumptions CA s 129


Constitution and replaceable rules complied with

50
s 129(1) A person may assume that the company’s constitution (if any), and any provisions of this
Act that apply to the company as replaceable rules, have been complied with.
Director or company secretary
s 129(2) A person may assume that anyone who appears, from information provided by the company
that is available to the public from ASIC, to be a director or a company secretary of the company:
(a) has been duly appointed; and
(b) has authority to exercise the powers and perform the duties customarily exercised or
performed by a director or company secretary of a similar company.
Office or agent
s 129(3) A person may assume that anyone who is held out by the company to be an officer or agent
of the company:
(a) has been duly appointed; and
(b) has authority to exercise the powers and perform the duties customarily exercised or
performed by that kind of officer or agent of a similar company.
Proper performance of duties
s 129(4) A person may assume that the officers and agents of the company properly perform their
duties to the company
Document duly executed without seal
s 129(5) A person may assume that a document has been duly executed by the company if the
document appears to have been signed in accordance with subsection 127(1). For the purposes of
making the assumption, a person may also assume that anyone who signs the document and states
next to their signature that they are the sole director and sole company secretary of the company
occupies both offices.
Document duly executed with seals
s 129 (6) A person may assume that a document has been duly executed by the company if:
(a) the company's common seal appears to have been fixed to the document in accordance
with subsection 127(2); and
(b) the fixing of the common seal appears to have been witnessed in accordance with that
subsection.
s 129(1) Constitution and replaceable rules complied with

 Statutory equivalent of Indoor Management Rule


o Example: quorum of four necessary for resolution to enable contract, 3PC can assume
the quorum has been met; Fixing of seal requires resolution, can assume resolution
has been achieved.
 Intended to protect 3PC from information and events beyond scope of reasonable control.
 All caveated on the loss of right to rely (s 128(4))
o Actual knowledge or actual suspicion any of these are wrong
o In the absence of that, this is what you can presume
s 129(2) Director or company secretary

51
 Statutory equivalent of customary authority (i.e. powers normally attached to appointment
– implied actual authority)
o (2)(a) – example: defects in appointment or min share requirements not an issue
o (2)(b) – customary or usual authority of the position
 Director – cannot bind on their own customarily (incl. chair). need express
grant of authority.
 Secretary – administrative contracts, etc
 Statutory equivalent is infinitely more generous to contractor than general law.
o Common law: if 3PC had actually pulled the records and reviewed, that’s ostensible
authority like anything else. Holding out, by a company, on which a 3PC relied
o s 129(2) takes out reliance – very broad scope for reliance. 3PC need not have even
seen something they thought they were relying on – they could assume the authority
s 129(3) Office or agent

 Statutory equivalent of ostensible authority, i.e, Freeman & Lockyear


 ‘Held out by the company’ has been interpreted to be equivalent to the test in Freeman. Still
requires holding out of somebody or something (organ) with actual authority
s 129(4) Proper performance of duties

 Duties: Procedural, fiduciary, statutory, etc


s 129(5) Document duly executed without seal AND s 129(6) Document duly executed with seal
Debate as to how two provisions should be interpreted:

 NO SEAL s 129(5): to the extent that you have a contract that appears to be two authorised
signatures as per s 127 = contract is binding
o Debate – is this enough?
 Document appears to be executed by two authorised persons, no actual
knowledge or suspicion that they’re not authorised – should this be binding?
 One view – that’s enough. Formal authority is enough – appears to
be above board, it’s binding
 Second view says no: you need to prove formal and substantive
authority
o Formal: signature, signed by bob, by jane – were they both
directors? If one was director, but other wasn’t s 129(6) can
save you. But saves signature only. You still need to prove
both have substantive authority.
o Substantive – actual or ostensible authority to bind.
Execution is insufficient. You need underlying agency
authority as well.
 SEAL s 129(6)
o Provisions includes a note back to s 127(2)
 This is a direct link back to direct contracts and how they are executed, which
suggests very strongly that it relates to formal authority as well as substantive
authority (likely to be second view – formal and substantive)
TAKEAWAY POINT: if you are seeking to rely on assumptions and document has not been
executed by D/D or D/CS you can be cured by formal authority under s 129(5) or (6)
But you still need another assumption to cure any defects in agency authority (e.g. ss 129(2) or (3))
i.e. ss 129(5) and (6) are formal authority only. You need substantive authority as well.

52
Exception
CA ss 128–130
* Brick and Pipe Industries Ltd v Occidental Life Nominees Pty Ltd [1992]

Facts: Co (B&P) taken over by a big group of co’s (controlled by Goldberg). After takeover, G became D of B&P but not appointed as MD.
However, he was allowed to act as if he were (de facto). Occidental wanted to enforce a guarantee given by B&P. G’tee given under
corporate seal, w Goldberg signing as D and Mr Furst purporting to sign as secretary as required under the company constitution. BUT Furst
was not secretary. Company collapses, Occidental want guarantee, B&P say no authority. BUT Occ solicitor searched company records and
had queried whether F was entitled to sign as secretary as not shown as secretary in records. Durlacher (financial controller of group of co’s)
assured solicitor F was secretary, paperwork hadn’t been processed. G was present when this was said but said nothing.

Was Occidental entitled to rely on the statutory assumptions to bind Brick and Pipe? Assumptions Occidental argued to relied upon:

s 129(1): resolution had been passed by the board authorising the use of the seal;
s 129(3): Furst was secretary – this was based on the argument that B&P held him out to be the secretary
s 129(6): guarantee had been properly sealed because the company seal had been affixed and appeared to have been signed by two directors
or one director and a secretary.

But did solicitor have actual knowledge or actual suspicion s 128(4)? See below.

s 129(3): holding out (Brick and Pipe)

 Mr Durlacher directly held out Mr Furst: But he didn’t have actual authority – at best he had
ostensible authority only. Can’t hold out.
 Goldberg sitting in meeting and doing nothing: He was effectively holding out Mr Furst as
well
 Goldberg was one director, but he was de facto MD – implied actual authority. Court
accepted this argument.
* BNZ v Fiberi Pty Ltd (1994)

Facts: Company basically had family home at Palm Beach, no other assets. Without knowledge of secretary, Bruce fixed co seal to
mortgage. The seal was affixed and witnessed by Bruce and son who purported to be company secretary.

Defect: secretary is not the company secretary; s 127(2)

What could cure it? s 129(6)

5.3 Breach of duty

5.3.1 General principles

5.3.2 Legislation

CA s 129(4)
CA s 124(2)

FURTHER REFERENCES:

Story v Advance Bank (1993)


Sunburst Properties Pty Ltd (in liq) v Agwater Pty Ltd [2005]
Australia and New Zealand Banking Group Ltd v Frenmast Pty Ltd [2013]
Soyfer v Earlmaze Pty Ltd [2000]

53
54
6. Directors’ Duties
6.0 Introduction

Duties are derived from 3 main sources: Equity: e.g. duty of loyalty and good faith; Common law:
e.g. duties under contract law; Statute: e.g. duty to prevent insolvent trading.
s 185: these duties overlap (means breach can attract remedies from multiple sources).
The duties of directors and other senior officers of a company are usually described as ‘fiduciary’.

The term ‘fiduciary’ refers to notions of trust and confidence  A fiduciary agrees to act for,
or on behalf of, or in the interests of another person the exercise of a power or discretion that will
affect the interests of that other person in a legal or practical sense: Hospital Products Ltd v US
Surgical Corp
Duty to:
 Act bona fide in the interest of the company and for a proper purpose
 Avoid conflict between duty and interest
 Exercise care, skill and due diligence
 Not fetter discretion
 Duties wrt insolvent trading

Policy considerations

 Balance btw supporting initiative and enterprise by D’s and managers with holding them
accountable for decisions and conduct in office (Redmond)
 CLERP: To promote investor confidence, facilitate expansion of capital markets investors
need to be satisfied they’ve had opportunity for redress against corporation and Ds in clear
cases of negligent reckless or fraudulent conduct.
o HOWEVER, Ds who effectively control the corporation, they must not feel so over-
burdened with a fear of responsibility that decision making is constrained

55
6.1 To Whom is the Duty Owed? s 9: ‘director’ of a co or other body means: (a) a person who:
(i) is appointed to the position of a D; or
(ii) is appointed to the position of an alternate D and is acting in that capacity;
Who owes the duties? regardless of the name that is given to their position; and
(b) unless contrary intention appears, a person who is not validly appointed as a
 Statutory obligations apply to director if: (i) they act in the position of a D [de facto]; or (ii) the Ds of the co or
body are accustomed to act in accordance with the person's instructions or wishes.
directors and officers and sometimes [shadow]
employees (and former directors, s 9: ‘officer’ of a corp’n means: (a) a D or secretary of the corp’n; or (b) a
person:
officers and employees) (i) who makes, or participates in making, decisions that affect the whole, or a
 Includes shadow and de facto substantial part, of the business of the corp’n; or
(ii) who has the capacity to affect significantly the corp’n’s financial standing; or
directors and officers. Senior [de facto]
managers could be captured by (iii) in accordance with whose instructions or wishes the Ds of the corp’n are
accustomed to act [shadow]
directors’ duties, depending on (c) a receiver, or receiver and manager, of the property of the corp’n; or
internal governance structures. e.g. (d) an administrator of the corp’n; or
(e) an administrator of a deed of co arrangement executed by the corp’n; or
company secretary, finance officer, (f) a liquidator of the corp’n; or
etc (g) a trustee or other person administering a compromise or arrangement made
btw the corp’n and someone else.

General principle and exceptions

General principle (starting point)


Duty is owed to the company as a whole, not individual shareholders, save certain scenarios
(Percival v Wright)

 A director’s fiduciary duties are owed to the company, not to the shareholders (or creditors)
(Bell Group confirmed Percival v Wright in Australia)
Percival v Wright [1902] 2 Ch 421

Facts: S/h wanted to sell their shares. Ds agreed to buy shares from s/h, incl Mr Percival at £12.50 each. Ds had been negotiating sale of
whole co at >> than that. Ds did not disclose takeover negotiations with s/h. Mr Percival claimed breach of D fiduciary duty – knowing this
would have changed how we made this decision (would have likely got a takeover premium).
Did Ds owe a fiduciary duty to s/h to tell them about takeover negotiations?

Held: duties are owed to co as a whole, nothing to suggest fraud or inappropriate behaviour from BOD, individual s/h approached board,
named their price, got price they wanted. Regret doesn’t affect sale of shares to the co.

Why do duties need to be owed to the company, not individual shareholders?

 The company is the proper plaintiff: Foss v Harbottle


o To the extent that a D breaches duties, to proper plaintiff is the company, not
individual shareholders.
 Any profits recovered via a shareholder derivative suit will be recovered for the company:
o Minority shareholder remedies: if they win, don’t get to keep the money, it goes back
into the pool.
Exceptions to the general rule (VERY RARE)
There may be circumstances where an officer of a co owes fiduciary duties to s/h. But, if so, the duties
arise b/c of the particular circumstance existing btw officer and those s/h, not from the position the
officer holds vis-à-vis the co.
When? In a relationship of mutual trust and confidence it could be said that D’s owe a fiduciary duty
to individual s/h (Coleman v Myers)

 Red flags = vulnerability, reliance, information disparity


 On in small, closely held, family co or converted partnership where there is an expectation
that your counterpart will act in your best interest  look at family history

56
* Coleman v Myers [1977]

Facts: Myers is family patriarch of co, his son was MD. They concoct a scheme which cause family members who are s/h to sell shares to
party which they control at undervalue. They represented to family that this is a true fair price and that they wouldn’t liquidate co assets after
transaction. They get enough acceptance to trigger compulsory acquisition to mop up remaining s/h. M and son have screwed over family.
Try to bring action that Ds breached fiduciary duties.
Were the minority shareholders entitled to bring an action on the basis that the directors had breached a duty owed to them individually?

Held: in favour of family; exceptional circumstances

Woodhouse J rejected argument that Percival v Wright stands for the proposition that directors can never owe a fiduciary duty to individual
shareholders.

Not a blanket rule – certain circumstances where exception may apply.

Woodhouse J: No exhaustive list, but gave some red flags for an exception. Very fact dependent:
- Dependence upon information and advice;
- Existence of a relationship of confidence;
- Significance of some particular transaction for the parties;
- The extent of any positive action taken by or on behalf of the directors to promote a particular transaction;
- The extent of inside knowledge of directors.
- Looking for relationship of trust and confidence – evidence of vulnerability

Brunninghausen v Glavanics (1999)

Facts: two brothers in law have shares in Scheme Imports. B was majority s/h. They fought a lot. G agrees to sell all shares to B. There is an
unsolicited advance to buy all shares in Scheme Imports. B knew what he was doing was morally questionable b/c he got 3P to sign
confidentiality agreement, got them to visit co’s manufacturing warehouse on Sunday when no one present, did not disclose he was in
discussions with prospective purchaser.

Held: facts not as strong as Coleman v Myers but there was such dependence (only 2 s/h) and B thus owed a duty to G.

Court said – relationship between them was such that a fiduciary duty was imposed on B to act in G’s best interests.

Appeal: Doesn’t matter what the actual relationship is – what we are interested in is not whether there is actually an expectation, it’s whether
G is entitled to expect B to act in best interests. B did not assume fiduciary duty for G’s interests – law imposed one. Because of notions of
vulnerability and power imbalances and information asymmetry.

Crawley v Short (2009) 262 ALR 654

Examples of when an independent fiduciary duty may arise between a director and shareholder:

1. Where one shareholder undertakes to act on behalf of another shareholder;


2. Where one shareholder is in a position to have special knowledge and knows that another shareholder is relying on her to use that
knowledge for the advantage of another shareholder as well as herself;
3. Where the company is, in reality, a partnership in corporate guise (quasi-partnership) – this is the point of interest. Essentially a
relationship so close that you presume partner is not going to act against your interests.

57
6.2 Duty to Act in Good Faith for the Benefit of the Company as a Whole and for
a Proper Purpose

s 181(1): a D or other officer of a corporation must exercise their powers and discharge their duties:
(a) In good faith in the best interests of the corporation; AND
(b) For a proper purpose

Bona fide or in good faith Company as a whole Proper purpose


The directors must exercise
their discretion bona fide in
what they consider – not what
a court considers – is in the
interests of the company and
not for any collateral purpose
(per Lord Greene in Re Smith
& Fawcett Ltd; Confirmed in
Australia: Bell Group)

6.2.1 Acting bona fide or in good faith

The directors must exercise their discretion bona fide in what they consider – not what a court
considers – is in the interests of the company and not for any collateral purpose (per Lord Greene in
Re Smith & Fawcett Ltd; Confirmed in Australia: Bell Group)

 Honestly, with the best of intentions (subjective)


 Genuinely, in the sense that the action was not distorted by some irregularity or impropriety
(objective)
* Re Smith & Fawcett Ltd [1942]

Facts: Private co’s C provided that D’s meet anytime at discretion and refuse to register shares. Smith and Fawcett held issued shares, F
died, son wanted shares reg’d in his name. S refused, offered 25%. Son did not accept offer but Smith had absolute/uncontrolled discretion.
Son appealed, dismissed, S refused to register transfer of shares. Could they compel S to register shares of F? Could claim be dismissed by
pointing to company C?

Held: Nothing to show that S had acted in bad faith, provisions in C are very common in proprietary company.

Confirmed in Australia: Bell Group

Australian Metropolitan Life Assurance Co Ltd v Ure (1923)

Facts: Minority s/h (Ure) purchased additional shares and sought to have share transfer reg’d by co, but was refused by Ds. Mr U was
formerly bankrupt/struck off role of solicitors so BOD didn’t like him. In Art 21 of C, BOD had power to refuse registered transfer of shares
w/o reasons to appoint Mr U.

Held: Upheld board’s refusal to register share transfers. Isaacs J common, reasons as to why they exist, want to ensure best people are on
the boards, here perfectly entitled to rely on express provision in company’s Co to refuse to register transfer of shares. Silence in and of
itself is not enough to give rise to an inference of impropriety.

NOTE: s 1072G (RR) provides directors of a proprietary company with the discretion to refuse to register share transfers.

58
6.2.2 Acting for the benefit of “the company as a whole”

Five distinct criteria contributing to meaning of ‘company as a whole’


A Corporators as a general body (default rule – s/h as a group)
B Present and future members (courts usually interpret company as a whole to be current s/h
– but some circumstances where future s/h could be considered)
C Creditors (insolvency changes everything. Can’t pay debts ∴ different rules apply)
D Other stakeholders: e’ees, customers etc
E Group companies

A. The corporators as a general body

Where there are preference and ordinary shares ... the question which arises is sometimes not a
question of the interests of the company at all but a question of what is fair as between different
classes of shareholders (Mills v Mills)
The phrase ‘the company as a whole’ does not mean the company as a commercial entity, distinct
from the corporators: it means the corporators as a general body (Ngurli Ltd v McCann)
* Ngurli Ltd v McCann (1953)

Facts: Clifford formed 4 cos for tax avoidance purposes and transferred 2,000 shares in Southcott Ltd to each co. Purchase price was an
unsecured debt. Each co had 61 issued shares – 1 controlling share held by C; 30 by his sister; 30 by his niece (McCanns). Made himself
gov D through controlling share. Each co received dividends from Southcott – used to repay unsecured debt. C died. Left his 4 controlling
shares in trustee co, in trust, to his bro Horace. Under articles, controlling share lost control when it ceased to be held by C (it ranked equally
with others). Trustee co took steps to transfer voting control to Horace. H issued shares to trustee co in satisfaction of C’s unsecured debt, on
the understanding the shares would be transferred to him – H trying to mimic controlling shares, got back in control but only after allotting
brand new shares to himself.

Held: the D could have taken advantage of the power to benefit himself if such a benefit was incidental to a bona fide exercise of power but
he could not use the power ostensibly to benefit the company but really to benefit himself at the expense of the McCanns.

Directors can benefit from the exercise of their powers where it’s bona fide for the company as a whole. Recognises D are often s/h as well.
Cannot hold your interest above those of all others.

Horace’s duty was to act in the best interests of the s/h as whole and he had failed in that duty. He had considered only his own interest, not
those of the other s/h, the McCanns.

H also breached proper purpose aspect of the duty – he was not issuing shares for any legitimate corporate purpose, such as to raise capital
for the co – merely to benefit himself and remove control of the co from the McCanns.

‘the director could take advantage of the power to benefit himself if such a benefit was incidental to a bona fide exercise of power but he
could not use the power ostensibly to benefit the company but really to benefit himself at the expense of the McCanns’ (Williams ACJ,
Fullagar, Kitto JJ)

B. Present and future members

It is permissible to consider future members as well as current members to the extent that it
affects the corporation as a whole (Gaiman, an extension of Ngurli)
Gaiman v National Association for Mental Health [1971]

Facts: Scientologists (hostile to MH treatment) had applied for membership of the MH org. The org’s governing council invoked power to
expel members who were ‘known or reasonably suspected of being scientologists’

Held: Megarry J said that he considered the phrase ‘interests of the company as a whole’ meant the interests of: present and future members
of the association as a whole, as being a helpful expression of a human equivalent.

The council did act in good faith and in the interests of the association and members as a whole, since Scientology posed a threat to the basis
of the org: e.g. could cause problems in the org b/c they were hostile; loss of revenue and support further down the track etc.

59
C. Creditors

Insolvency changes the interpretation of s 181: it switches the interest of the company from s/h to the
creditors (Kinsela)
Confirmed by Bell Group:
“In a solvent company the proprietary interests of the shareholders entitle them as a general body to
be regarded as the company when questions of the duty of directors arise.”
“It is, in my view, incorrect to read the phrases ‘acting in the best interests of the company’ and
‘acting in the best interests of the shareholders’ as if they meant exactly the same thing. To do so is to
misconceive the true nature of the fiduciary relationship between the director and the company. And it
ignores the other range of interests that might…legitimately be considered.”
* Kinsela v Russell Kinsela Pty Ltd (1986)

Facts: K family operated a funeral business through a co which included a type of insurance to cover the cost of funeral services. The co
signed a lease with the husband and wife (who were also s/h and Ds in the family co) to rent premises at a price substantially lower than
market value. After it was done, they sought and got unanimous consent from the GM. This occurred when the co was clearly insolvent.

Held: (1) Ds breached duties to the co by engaging in an uncommercial transaction that disadvantaged co’s creditors (2) breach of duty
occurred even s/h approved transaction. Insolvency acts as a bar to s/h ratification.

(see Doctrine of Waiver - Shareholders can’t waive a breach that affects a group other than themselves. Once insolvent: affects creditors).

Spies v R (2000)

Facts: Spies was D of Stirling Nicholas which owed various amounts to different creditors. S caused SN to purchase shares in another co,
which impaired its ability to repay its creditors. S prosecuted and convicted for criminal offence of defrauding the co’s creditors. S appealed
on basis he had no direct relationship to creditors and could not therefore have defrauded them by the transaction.

Did spies have an obligation as director to refrain from acting to detriment of co’s creditors?

Held: Directors should consider interests of creditors. When a co is approaching insolvency, it’s more likely Ds will gamble with creditors
money. As such, they should consider interest of creditors going forward. Gummow J.

- Spies did not owe duties to the co’s creditors – company was solvent
- His duties were owed to the co and the co had a contractual r’ship with the creditors. Any fraud committed by Spies was done to co not to
creditors.

D. Others: employees, customers, community interests etc

Will directors be in breach of duties if they consider interests of other stakeholders?

 Traditionally it would be a breach. Courts are more willing to interpret what would be in the
interests of the company more broadly now (e.g. philanthropic stuff)
Directors can consider stakeholder interests, but management action will amount to breach of
fiduciary duty if there is no possible way that current s/h can benefit from the exercise of power
(Parke v Daily News)
* Parke v Daily News Ltd [1962]

Facts: News co doing poorly – DN published 2 papers struggling to survive. Papers were main asset of the co. Decided co was going to be
wound up. As action of goodwill, gave some of proceeds to soon to be made redundant employees. Parke complains (reduces his dividends)

Held: Directors could not benefit the employees over s/h. It amounted to a breach of duty. Should have sent proceeds from K of sale as
final dividends to s/h.

60
* Teck Corp Ltd v Millar (1973) ‘if [the directors] observe a decent respect for other interests lying beyond those of the company's s/h in
the strict sense, that will not, in my view leave directors open to the charge that they have failed in their fiduciary duty to the company’

Need to justify (directly or indirectly) the management action as benefiting current shareholders in some way.

Will be a breach where there is no possible way that current shareholders can benefit from the management action.

POLICY CONSIDERATION: UK equivalent to duty to act in good faith for the benefit of the
company as a whole and for a proper purpose
We have considered whether it is permissible to consider stakeholder interests as being for the benefit
of the company as a whole. But should stakeholder interests be considered?
UK has been quite pioneering: enacted pure stakeholder model:
Companies Act 2006 (UK) s 172: Duty to promote the success of the company
A director of a company must act in the way he considers, in good faith, would be most likely to
promote the success of the company for the benefit of its members as a whole, and in doing so have
regard (amongst other matters) to:
(a) the likely consequences of any decision in the long term;
(b) the interests of the company’s employees;
(c) the need to foster the company’s business relationships with suppliers, customers and others;
(d) the impact of the company’s operations on the community and the environment;
(e) the desirability of the company maintaining a reputation for high standards of business
conduct; and
(f) the need to act fairly as between members of the company
Intended to capture all scenarios considered above under our common law.
Discussion:

 If you are a director who needs to abide by this provision when making a decision, is this
pandering? Are these criteria that directors consider anyway?
o It’s a non-exclusive list
o Interests are also going to conflict – no hierarchy, which is most important? do you
need to consider all of them all the time?
o How do you enforce this provision as a court? How do you establish breach?
 Everything could be a breach, because they conflict.
 It’s a right without a remedy.
o No prosecutions based on breach of this provision
o It’s not achieving its objective – it’s not forcing directors to consider stakeholder
interests
o Instance of law on the books – sections that do not work. Everyone knows they do not
work. But that doesn’t mean that it’s not necessarily useless.
 Expressive nature of law: nudging directors to think in a particular way
 Can’t enforce it, forcing them to consider stakeholder interests by turning
their mind to it

61
E. Interests of the company in a group of companies

In a corporate group, directors owe a fiduciary duty to the entity to which they are appointed, but
there are very limited exceptions where an enterprise approach is required.
Very narrow circumstance and rare scenario: imminent insolvency of entire corporate group and
decision made against that backdrop. Rare circumstance where it might be more appropriate to
consider corporate group (see Equiticorp).

STRICT APPROACH LIMITED APPROACH


Subjective test: directors must actually turn their “The proper test, I think, in the absence of
mind to interest of individual entity. actual separate consideration [of the individual
company's interests], must be whether an
This is the default approach. Adheres to intelligent and honest man in the position of a
principles of limited liability and the corporate director of the company concerned, could, in
veil. the whole of the existing circumstances, have
reasonably believed that the transactions were
“[T]he emphasis given by the primary judge to for the benefit of the company” (Pennicuick J,
the circumstance that the group derived a Charterbridge Corp Ltd v Lloyd's Bank Ltd)
benefit from the transaction tended to obscure
the fundamental principle that each of the Test is essentially saying: To extent director
companies was a separate and independent can’t prove director actually turned mind to
legal entity, and it was the duty of the what was going on, then it’s appropriate for an
directors to consult its interests and its objective test: what would a reasonable director
interests alone in deciding whether payments in that circumstance have done? If it’s the same,
should be made to other companies” no breach of directors duties.
(Mason J, Walker v Wimborne)

Equiticorp applied an intermediate approach combining the subjective and objective elements

"A preferable view may be that where the directors have failed to consider the interests of the
relevant company they should be found to have committed a breach of duty. If, however, the
transaction was, objectively viewed in the interests of the company, then no consequences would flow
from the breach" (Clarke and Cripps JJA, Equiticorp Finance Ltd (in liq) v Bank of NZ)
Charterbridge Corporation v Lloyds Bank

There may be more scope to consider whether D has breached obligation in circumstances where they haven’t given explicit consideration
but rather taken a group think approach.

Taking the collective interests of the group is ok if an honest and intelligent person in similar circumstances to the D could reasonably
believe that the actions they took was for benefit of subsidiary then no consequences flow in terms of a breach of duty.

Equiticorp Finance Ltd v Bank of New Zealand (1993)

Facts: BNZ lent $200m to Uruz (a company in the Equiticorp Group) to finance a takeover. BNZ subsequently reviews its risk exposure re:
Equiticorp group. Equiticorp Finance Ltd (EFL) and Equiticorp Financial Services Ltd (Aust) (EFSA) apply $50m in liquidity reserves to
reduce Uruz debt to BNZ. Staved off the debts, but EFA and EFSA go into liquidation and their liquidators seek orders that application of
$50 million was in breach of directors duties of the two companies: want the cash back.

Those directors, by taking enterprise approach, have prima facie breached. But if you view objectively the transaction and it’s in
the interests of the company as a whole, then no consequences will flow from that breach.

Kirby dissented fervently: cited long line of case law after Walker v Wimborne that said you cannot consider corporate group as a whole.
B/c every entity has separate creditors. It affects creditor interests. So inappropriate to consider as a corporate group.

62
Statutory power to consider entire corporate group for wholly owned subsidiaries
s 187: A director of a wholly-owned subsidiary will be deemed to have acted in good faith and in the
best interests of the subsidiary if:
(a) The subsidiary’s constitution expressly authorises the director to act in the best interests of
the holding company;
(b) The director acts in good faith in the best interests of the holding company;
(c) Subsidiary is not insolvent and does not become insolvent as a result of the director’s act

 If any issues, will likely be with (a): constitution needs to expressly authorise

6.2.3 Duty to exercise powers for a proper purpose

 Cases on proper purpose predominantly arise in the context of share issues by directors
designed to ward off hostile takeovers.
 A key takeover defence is to issue shares to someone sympathetic with management, making
it difficult for the offeror to gain sufficient acceptances to gain control.
 Share issuances also raise capital for the company.
General approach:
1. Analysing the purpose for which a power may and may not be exercised
2. Determining the purpose of a particular exercise of power
Key question: Was the share issue made to raise capital (a proper purpose) or to defeat a takeover (an
improper purpose)?
What Role Does the Duty to Act for a Proper Purpose Perform?

 Restricts what directors can do to prevent takeovers, where directors have a self-interest in
entrenching themselves.
 Adds an extra restriction to what is generally seen as a subjective duty to act bona fide for the
benefit of the company – it provides some form of protection against the ‘honest lunatic’:
Hutton v West Cork Railway Co
Consequences of breach by directors of the duty to act for a proper purpose?

 Transaction will be voidable.


 The rule in Foss v Harbottle isn’t applied stringently in this area of the law.
o Foss v Harbottle rule: proper claimant is the company itself
 E.g. in Howard Smith v Ampol, King CJ thought that there exists in shareholders a personal
right to be protected against dilution of voting rights.
How Do Courts Determine Whether the PP Duty Has Been Breached?
In Howard Smith v Ampol, the court used a 2-limb process to determine whether PP duty had been
breached:
1. Analyse the power, and as a matter of law, the purpose for which it may be exercised (e.g.
allotment of shares);
2. Analyse as a matter of fact, purpose for which the power has actually been exercised in this
case eg what was moving the majority of BOD to issue shares

63
General principles

 In share transfer cases, the courts will defer more to director’s view of what is bona fide for
the benefit of the company than in share allotment cases Smith & Fawcett
 The courts have rejected the view that the only ‘proper purpose’ for the allotment of shares is
to raise capital, which is immediately needed. Howard Smith v Ampol
 The court will scrutinise the directors’ stated purpose and seek the real purpose for the
transaction Howard Smith v Ampol; Teck Corp Ltd v Miller
 Some purposes are prima facie improper purposes:
o Self-Interest (Ngurli v McCann);
o Defeating a takeover (Howard Smith v Ampol);
o Maintaining directors’ control (Hogg v Cramphorn);
o Interfering with exercise of the majority of their constitutional rights (Hogg v
Cramphorn)
* Howard Smith Ltd v Ampol Petroleum Ltd [1974]

Facts: Howard Smith and Ampol were competing to takeover Miller. Ampol owned approximately 55% of Miller. Miller issued shares to
Howard Smith on the basis it would offer more for the company than Ampol. The dilution of Miller’s shares tuned Ampol’s majority
shareholding into a minority interest, making Howard Smith’s bid more likely to succeed. Ampol sought a declaration that the share issue
was undertaken for an improper purpose.

Held: Miller’s directors were not acting for a proper purpose when they issued shares to assist with Howard Smith’s takeover. The shares
were issued for an improper purpose because it was primarily undertaken to dilute the majority shareholding. Merely because the directors
acted in what they thought was the best interest of the company is not sufficient to render the conduct for a proper purpose.

In allotment cases, subjective belief by directors that they are acting in the best interests of the
company will not be sufficient to avert breach of fiduciary duty if purpose is not a proper one
(Whitehouse v Carlton; Hogg v Cramphorn)
* Hogg v Cramphorn [1967] (UK)

Facts: Co was threatened by takeover by Baxter. Colonel Cramphorn viewed current mgmt. as better for the co. Directors set up a trust for
the co’s employees and issued shares, each with 10 votes to the trust. Directors loaned money to the trust so that it could buy the shares
offered to it. This plan blocked the takeover, as directors, through the trustees, then controlled the votes attached the majority of shares.

Held: Co breached their fiduciary duty, despite the fact that:


- D had acted honourably and were not motivated by unworthy motives for maintaining their jobs;
- no personal advantage;
- honestly believed that they were acting in co’s best interest;
- genuinely considered that giving employees an indirect stake in the co would benefit both the employees and the co

* Teck Corp Ltd v Millar (1973) (Canada)

Facts: Teck wanted control of Afton mines. Teck intended to replace Afton board with its own nominees then cause Afton to enter into an
‘ultimate deal’ for the exploitation of mineral rights with Teck. After Teck obtained majority control, the Ds of Afton (not yet replaced)
entered into ‘ultimate deal’ with Canex. Even before Teck’s takeover bid, Afton had decided to enter into an ‘ultimate deal’ agreement with
Canex.

Held: Directors acted for proper purpose by contracting to frustrate takeover bid.

There may be some extreme circumstances in which defensive tactics will be justified b/c of apprehended damage to the company.

‘courts should apply the general rule in this way: The directors must act in good faith. If they say that they believe there will be substantial
damage to the company’s interests, there must be reasonable groudns for that belief. If there are not, teat will justify a finding that the
directors were actuated by an improper purpose’

64
Whitehouse v Carlton obiter: In certain circumstances, directors’ duties may have attenuated via the
constitution to allow interference with the balance of control, w/o it constituting an improper purpose
* Whitehouse v Carlton Hotel Pty Ltd (1987) (HCA)

Facts: Carlton Hotel co was owned by Whitehouse family. Mr W was the ‘governing director. There were 3 classes of shares:
A: Mr W – unrestricted voting powers
B: Mrs W – voting rights only after Mr W’s death
C: Children – profit but no voting rights
Mr and Mrs W divorced. Mr W issue ‘B’ shares to his two sons to ensure his sons controlled the co after he died. Mr W had falling out with
his sons and directed co to challenge the share issue as being for an improper purpose. Mrs W died and he has realigned with daughters.

Held: share issue invalid b/c it was not for a proper purpose. Mr W’s purpose in issuing the shares was to dilute the control of the co away
from his wife and daughters after his death.

Laid out the test to work out whether something is for an improper test – ‘but for’. Attempts to manipulate voting power by issuing
shares is an improper purpose and invalid.

Directors’ conduct may be regarded as for a proper purpose if its goal can be characterised as not
simply to defeat a bid, but rather to provide s/h with an alternative or superior bid, in which case the
court will uphold the board’s action (Darvall).
Darvall v North Sydney Brick and Tile Co Ltd (1989)

Facts: Norbrik was an unlisted public co with a large tract of undeveloped land as its principal asset. The land was radically undervalued in
the co’s accounts, appearing at a value of $1.5m, when its possible developed value was >$60m. A s/h, Darvall, saw the development
potential and launched a takeover offer for the co at $10/sh (last sale was 87c). Ds viewed Darvall’s offer as too low and entered into a 2
step transaction:
(1) JV comprising 100% subsidiary of Norbrik and Chase under which a new company would purchase the land for $60m;
(2) a competing takeover offer by Mr Lanceley (MD of Norbrik) at $12.50/sh.
Darvall sought orders setting aside the JV agreement.

Held: Ds had not acted for an improper purpose.

Majority rejected idea that defeating takeover will always be for an improper purpose
 Mahoney interprets Howard Smith narrowly as holding that the power to issue shares may not be used to shift power or to benefit a
minority. Distinguished btw transaction for purposes of defeating a takeover (improper) and one prompted by a takeover offer, but
ultimately entered into b/c Ds thought it was in the best interests of the co as a whole (proper purpose)

Dual purposes (Darvall)


Where there are dual purposes, mere presence of impermissible purpose does not necessarily taint
directors’ actions – only if impermissible purpose is dominant purpose.
1. Mills v Mills: ‘substantial object the accomplishment of which formed the real ground of the
board's action’:
2. Whitehouse v Carlton; Hogg v Cramphorn: ‘but for’: Improper purpose was causative in the
sense that but-for that improper purpose, you would not have exercised that power

6.2.4 Statutory duty

CIVIL CRIMINAL
s 181(1) a D or other officer of a corporation s 184(1) A director or other officer of a
must exercise their powers and discharge their corporation commits an offence if they:
duties: (a) are reckless; or (b) are intentionally
(a) In good faith in the best interests of the dishonest;
corporation; AND AND fail to exercise their powers and discharge
(b) For a proper purpose their duties:
Note 1: civil penalty provision; Note 2: s 187 deals w (c) In good faith in the best interests of the
position of Ds of wholly owned subsidiaries corporation; OR (d) For a proper purpose

65
Diakyne Pty Ltd (ACN 099 168 402) v Ralph (2009)

“…In exercising his power as a director of Diakyne to sign a resolution of Diakyne’s board which resolved “to pay the prescribed bonus of
$100,000 to [Colorado] … as per the Colorado [contract]” and an instruction to UBS Warburg authorising a payment of $110,000 from
Diakyne to Colorado, Mr Ralph did not act in good faith for the best interests of Diakyne and for a proper purpose and did improperly
use his position to gain an advantage for Colorado (and, thereby, for himself). Mr Ralph intended to and in fact gained an advantage for
Colorado by ensuring Colorado received the money without Diakyne having any opportunity to debate or dispute the making of the
payment. The actions of Mr Ralph, assessed against the objective standard of impropriety, warrant that description.”

6.3 Directors' Obligations to Avoid Conflict between Duty and Interest

“As fiduciaries, directors must not place themselves in a position in which there is a conflict
between their duties to the company and their personal interests or duties to others. Good faith must
not only be done but must manifestly be seen to be done, and the law will not allow a fiduciary to
place himself in a position in which his judgement is likely to be biased and then to escape liability by
denying that in fact it was biased… the “no conflict” rule is probably the most important of the
director’s fiduciary duties”.
Paul L. Davies, Gower and Davies’ Principles of Modern Company Law (7th Edition) Sweet &
Maxwell 2003 at 391-392 (citations omitted)

6.3.1 Directors' interests in contracts with their own company

A. General law

Director’s interests in Ks with their own company governed by:

 General law (equity  very harsh and inflexible)


 Constitution / replaceable rules
 Statutory provisions
BUT the no conflict duty can be attenuated via the constitution
The fiduciary duty to avoid conflict and duty not to profit:
1. Conflict rule: company directors must not, in any matter falling w/in the scope of their
service, have a personal interest or inconsistent engagement with a 3P, except with the co’s
fully informed consent
2. Profit rule: company directors must not misuse their position for their own or a 3P’s possible
advantage, except with the company’s fully informed consent and therefore they must account
to the company for any gain which they make in connection with their fiduciary office
Except with the company’s full informed consent
1. Provision in the constitution which authorises a director have an interest in a contract with the
company; OR
2. Interested director making full and frank disclosure of their interest in any K with the co;
AND
3. GM approval
Aberdeen Railway Co: if a D had an interest in a corporate transaction, the transaction is voidable at
the co’s will, and it is the duty of Ds to avoid any possibility of conflict. It does not matter if terms of
K are otherwise fair.

66
* Aberdeen Railway Co v Blaikie Bros (1854)

Facts: Very simple direct conflict situation: playing both sides of contractual fence. ARC entered K to buy iron chairs from BB. ARC tried
to get out of K b/c at time of K-ing was Mr Blaikie, a partner in BB.
Was the K voidable upon the ground that one of the Ds was a member of the partnership?

Held: Conflict – Mr B on behalf of BB wanted the highest possible price; as a D on the board for ARC wants to get the lowest possible
price. K was voidable and it was rescinded. Mr B should have given co-Ds ‘full benefit of all the knowledge and skill which he had in
relation to the subject’.

[strict interpretation of the Conflict Rule]

“Real sensible possibility of conflict”

 Objective test by the reasonable man as to whether there is a “real sensible possibility of
conflict”.
 Director in a position of conflict, or possible conflict, does not have to actually pursue or
prefer their personal interest.
Boardman v Phipps [1967] Lord Upjohn: “In my view it means that the reasonable man looking at
the relevant facts and circumstances of the particular case would think that there was a real sensible
possibility of conflict; not that you could imagine some situation arising which might, in some
conceivable possibility in events not contemplated as real sensible possibilities by any reasonable
person, result in a conflict”.
“no contract or arrangement entered into on behalf of the company with any directors, or any firm of
which a director is a member, shall be avoided, nor shall such directors be liable to account to the
company for any profit realised by any contract or work, by reason of such directors holding that
office or of the fiduciary relation thereby established, provided he disclosed the nature of that interest;
but no director shall vote in respect of any contract in which he is concerned.” (Transvaal Land Co)

 This principle replicated in s 194


Key points (Transvaal Land Co):

 The rule against conflicts of interest may be modified by the company’s constitution.
 The rule extends to shareholdings by directors and even indirect holdings as trustee for others.
* Transvaal Land Co v New Belgium (Transvaal) Land and Development Co [1914] Ch 488 [Redmond [7.375]]

Facts: In the company constitution, art 98 provided for D entering in K for which they had an interest. As long as they did not vote on such
issues and provided full and frank disclosure to the BOD, it’s fine. Samuel is D of TLC and a D party called New Belgium. Samuel and
Harvey buy a whole heap of shares in NB. Samuels didn’t vote, but didn’t disclose that he owned shares. Harvey didn’t vote and didn’t
disclose and didn’t disclose that Samuels offered Harvey a gig on BOD of NB (which he took 3 days later). After purchase finalised,
Transvaal figure out that S and H had interest in NB and sought to have parcel of shares set aside.
Does a conflict arise btw the D’s duty to the co and their duty to a trust of which he/she is a trustee?

Held: a conflict arises where a person has an interest as a s/h in company A or is in a fiduciary position towards and owed a duty to
company A which is proposing to enter engagements with company B, where the person is a D.

Where a D has an interest as s/h in another co or is in a fiduciary position towards and owed a duty to another co which is proposing to enter
into engagements with the co of which he is director, a conflict arises.

Curing breach by company consent

 Members’ authorisation or ratification


o Company in GM can with full knowledge of the D’s interest:
 Authorise an interested D to act in what would otherwise be a breach of duty;
or
 Ratify a completed act in breach of duty (Regal (Hastings) Ltd v Gulliver –
honest breach)

67
Methods of attenuating no conflict/no profit rule for direct conflicts
(a) Full disclosure and consent of the General Meeting; Woolworths v Kelly (1991); or -
conflicted director could vote: North-West Transportation Co Ltd v Beatty
(b) A provision in the constitution relaxing the strict duty;
a. i.e. modification of the duty itself amounts to a species of consent, that is it falls
within the scope of “unless expressly provided” in Bray v Ford
(c) Attenuation under s 194 (RR)
If a director of a proprietary company has a material personal interest in a matter that relates
to the affairs of the company at a meeting of directors, then provided that the director has
disclosed the nature and extent of the interest to the directors as required by s 191 (or is
exempt from doing so), then:
 the director can vote;
 any transactions that relate to the interest can proceed;
 the director may retain benefits or profits;
 the company cannot avoid the interest merely because of the existence of the interest.
 No equivalent for public companies. Protectionist nature of CA over public
companies – they have access to voting public, so stricter rules. They could
have something equivalent, but would need to affirmatively draft. Caveat:
director would not be able to vote due to s 195 prohibition.
Judicial and Statutory Constraints on relaxing the "No Conflict" Duty under the Constitution
What can you do and what can’t you do?
1. Strict interpretation of the conditions of any constitutional provision relaxing the general
law requirement needed to waive a breach of duty. Interpretation is black and white: no
reading into the provision; no extending it; no making it suit your purposes. See Guinness.
2. Full disclosure is required: mere disclosure that a director has an interest in the contract is
inadequate – there must be full disclosure of amount of interest, profit etc.
3. Statutory overlay setting out, at a minimum that a director with a conflict of interest must
disclose to the board: ss 191-5
Why conflicts duty is so important. So many corporate scandals and there are statutory overlays to try
to protect, but common law is important too.
Strict interpretation: can’t read into articles of association beyond the exact wording of the
provision. Articles vested power to determine remuneration in entire board. Need consent of entire
board. The court said that it could not relax the rule of equity any further than the articles. (Guinness
plc v Saunders)
Guinness plc v Saunders [1990]

Facts: Guinness appointed a committee of 3 directors, to handle the company’s affairs during a takeover bid for Distillers Company.
Guinness was successful in its bid, though only after (among other things) one of the directors, Ward, had been paid £5.2m. Ward claimed
that this “fee” was agreed among the committee of 3 directors. Guinness’s articles gave power to fix directors’ remuneration to the whole
board, which could then delegate any of its powers. Ward argued that the company’s articles should be construed so that the committee
could be vested with power to pay remuneration to its members. The new owners of Guinness argued that there had been no such power and
the £5.2m had to be paid back.

Held: The House of Lords held that the power to pay remuneration under the articles of the company should be strictly followed.

68
B. Effect of the corporate constitution or replaceable rules

Prior to 2000, used to apply to material personal interest in contract or potential contract. now it is
broader – intending to capture things that weren’t captured earlier.
“Material personal interest” applies to

 Money, contracts, etc


 But what about information from personal relationships between directors on different
companies? Is that knowledge something you could use to material affect the interests of the
company? Not just cash and contracts, very broad. Could read a lot into the definition.
Full and complete disclosure affirmed by resolution

C. Statutory provisions

[VERY IMPORTANT]
Disclosure of interests [Proprietary and public companies]
IMPORTANT ONE: s 191(1): Mandatory rule (which constitution cannot override) that any
director, who has a material personal interest in matters that relate to the affairs of the corporation,
must give the directors notice of it as soon as practicable at a board meeting.

 Consequences of contravention does not affect validity of any act, transaction, agreement,
instrument etc: s 191(4)
 applies to both public and proprietary companies: apex – must have s 191 disclosure, then
from there different paths for public and private companies.

Material: if D’s interest will be Personal interest: Can include indirect advantage
substantially affected by the outcome of the to the D, pecuniary or otherwise
board’s deliberation, it clearly must be An interest which does not give rise to ‘real
disclosed sensible possibility of conflict’ would not be MPI

When notice is not required


s 191(2)

i. membership interests held in common with all other members of the co;
ii. Directors remuneration;
iii. A K requiring s/h approval;
iv. D giving guarantee, indemnity or security in relation to loan; or
v. Ks with a related body corporate, where the D’s interest arises merely b/c the D is a member
of both boards

Interests to be disclosed and content of notice

s 191(3): provides that the notice of disclosure must


(a) Give details of:
i. Nature and extent of interest; and
ii. Relation of interest to the affairs of the co; and
(b) Be at the directors’ meeting as soon as practicable after the D becomes aware of his/her
interest in the matter

69
s 192(1): A director of a company who has an interest in a matter may give the other directors
standing notice of the nature and extent of the interest in the matter in accordance with s 192(2). The
notice may be given at any time and whether or not the matter relates to the affairs of the company at
the time the notice is given.
s 192(2) The notice must: (a) give details of the nature and extent of the interest; and (b) be given: (i)
at a directors' meeting (either orally or in writing); or (ii) to the other directors individually in writing.

Common law and statute


s 193: ss 191 and 192 have effect in addition to (a) general law about conflict of interest; and (b) any
provision in constitution that restricts a D from (i) having material personal interest in a matter; or (ii)
holding office or possessing property.
Voting and completion of transactions [Proprietary companies only]
s 194 (RR): If a director of a proprietary company has a material personal interest in a matter that
relates to the affairs of the company and:
(a) under s 191 the D discloses the nature and extent of the interest and its relation to the affairs
of the company at a meeting of the Ds; or
(b) the interest is one that does not need to be disclosed under s 191;
then:
(c) the D may vote on matters that relate to the interest; and
(d) any transactions that relate to the interest may proceed; and
(e) the D may retain benefits under the transaction even though the D has the interest; and
(f) the company cannot avoid the transaction merely because of the existence of the interest.
Restriction on voting for directors of public companies [Public companies only]
s 195 (MR) (public companies)
(1) Director who has a material personal interest in a matter being considered at a director’s
meeting: must not be present + must not vote.
Must be completely disinterested vote – throwing in comments, attending meeting?
(2) Director may be present and vote if other directors (who do not have a MPI) have passed a
resolution that:
(a) identifies the director, nature and extent of interest in the matter and its relation to the
affairs of the company; and
(b) states that those directors are satisfied that the interest should not disqualify director
from voting.

70
6.3.2 Secret Profits: The Appropriation of Corporate Property, Information and Opportunity

A. General law

The profit rule: a director or senior employee who takes up a business opportunity which is within
the scope of the company’s actual or potential line of business, without the company’s consent,
may be required to account to the company for any profit made or to compensate for any loss
suffered. (Cook v Deeks)

 Ds are not permitted to exercise their powers to secure a corporate benefit for themselves
(Cook v Deeks)
 Ds who are also majority s/h may not be able to approve/ratify their breach of duty (Cook v
Deeks)
The Conflict and Profit Rule: Directors cannot claim as a defence that they:

 Acted honestly and in good faith: Regal (Hastings) Ltd v Gulliver


 Acquired the profit through their own skill, while they happened to be a director: IDC v
Cooley; or
 Caused no loss to the company: IDC v Cooley.
* Cook v Deeks [1916] (UK) [must read case]  diversion of ‘corporate opportunities’

Facts: Toronto Construction Co K’d with Canadian Pacific Railway Co to construct a railway line. K1 was successfully completed. CPR
commenced negotiations w two Ds of TC (Deeks and Hinds) for new K2. Deeks, brother Deeks and Hinds held ¾ capital in TC. Cook held
remainder. Four were only company directors. Dk, Dk and H decide to exclude C from K2 by forming new co Dominion Construction Co.
DC carried out new K. At GM of TC, Dk and H voted to sell part of plant to DC and declare TC had not interest in K2. Cook brought
proceedings.
Could TC be at liberty to claim from 3 defendants benefit of the K which they obtained from CPR?
If yes, could majority of s/h in TC ratify and approve what was done thus releasing a claim against them as D’s?

Held: Majority directors could not retain benefit of the extension of K. Breach of fiduciary duty because extension K was an opportunity
that belonged to TC, not the directors themselves. They learned of opportunity in their capacity as directors of TC.

“…those who assume the complete control of a company’s business must remember that they are not at liberty to sacrifice the interests
which they are bound to protect, and, while ostensibly acting for the company, divert in their own favour business which could properly
belong to the company they represent.”

“Intentionally concealed all circumstances relating to their negotiations until a point had been reached when the whole arrangement had
been concluded in their own favour and there was no longer any real chance that there could be any inference with their plans. This means
that while entrusted with the conduct of the affairs of the company they deliberately designed to exclude, and used their influence and
position to exclude, the company whose interest it was their duty to protect”. Lord Buckmaster LC

Directors may seek relief from the company by giving full disclosure and obtaining ratification by
the company, b/c it did not amount to an actual taking of company property. (Regal (Hastings) Ltd)

71
* Regal (Hastings) Ltd v Gulliver [1967] (UK)  the company’s inability to exploit the opportunities

Facts: Decided in the 40s, reported 20 yrs later. Co R(H) Ltd owned cinema. Had s/h and Ds. It was thought by Ds that to expand interest in
cinema complexes, they wanted to lease properties for cinemas, lessor wanted personal guarantee. Ds weren’t prepared to give guarantee,
lessor said put more capital into co. Extra £3,000 capital, Ds thought co didn’t funds, Ds chipped in themselves. BOD asked chair and
solicitor to chip in. Co is profitable, there’s a change in control, look at books and realise Ds breached fiduciary obligation to co which was
duty to avoid profits. Sued to recover profits from Ds, solicitor and chair. The Ds claimed no breach, acted in good faith.

Held: The four Ds had breached their duty and their profits were accountable, despite the fact they had acted honestly and company had
benefited.

Ds were clearly at all times in a fiduciary r’ship with L(H) ltd and acted upon exclusive knowledge acquired as Ds at L(H) ltd, framed
resolution to chip money in from which they got profit, they sought no authority from L(H) ltd to do so (no ratification) and by reason of
their position and actions, they made large profits which they are liable to account to the co (Viscount Sankey).

Never a suggestion they acted in bad faith, they simply thought L(H) ltd didn’t have capacity to invest. Unfortunately, when investing
themselves, they didn’t convene GM with s/h to ratify their actions. The chairman of the directors, since he did not take the shares
beneficially, was not liable to repay the profit made by those who took the shares from him. Since Garton was not a D he was not in a
fiduciary r’ship to it, and was not liable to make any repayment. He took the shares at the express request of the Ds.

Can directors exploit corporate opportunities which come to them in their ‘private’ capacity?
Can: A director must not make a profit out of property acquired by reason (and only by reason) of his
relationship to the company of which he is a director. c.f. opportunity received outside course of
fiduciary relationship (Peso)
Can’t: MD has very limited or potentially no ‘individual capacity’’. Fiduciary duty to take
opportunity to company. (Cooley)
[Distinction with Peso: company wanted the tender here. Also resignation is intentionally dishonest to
get opportunity.]
Exceptional: Queensland Mines suggests that board consent is sufficient to waive what would
otherwise constitute breach of fiduciary duty.
* Peso Silver Mines Ltd v Cropper (1966)

Facts: Mining Co in Canada. Cropper was MD of Peso. They get prospectors knocking on the door weekly. The board considered Dr Aho’s
claims and rejected. Aho, geologist, approached MD in personal capacity. Agreed to form syndicate to explore mining claim, made lots of
$$$.
Did Cropper breach his fiduciary duty to Peso in developing claims?

Held: Cropper and other Ds acted in good faith, solely in interests of Peso with sound business reasons for rejecting the offer. Cropper had
not acquired any information as a D that was unavailable to prospective purchaser. Dr Aho approached Cropper in private capacity as co-
venturer.

Distinction between Regal Hastings and Peso

 Opportunity in Peso came to D in his individual capacity: only knew about syndicate
opportunity after board meeting. Offer in board meeting was different. Did not come to him
only in course of fiduciary relationship.
 Opportunity in Peso was reactive: did not initiate transaction, he accepted it. Could argue
Regal Hastings was reactive, but less so.
 Type of transaction: Regal expansion of cinemas, very rare opportunity. Peso got those
offers 2-3 times every weeks.
 Issues in the future? Yes. Both Peso and Regal Hastings Ds would need to disclose interest
going forward.

72
* Industrial Development Consultant Ltd v Cooley [1972]

Facts: Cooley was ‘rainmaker’ – made good deals, popular. Celebrated architect and MD of IDC Ltd, he was approached by Gas Board on
behalf of co. They said we like you but not your co, if you do something alone give us a call. Cooley feigns illness, finishes as MD,
miraculously recovers, takes K with Gas Board and makes lots of money
Was Cooley accountable to IDC for his profits as MD?

Held: Cooley was under a duty therefore to disclose all information which he received in the course of his dealings with the gas board.
Instead, he embarked on a deliberate course of conduct which had put his personal interests as a potential contracting party with the gas
board in direct conflict with his pre-existing and continuing duty as MD to Ps.
B/c of breach of duty the D was liable to account to the Ps for all the benefit he had received or would receive under the K with the gas
board.

“Information which came to him while he was managing director and which was of concern to the plaintiffs and was relevant for the
plaintiffs to know, was information which it was his duty to pass on to the plaintiffs because between himself and the plaintiffs a fiduciary
relationship existed ….”. Roskill J at 451

“It is an overriding principle of equity that a man must not be allowed to put himself in a position in which his fiduciary duty and his
interests conflict. The variety of cases where that can happen is infinite. The fact that there has not previously been a case precisely of this
nature with precisely similar facts before the court is of no import. The facts of this case are, I think, exceptional and I hope unusual. They
seem to me plainly to come within this principle.” per Roskill J at 453-4.

* Queensland Mines Ltd v Hudson (1978)

Facts: JV agreement to explore lands for mining in Tasmania. One JV ran out of cash so Hudson took up mining exploration himself – he
told board at board meeting.
Did Qld Mines give its fully informed consent to Hudson to seek the licence?

Held: opportunity to earn these royalties arose initially from the use made by Mr Hudson of his position as MD of Qld Mines. Qld Mines
renounced its interest and assented to Mr Hudson ‘going it alone’ at his own risk and expense and for his own benefit.

Discussion of Queensland Mines

 Queensland Mines seemed to suggest that board consent is sufficient to waive what would
otherwise constitute a breach of fiduciary duty. The people on the board understood that
Hudson was going to pursue in his own right and they consented.
o PC said they weren’t sure: either there’d been no breach to begin with, or it’s been
cured by consent of board.
 Completely inconsistent with long line of case law – to cure fiduciary breach,
needs to be constitutional provision or consent of general meeting
(Woolworths v Kelly)
 Possible distinction: JV to exploit opportunity. Company was Hudson,
Korman and nominees of each. Only rationale (not stated in judgment) is that
the composition of the BOD was identical to the composition of GM.
o This is not good law: consent of GM is required. Convenient tapering of courts here
to achieve desired outcome.
 Quote that suggests the composition of board = composition of GM was the key point: “The
board of Queensland Mines, fully informed as to all relevant facts, had reached a firm
decision to renounce all interest in the exploitation of the licence and had assented to Mr
Hudson taking over the venture for his own account.”
 This case is not as clear as Peso Silver Mines: no individual capacity here, grey comments
going back and forth in the minutes. Nowhere near as firm as Peso which had affirmative
board resolution declining the opportunity. But enough for court to satisfy them that he did
not owe account of profits.

73
Australian Careers Institute Pty Ltd v Australian Institute of Fitness Pty Ltd [2016] [discussed in reasonable detail]

Facts: Confusing!

Did Mr Hornsey’s conduct create a real or substantial possibility of a conflict between his duties as a director of AIF National and his
personal interests in promoting ACI’s business?

Held: Conflict

Per Sackville AJA:

On the primary Judge’s findings, AIF (Vic/Tas) and ACI had overlapping staff with dual roles. His Honour regarded it as “absurd” to
suggest that Mr Hornsey assiduously kept separate the interests of AIF National and the Sage Institute of Fitness. His Honour also found
that the overlap led to a confusion of roles and created a very real risk of redirecting customers of AIF National to the Sage business.
Whether or not it can be said that AIF National had its own “customers” (it did maintain stands at trade shows), it must follow that there
was a serious risk that customers who might otherwise use the services of AIF (Vic/Tas) would be redirected to ACI.

Redirecting business from AIF (Vic/Tas) to ACI would not only harm the former’s business, it would potentially impair, if not destroy, AIF
National’s capacity to promote the business of its Victorian licensee.

In my view, an objective observer, having regard to the terms of the Shareholders Agreement, and the scope of Mr Hornsey’s functions and
responsibilities as a director of AIF National, the primary Judge was entitled to find that there was a real or substantial possibility of such
a conflict.

74
B. Statutory provisions

s 182(1) Improper use of position [very broad]


A director, secretary, other officer or employee of a corporation must not improperly use their
position to (a) gain advantage for themselves or someone else; OR (b) cause detriment to the
corporation. [Consider Doyle v ASIC
Elements:
1. Defendant was at relevant time an officer or employee of corporation; OR
2. Defendant made improper use of his or her position;
3. Defendant made that improper use for the purpose of gaining and advantage or causing
detriment to the corporation; AND
4. Such an advantage was either for the officer or someone else
S 183(1) Improper use of information [even broader = are, or have been]
A person who obtains information because they are, or have been, a D or other officer or
employee of a corporation must not improperly use the information to:
(a) Gain an advantage for themselves or someone else; or
(b) Cause detriment to the corporation
Elements
1. defendant was at the relevant time an officer or employee of the corporation;
2. defendant acquired the relevant information;
3. defendant acquired that information by virtue of the position as officer or
4. employee of the corporation;
5. defendant made improper use of the information;
6. defendant made that improper use for the purpose of:
a. Gaining an advantage or, alternatively,
b. Causing detriment to the corporation; and
7. that such advantage was either for the officer or for someone else
Statutory vs equitable conflict duties

 Statutory provisions are wider than the general law rules


o Apply to any officer or employee who would probably not be regarded as fiduciaries
at general law; and
o s 183 constrains former officers and former employees w/o any time limit
 ss 182 and 183 are narrower than general principles in following respects:
o Statutory provisions apply only where the improper use of info or position ies done
for purpose of gaining an advantage for the officer or employee or any other person
or causing a detriment to the co
o General law duties do not require either a loss or detriment to the co or a profit or
advantage to the officer

75
6.3.3 Related Party Transactions and Ch 2E of the Corporations Act

 Chapter 2E Corporations Act:


o Purpose: s 207
 Basic Requirement:
o need for member approval
for financial benefit: s 208
 Consequences of a breach: s 209
 Exemptions: 2 key provisions:
o s 210: arms length
transactions
o s 211: reasonable
remuneration

Transactions proposed to give a financial benefit (s 229) to a related party (s 228); member
approval required unless excepted:
Expected transactions Member approval process requires:
 Arms' length transactions: s 210 1. Co must prepare a proposed resolution and
 Reasonable remuneration of officers and explanatory statement (s 219) to be lodged
employees: s 211(1) with ASIC 14 day before members are
 Payment of expenses incurred as an notified of meeting (s 218); explanatory
employee or office: s 211(2) statement must give particulars of related
 Indemnities, exemptions or insurance parties involved and fin benefit(s) (s 219)
premiums incurred as an officer: s 212(1) 2. When members notified of meeting, notice
 Payment of an officer’s legal costs in must be accompanied by:
defending an action for liability incurred as a. Proposed res’n and expstatements as
an officer of the co: s 212(2) submitted to ASIC (s 218); and
 Amounts given are <$5000: s 213, Reg b. Any comments ASIC has made thereon
2E.1.01 (in accordance with s 220) (s 221) (not
 Amounts transferred to related parties in resolution cannot be varied from that
their capacities as members and the gift does submitted to ASIC: s 223);
not discriminate btw members: s 215; or 3. The resolution must pass in GM but no
interested party may vote thereon (ss 224,
 amounts transferred to related parties under
225); and
court order: s 216
4. Once passed the co must lodge notice with
ASIC w/in 14 days (s 226)

Note ASIC v Adler (2002): breach of duties are, s 9 (director’s duties), s 180 (duty to act with care
and diligence), s 181 (duty to act in good faith and for a proper purpose), s 182 (improper use of
position), s 182(2) (business judgement rule), s 183 (duty not to improperly use information) and s
260A (financial assistance).

76
6.3.4 Further Potential Conflicts and Conflict Avoidance Obligations

Duty not to fetter discretion

 D cannot validly K (either with one another or with 3P) as to how they shall vote at future
board meetings

 D is bound to act in best interests of company and cannot enter into any arrangements or
agreements with third parties as to how he should act/vote at board meetings.

 Each D owes independent mind and judgment to the company. To do otherwise, is called
fettering their discretion.

 But not interpreted absolutely strictly in case law. Directors can enter into side agreement as
long as at the time they entered into that side agreement that pre-decides how they are going
to vote in the board meetings, that they do so bona fide for the best interests of the
company.
o You still owe the company your independent mind and judgment, it just attaches at
the time of entering the board room.
o Thorby v Goldberg: if, at the time of entering the agreement: they are bona fide of the
opinion that it is in the interests of the company that the transaction should be
entered into and carried into effect, I see no reason in law why they should not bind
themselves to do whatever under the transaction is to be done by the board.
* Thorby v Goldberg (1964)

Facts: agreement to develop plot in Sydney. G tried to get out of K on basis they’d fettered their discretion. At the meeting 3 of the 5
directors are to resign and 2 members of the G Group are to be appointed to the board.

Held: threw out claim. Co’s contracting so it was fine. If at the former time they are bona fide of opinion that it is in the interests of the co
that the transaction should be entered into and carried int o effect, I see no reason in law why they should not bind themselves to do
whatever under the transaction is to be done by the board.

Nominee Directors

 People appointed to be on board of company, by somebody else. Major s/h, major lender,
certain percentage of shares, etc. Constitution allows you to appoint a nominee director to
board.

 Nominee director has been appointed by somebody else. However, nominee directors still
owe duties to company they were appointed to, not to person/entity that appointed you.

 If a D is appointed as a nominee, they may find it difficult both to fulfil their duty to the
company and to act as the nominee of a shareholder or other interest group.

 The overriding interest must be to the company as a whole and not to any other
shareholder.

 A nominee D is not permitted to disregard the interests of the company.


Policy considerations?

 Directors on Company A, B and C


 Divided loyalties
 Too useful, too pragmatic, turn a blind eye

77
s 187 A director of a corporation that is a wholly-owned subsidiary of a body corporate is taken to act
in good faith in the best interests of the subsidiary if:
(a) the constitution of the subsidiary expressly authorises the D to act in the best interests of the
holding company; and
(b) the D acts in good faith in the best interests of the holding company; and
(c) the subsidiary is not insolvent at the time the director acts and does not become insolvent
because of the D’s act.

Competing Directors

 No general and absolute principle to preclude a non-executive director from being appointed
to the board of a competing company  Competing directorships are permissible, with the
caveat that you can’t use confidential information between companies

 D’s conduct is still subject to the fiduciary principles that require the director to not place
himself or herself in a position that gives rise to a real sensible possibility of conflict; and

 D may be bound by an express or implied term in a contract of service which will exclude
acceptance of such a position, breach of that term will lead to equitable accountability, but the
constraint imposed by the term may be avoided by resignation.
London and Mashonaland Co Ltd v New Masonoland Co Ltd (1891)

Longtime authority that directors acting as directors for competing companies won’t necessarily breach their fiduciary duties

Poon Ka Man Jason v Cheng Wai Tao [2016] (HK)

(Spigelman J) Look at the above rule in context.

‘In the light of this considerable body of criticism, Mashonaland and its adoption in Bell v Lever Bros, should be regarded as standing for
no wider a proposition than the trite statement that the law will not interfere in the absence of evidence of a real possibility of breach of
fiduciary duty, including of the conflict rule.’ Per Spigelman NPJ

78
6.4 Duty of Care, Diligence and Skill

6.4.1 Source of the duty

Requirement of duty of care, skill, diligence is very factually dependent on specific role within
company. Examples of breach: failure to monitor; failure to put in place internal controls; failure to
attend board meetings  but it all depends on the facts.
Originally, duty was seen as a tangential incident of original fiduciary duty. More recently, courts and
academics have been more interested sources of duty of care. Very broad sources. Has implication for
standing and the remedies available. (e.g. action against director under equity and common law; but
only ASIC can bring action under statutory equivalent).
General law duties to take care can arise from:

 Contract: Example – executive service contract of MD says must perform functions with due
care skill and diligence. Even if not written, will be implied.
 Equity: Distinction between fiduciary and equitable duties. Strictness of application varies.
 Common Law
 Statute

Contract

Duty of care owed to a company by an executive director employed under a contract of service will
arise from:

 An express term in the contract; or


 From an implied term that the employee will exercise the care and skill to be expected of a
person who occupies the position in question

Equity

A director's duty to exercise reasonable care and skill arises in equity but not a fiduciary duty:
“As directors, I am not aware that there is any difference between their legal and equitable duties. If
directors act within their powers, if they act with such care as is reasonably to be expected from
them, having regard to their knowledge and experience, and if they act honestly for the benefit of the
company they represent, they discharge both their equitable as well as their legal duty to the
company”. Permanent Building Society (in liq) v Wheeler

Negligence

Directors and non-executive directors are subject to a common law duty to exercise reasonable
care and skill in addition to any contractual, equitable or statutory obligations.
“(with respect to non-executive directors) There was no reason why a common law duty to be careful
would not arise simply from the relationship of proximity created by acceptance of the office of
director and the entry on the task of directing the affairs of the company”. Permanent Building
Society (in liq) v McGee

79
Statutory

Must know: s 180(1): a director or other officer of a corporation must exercise powers and
discharge duties with degree of care and diligence that a reasonable person would exercise if they
(a) were a director or other officer of a corporation in corporation’s circumstances; AND (b) occupied
the office held by and with same responsibilities as director / officer.
Note: This subsection is a civil penalty provision (see s 1317E).

Test in s 180(1) is objective – the question is what an ordinary person, with knowledge and
experience of defendant might be expected to have done in the circumstances if he was acting on his
own behalf (ASC v Gallagher)
Reconcile with Daniels v Anderson?

 both say scope and duty vary depending on nature of office and the skills the person held
themselves to have in that office
Factors to consider in determining a breach of s 180(1):

Company’s circumstances Director/officer position/responsibilities in co


 Size of co  Skills and experience of particular D
 Nature of business  Any arrangements affecting work of D
 Provisions in constitution would be expected to carry out
 Whether Ds or non-exec Ds

6.4.2 Scope of the duty

Historically – Re City Equitable Fire Insurance Co Ltd (1925)


The City Equitable Fire Insurance Company was placed in liquidation after extensive losses through poor management, including the MD
stealing money from the company. The liquidator took action against the company’s directors; however, the company’s constitution
indemnified the directors from liability except that arising from ‘wilful neglect or default.’ [Note: no longer be possible under s199A]
Did the directors act with wilful neglect or default? The directors did not commit wilful neglect or default.

Held that directors had been negligent but that they were exonerated by article relieving them from the consequences of default.

Justice Roma’s comments about duty of care, skill and diligence at that point in time
1. A director need not exhibit in the performance of his duties a greater degree of skill than may
reasonably be expected from a person of his knowledge and experience
2. A director is not bound to give continuous attention to the affairs of the company. His
duties are of an intermittent nature to be performed at periodical board meetings.
a. Evaluated with respect to decisions made at board meetings. Anything outside
doesn’t attract liability.
3. In respect of all duties that...may properly be left to some other official, a director is, in the
absence of grounds for suspicion, justified in trusting that official to perform such duties
honestly.
a. Can delegate as long as no suspicion. This says that if a delegate does something
wrong, delegator is absolved unless there are massive red flags
b. Very low standard
At this point in history: not much required of directors re: duty of care/skill/diligence

 Low, subjective bar: what do you know as a director


 Duties only performed at intermittent board meetings, whenever they are scheduled
 Justified in delegating in the absence of suspicion

80
AWA Ltd v Daniels (1992)

Facts: Koval, e/ee of AWA Ltd, engaged in forex currency trading w/o any effective supervisions by the co. Reported large profits through
speculative trading, not disclosing losses from unprofitable Ks and making unauthorised loans to cover losses. Audit conducted by Daniels
revealed major deficiencies in internal records and control in AWA, particularly over K’s trading activities. Vital info conveyed by auditor
to Hooke (MD and Chairman). Hooke and auditor failed to convey key info to BOD of AWA who were unaware of deficiencies in nernal
control. AWA continues losing money.
Did all Ds (exec and non-exec) of AWA act in breach of duty of care, skill and diligence?

Held: Non-executive directors of a large company, meeting once a month, cannot fairly be executed to contribute anything more than on
questions of policy

Rogers CJ found: auditors and executive directors liable in negligence (day to day); non-executive directors were not liable in negligence.

Rogers CJ: What constitutes the proper performance of the duties of a director of a particular company is considered to be dependent:

(a) upon the actual knowledge and experience of the individual director;
(b) the nature and extent of the corporation's business;
(c) on the distribution of responsibilities in the particular corporation.

Non-executive directors owe a lesser duty than executive directors (consistent with Romer J in Re City Equitable);

D may properly rely upon the advice given by the company’s internal auditors without breaching their duty: delegation and reliance on
people you have trusted is justified in absence of suspicion (consistent with Romer J in Re City Equitable).

AWA Cases – Delegation


Why were non-exec directors found not to be negligent?

 NEDs not bound to give continuous attention to affairs of corp’n


 Duties are intermittent – duties are to be performed at periodic board meetings, and at
meetings of any committees
 No objective standard of the ‘reasonably competent company D’ to which they aspire; can’t
give legal checklist for NEDs, no base standard
 Diversity of co and variety of business endeavours do not allow for uniform standard: not in
our interests to define a legal test for DOC, skill, diligence
BASE POSITION FOR DUTY OF CARE, SKILL AND DILIGENCE AT GENERAL LAW
Clarke and Sheller JJA in Daniels v Anderson (1995): Ds need to place themselves to guide and
monitor the mgmt of the company. That includes:

 A D has a continuing obligation to understand the company’s business and keep informed
about its activities;
o A D need not undertake detailed inspection of day to day activities but must generally
monitor corporate affairs and policies;
 A D should maintain familiarity with the financial status of the company by regularly
reviewing financial statements;
o A D must make inquiries into matters revealed in the financial statements that
call for inquiry (not necessarily breaching when things look dicey, breaching by
failing to make the further enquiry)
Special vigilance above base level for duty of care, skill and diligence (PBS v Wheeler)

 Special skills of Ds (especially MDs) can require them to “scrutinise the proposed transaction
with caution and thoroughness”.
 Combined with the subject matter outside company’s usual operations: doesn’t do, hasn’t
done and no other board members have the skills.
 ALSO: Duty of care is not a fiduciary duty, it is an equitable duty. In addition to breach of
duty of care, the plaintiff must also demonstrate a causal connection between breach and loss.

81
* Daniels v Anderson (1995) [must know case]

Facts: Appeal from AWA Ltd v Daniels [1992].

Held: Clarke and Sheller JJA agreed with Rogers CJ in some respects and disagreed in others. Notably: Level of diligence required of a
particular director and Standard of care that attaches to a an executive and NEDs

“The director's duty of care is not merely subjective, limited by the director's knowledge and experience or ignorance or inaction… (that the
common law duty of a director) will vary according to the size and business of the particular company and to the experience or skills that
the director held himself or herself out to have in support of appointment to the office”.

Adopted American principles: Pollock J Supreme Court of New Jersey in Francis v United Jersey Bank

 A D should acquire at least a rudimentary understanding of the business of the corporation. Accordingly, a D should become
familiar with the fundamentals of the business in which the corporation is engaged;
 Ds are under a continuing obligation to keep informed about the activities of the corporation;
 D management does not require a detailed inspection of day to day activities, but rather a general monitoring of corporate affairs
and policies. Accordingly, a D is well advised to attend board meetings regularly; and
 While Ds are not required to audit corporate books they should maintain familiarity with the financial status of the corporation by a
regular review of financial statements.

* Permanent Building Society v Wheeler (1994)

Facts: PBS Ds entered into K with Tower for parcel of industrial land in 1991, and in so doing, Tower would purchase business from JCLD.
Wheeler and Holding controlling interest in CHL with controlling interest JCLD and PBD. PBS had no expertise in property development.
Hamilton was MD of PBS and JCLD at time of transactions. He attended PBS board meetings at which decision was taken to purchase land.
He declared his interest and took no part in voting.

Held: As CEO and MD of PBS, H had +ve obligation to protect interests of PBS, unaffected by the fact tH believed he had conflict of
interest and accordingly did not vote.

Since PBS had never embarked on development and had no expertise there, heavy duty on Ds in general and MD in particular to ‘scrutinise
proposed transaction with caution and thoroughness’. H was obliged to consider certain aspects of the transaction – foreseeable risk of harm
vs potential benefits – and could no delegate this task.

Hamilton needed to discharge duty of care by attending the meeting. Breach of s 195? Maybe. Could have sought a board resolution.

Statutory equivalent? s 180(1)  above, must know!


Essentially: applying general law duty of care, skill and diligence to an overall, objective,
reasonable D test. What would a reasonable director in a similar company in similar
circumstances (e.g. insolvency?) have done?
s 180 mirrors general law. Proved a breach under general law, you have a statutory breach.
Note:

 Absence of word ‘skill’. Equivalent at general law includes skill.


 However, all modern law decisions determined under s 180(1) have read skill into the
provision, that’s what the cases are about: ASIC v Adler; Daniels v Anderson.
Factors to take into account in determining a breach of s 180(1)
1. 1. Company’s circumstances
a. Size of the company
b. Nature of the business
c. Provisions in constitution
d. Whether directors or non-executive directors
2. Director or officers position and responsibilities within the company
a. The skills and experience of particular director
b. Any arrangements affecting the work the director would be expected to carry out

82
Interpretation of modern duties and subtle exceptions
Austin J ASIC v Vines (2005): An objective standard of care measured by reference to what a
reasonable person of ordinary prudence would do, which could be more demanding in
circumstances where the individual has been appointed by reference to a particular skill possessed by
that individual
Vines v ASIC (2005)

Facts: Vines CFO of the GIO Group when AMP launched a hostile takeover bid in 1998 and GIO needed to prepare target statement to
shareholders. Vines had general responsibility for the company's financial affairs and undertook specific responsibilities in relation to GIO's
response to the takeover in preparing profit forecast. GIO was exposed to substantial losses and Vines had a duty to update the financials in
the target before sending it out.

Held: Failed to take positive steps to advise the Due Diligence Committee of the basis of the assumptions underlying the profit forecast
which he knew or ought to have known was improbable. When Vines supported the integrity of the GIO profit forecast to the Due Diligence
Committee when he knew or ought to have known was improbable. When in the period after the Part B statement was issued by GIO he
failed to give attention to whether the GIO Re profit forecast would be achieved

Santow J dissented on the basis that Vines' role as chief financial officer was supervisory rather than operational and he was entitled to rely
on the other executive's advice unless he had grounds for suspicion

ASIC v Rich (2003)

Facts: ASIC commenced proceedings against several officers of One.Tel Ltd. Greaves, was chairman of the company, chairman of the
finance and audit committee and non-executive director of One.Tel. ASIC alleged that G had special responsibilities beyond those of the
other non-executive directors by reason of the additional positions he held and by reason of his qualifications, experience and expertise.
Qualified charted accountant and experience as financial director and CFO on large listed companies

Did the chairman of the board, in the particular circumstances of this case have special responsibilities above and beyond the other non-
executive directors?

Held: G failed to remain informed about the company’s financial position and rejected the contention that a company chairman had no
greater responsibilities or duties than other directors

Care, Skill and Diligence

 Reasonable Care
o Director owes the company a duty to take reasonable care in the performance of their
office
o Standard of care is of an ordinary prudent person
 Skill
o Objective standard of skill and a director is under a duty to make reasonable efforts to
become familiar with the affairs of the company
o If a director possesses special knowledge or skills a higher standard of skill attaches
 Diligence
o The responsibilities of a directors require that they take reasonable steps to place
themselves in a position to guide and monitor the management of the company

83
6.4.3 Statutory right to delegate and rely on others

Starting point  can delegate to anyone


s 198D(1) unless company’s constitution provides otherwise, the directors of a company may delegate
any of their powers to (a) a committee of directors; or (b) a director; or (c) an employee of the
company; or (d) any other person.
What if delegate is negligent in exercising power delegated to him/her?
Look to: s 190(1) if directors delegate a power under s 198D, a director is responsible for the exercise
of the power by the delegate as if the power ad been exercised by the directors themselves.
BUT: s 190(2) a director is not responsible if (a) director believed on reasonable grounds that
delegate would exercise power in conformity with duties imposed on Ds; and (b) D believed (i) on
reasonable grounds; and (ii) in good faith; and (iii) after making proper inquiry if circumstances
indicated need for inquiry; that delegate was reliable and competent in relation to power delegated.
Some duties so important that they are non-delegable. Some examples:

 PBS v Wheeler: Something so fresh and new, no-one else skilled, had to do himself
 ASIC v Vines: Assumed responsibility on himself to ensure accuracy = non-delegable
When can a director rely on others?
s 189 creates a rebuttal by evidence that reliance on information relied on by Ds provided by others
will be relevant if all those elements are satisfied.
Determining Reasonableness of the Reliance or Delegation

 function that has been delegated is such that it may properly be left to such officers
 extent to which D is put on inquiry, or given facts of a case, should have been put on inquiry;
 relationship btw D and delegate, must be such that the D honestly holds the belief that the
delegate is trustworthy, competent and someone on who reliance can be placed;
 the risk involved in the transaction and the nature of the transaction;
 extent of steps taken by D, eg inquiries made or other circumstances engendering “trust”; and
 whether the position of the D is executive or non-executive
ASIC v Healey (2011) (“Centro liability decision”) [spent a while on this – ST vs LT liability]

Facts: BOD of Centro companies approved the consolidated financial statement and declared under s 294(2) that the financial statements
complied with the Act and the Australian accounting standards. Financial statements misclassified $2.6 billion of borrowing as non-current
when it should have been classified as current liabilities (due within 12 months) under the accounting standards. Failed to disclose
guarantees of short term liabilities of $1.75 billion. Ds received advice from management and external auditors regarding the financial
statements. The defendants argued that they were entitled to rely upon the external auditors and the knowledge and expertise of Centro’s
audit committee.

Did the board, CFO and MD breach their duty of care and diligence, and if so, were they entitled to rely on others? Are the Ds required to
apply their own minds to and carry out a careful review of fin statements?

Held: Although the Ds (including non-executive) were entitled to rely on management and auditors, they had a non-delegable primary
responsibility for the company’s financial report and D’s report.

The importance of the financial statements is one of the fundamental reasons why the Ds are required to approve them and resolve that they
give a true and fair view.

‘In the light of the significance of the matters that they knew, they could not have, nor should they have, certified the truth and fairness of the
financial statements, and published the annual reports in the absence of the disclosure of those significant matters. If they had understood
and applied their minds to the financial statements and recognised the importance of their task, each D would have questioned each of the
matters not disclosed. Each D, in reviewing financial statements, needed to enquire further into the matters revealed by those statements’.

Ds have responsibility to ‘read, understand and focus’ and ‘make further enquiries if matters revealed in those fin statements call for it’

84
ASIC v Cassimatis (No 8) [2016] FCA 1023

‘A reasonable director with the responsibilities of Mr and Mrs Cassimatis would have known that the Storm model was being applied to
clients such as those who fell within this class and that its application was likely to lead to inappropriate advice. The consequences of that
inappropriate advice would be catastrophic for Storm (the entity to whom the directors owed their duties). It would have been simple to
take precautionary measures to attempt to avoid the application of the Storm model to this class of persons’.

Current summary of content of duty of care, skill and diligence


Consistent since Daniels v Anderson
Two new points from ASIC v Healey: understanding of financial statement; D questioning mind.
1. Should acquire at least a rudimentary understanding of the business of the corporation and
become familiar with the fundamentals of the business in which the corporation is engaged;
2. Should keep informed about the activities of the corporation;
3. Whilst not required to have a detailed awareness of day-to-day activities, should monitor the
corporate affairs and policies;
4. Should maintain familiarity with the financial status of the corporation by a regular review
and understanding of financial statements;
5. A director, whilst not an auditor, should still have a questioning mind.

6.4.4 Statutory Business Judgment Rule

s 180: Business judgment rule [safe harbour – but if you’re doing your job shouldn’t need it]
(2) A D or other officer of a corp’n who makes a business judgment is taken to meet the
requirements of s 180(1), and their equivalent duties at common law CL and E, in respect of
the judgment if they:
(a) make the judgment in good faith for a proper purpose; and
(b) do not have a material personal interest in the subject matter of the judgment; and
(c) inform themselves about the subject matter of the judgment to the extent they reasonably
believe to be appropriate; and
(d) rationally believe that the judgment is in the best interests of the corp’n.
The D's or officer's belief that the judgment is in the best interests of the corp’n is a rational
one unless the belief is one that no reasonable person in their position would hold.
Note: This subsection only operates in relation to duties under this section and their equivalent duties at common law or in
equity (including the duty of care that arises under the common law principles governing liability for negligence)—it does not
operate in relation to duties under any other provision of this Act or under any other laws.

(3) “business judgment” means any decision to take or not take action in respect of a matter
relevant to the business operations of the corp’n.
Basically: Constant exercise of Judgment + Informed decision + rational belief judgment is ‘in the
best interests’ of the corporation
* ASIC v Adler (2002)

Facts: ASIC instituted proceedings against:Adler (non-executive director of HIH and officer of subsidiary HIHC); Williams (director of
HIHG and HIHC and CEO of HIH); and Fodera (Executive Director and Finance Director of HIH and HIHC

Did Adler, Williams and Fodera engage in multiple breaches? If so, did the BJR s 180(2) apply to protect the directors from breach of duty
of care and diligence?

Held: there was no “business judgment” (as either a defence or as an element in the contravention) shown by Mr Adler to have been made
“in good faith for a proper purpose” (s180(2)(b)). Clearly Mr Adler did have “a material personal interest in the subject matter of the
judgment” so also precluding application of that rule. No equivalent defence at general law is made out either.”

85
Application of business judgment rule in ASIC v Mariner
1. ‘business judgment’: considering the nature of Mariner’s business and the potential benefits
to Mariner of attaining control of Austock, the decision to commence the takeover and make
the announcement was a business judgement.
2. ‘good faith for proper purpose’: believed that the decision to make the announcement and
pursue a takeover bid was in the best interests of Mariner due to the potential for Mariner to
make a significant profit.
3. ‘no material personal interest’: satisfied.
4. ‘inform themselves of the subject matter of the judgment to the extent they reasonably
believe to be appropriate’: provided with information; various meetings and discussions;
and personal knowledge as to level of interest in the target.
5. ‘rationally believed that the judgment was in the best interests of company’: It was not a
belief that no reasonable person in [directors] position would have held - the belief was a
rational one. Alternatively, as a matter of substance and in any event, [directors] process of
reasoning was rational. Considerations:
o the importance of the business judgment to be made;
o the time available for obtaining information;
o the costs related to obtaining information;
o the director or officer’s confidence in those exploring the matter;
o the state of the company’s business at that time and the nature of
o competing demands on the board’s attention; and
o whether or not material information is reasonably available to the director
*ASIC v Mariner Corporation Limited [2015]

Dismissal of an application by ASIC for declarations that each of the directors of Mariner breached s180(1) by making a reckless decision
that Mariner would announce a takeover bid for Austock without securing funding.

Directors were very confident they would secure the funding – very tight industry, no competitors, etc Beach J. rejected the proposition that
a breach by a company automatically meant directors had breached their individual duties.

Business judgment rule: The director’s or officer’s belief that the judgment is in the best interests of the corporation is a rational one unless
the belief is one that no reasonable person in their position would hold.

Limitations of the business judgment rule

 The rule does not protect all actions and decisions made by directors, only operational
decisions to take or refrain from certain action concerning the company’s business
o e.g. it would protect decisions to undertake new business, but would not protect in
areas such as insolvent trading, misstatements in prospectus or takeover documents:
see ASIC v Fortescue Metals
 No protection is given where the “process” leading up to the decision is flawed e.g. due to
manifestly inadequate information.
o Protection is limited to the duty of care in s 180(1) and its CL equivalents.
o The onus of proof rests on defendant directors: ASIC v Rich
 Reliance failed in ASIC v MacDonald as Mr MacDonald did not testify – i.e. there was no
evidence as to ‘rational belief’.
 “Business judgment” was interpreted in ASIC v Rich to include operational activities, such as
planning, budgeting and forecasting.

86
6.5 Insolvent Trading

6.5.1 Liability under Corporations Act s 588G

Circumstance of statutory veil piercing.


Harsh test: motivate directors to stop trading as soon as they suspect insolvency. Have to go from
being entrepreneurs to trustees. Need to preserve assets for creditors. Threat or motivation for them to
do this is personal liability.
s 588G(1) & (2) apply to impose liability upon a person if:

 the person is a director of the company when the company incurs a debt; and
 the company is insolvent when it incurs the debt or becomes insolvent because it incurs the
debt; and
 at the time the debt is incurred, there are reasonable grounds for suspecting that the
company is insolvent or would become insolvent because it incurs a debt; and
 the director is aware at the time the debt is incurred that there are reasonable grounds for
suspecting the company is insolvent or a reasonable person in a similar position in a
company in the company’s circumstances would be so aware
 the debt was incurred after commencement of this Act [23 June 1993]
Where these elements exist, s 588G(2) establishes a contravention only where:
SUBJECTIVE: The person is aware at that time that there are such grounds for so
suspecting; OR
OBJECTIVE: A reasonable person in a like position in a company in the company’s
circumstances would be so aware.
Contravention of s 588G(2) attracts civil liability by default
If you can prove they actually knew: Criminal offence if the person actually suspected at the time that
the company incurred the debt that it was, or might become, insolvent: s 588G(3)

Director + Debt + Insolvency + Reasonable Grounds for Suspicion

ELEMENT 1: A person is a director of a company


Includes shadow directors and de facto directors. Directors only (NOT OFFICERS)
ELEMENT 2: At the time the company incurs a ‘debt’. Mainly contractual debts for this course: look
at date debt is due, entered into contract, guarantee, loan, etc
s 588G(1A) sets out table certain company transactions will be deemed to constitute “a debt” for the
purposes of the section and the time at which the debt is actually “incurred”.
ELEMENT 3: Company is insolvent at that time, or becomes insolvent by incurring that debt
Test of insolvency: s 95A(1): A person is solvent if, and only if, the person is able to pay all the
person’s debts, as and when they become due and payable
95A(2): A person who is not solvent is insolvent
Presumption of insolvency: s 588E(3)

87
ELEMENT 4: At that time, there are reasonable grounds for suspecting that the company is
insolvent, or would so become insolvent.
Objective test: reasonable grounds (doesn’t depend on what the director actually thought)
Suspicion is a much lower test than expectation or actual knowledge.

 What is suspicious? Example in the context of bankruptcy act:


o The word "suspect" has been taken to mean:
 more than a mere idle wondering whether something exists or not.
 It is a positive feeling of actual apprehension or mistrust; of having an
apprehension or mistrust amounting to a slight opinion but without sufficient
evidence.
 Thus, a reason to suspect that a fact exists is more than a reason to consider
or look into the possibility of its existence
 Almost short of having just enough evidence to prove it – all you need is the smoking gun.
Very high bar. Just short of actual proof.
Consider entire circumstances when determining whether reasonable grounds to suspect (more than
mere idol wondering) (Metropolitan Fire Systems)
Metropolitan Fire Systems Pty Ltd v Miller (1997)

Facts: the Millers were directors of a company called Raydar. Raydar accepted a contract from Reed to install electrical systems. When
Raydar accepted this contract, it had a number of unpaid creditors. Raydar was unable to complete the entire job and contracted with MFS to
perform some of the work. MFS completed the work but Raydar was unable to pay for the work that MFS had done as Raydar had not yet
been paid by Reed for the work. Payment from Reed failed to arrive before one of Raydar’s creditors issued a statutory demand (usually first
red flag of insolvency). MFS heard about this then sought a court declaration that Raydar was insolvent and that the Millers breached
s588G by allowing Raydar to incur debts to MFS.

Held: Directors breached duty under s 588G

There were reasonable grounds for suspecting Raydar was insolvent. Raydar was insolvent as it had large amounts of unpaid debts when it
incurred liability to MFS. It had $200,000 in assets and $400,000 in liabilities.

Even though it’s a cash flow test, the balance was outweighed enough to justify reasonable grounds to suspect (more than mere idle
wondering, just short of proof)

Director’s state of mind

s 588GA Safe Harbour – Taking course of action reasonably likely to lead to a better outcome for the
company.

 s 588GA(7) – ‘better outcome’


 s 588GB Information or books not admissible to support the safe harbour if failure to permit
inspection etc
 s 588HA Review relating to safe harbour

6.5.2 Defences

s 588H Long list of defences to breach of duty to avoid insolvency: most are quite a high bar

 s 588H(2): Reasonable grounds to expect (and did expect) that the company was solvent
o Objective AND subjective test
o Test to be exculpated from liability for insolvent trading is very high (expect)
o Test to establish liability in the first place is much lower threshold (suspect)
 s 588H(3): Reasonable reliance on the information from a reliable person who was
responsible for providing information to the director

88
o Appears to be a massive exception, but see McLellan  high threshold
 s 588H(4): Justifiable non-participation: illness or some other good reason.
o Intended to be used in a wide variety of circumstances, but see: DCT v Clark
 s 588H (5) and (6): Reasonable steps taken to prevent incurring of a debt
McLellan, in the matter of the Stake Man Pty Ltd v Carroll [2009]

Facts: To expand production capacity company purchased plant and equipment. Huge problems with delay in delivery and impacted
business. The sole director, Mr Carroll, engaged an accountant, Mr Bright to provide business advice. Mr Bright advised that the company
was close to insolvency and that a capital injection was required. Funds were advanced in June 2005. All funds exhausted by February 2006
and Mr Carroll engaged an insolvency specialist. The company placed into voluntary administration in May 2006.

Held: Company was insolvent and each element of s 588G was satisfied.

DCT v Clark [2003]

Facts: Family co with 2 Ds who were hubby and wife (Clarks). Mrs C did not take part in mgmt. of co and relied on hubby to manage
business. When co got into fin difficulties, payment was made to Commissioner of Taxation. This payment was then reclaimed by co’s
liquidator as an unfair preference. Commissioner of taxation may seek an indemnification from company Ds when their co’s taxation
payments are reclaimed as unfair preferences. The defence to this indemnification provision is s 588FGB which is same as s 588H.
Trial court held Mrs C had defence by relying on her hubby and not taking part in mgmt. of business for ‘good reason’. Commissioner
appealed.

Held: Mrs C’s reliance on her hubby did not justify failure to take part in mgmt. of co for ‘some other good reason’ as per s 588H(4)
equivalent.

Mrs C could not avoid liability on basis that leaving the business to her hubby was ‘some other good reason’. Court held provisions must be
read w/in broader context of legal expectations that Ds will take an active part in managing / monitoring the mgmt. Blindly delegating
power and responsibility to another person would not satisfy this duty.

6.5.3 Compensation remedies with respect to insolvent trading

Consequences of breach of duty to avoid trading while insolvent

 s 588G(2) is a civil penalty provision: Pt 9.4B


 Specific remedies in insolvent trading regime: Pt 5.7B, Division 4.
 Court may also order a director to pay compensation to the company for loss or damage as
part of criminal proceedings against the director for knowingly (or suspectingly) engaging in
insolvent trading: s 588K
 Director disqualification by the court under s 206C.

6.5.4 Liability of a holding company for insolvent trading by a subsidiary company under s
588V

s 588V: When holding company liable


Imposes a parallel insolvent trading regime for holding companies:

 If their subsidiary incurs a debt at a time of insolvency or near insolvency; and


 The holding company or one or more of its directors was aware or ought to have been aware
of that fact.
s 588W Liquidator of the subsidiary can recover from the parent company the amount of loss or
damage suffered by any creditor as a debt due to the company:

89
6.6 Remedies: All roads lead to the Civil Penalty Regime

Any defence to Consequences of Other excape routes


Prove the breach
breach? breach (6.6) for Ds

6.6.1 Equitable remedies: injunction, compensation, avoidance

Remedy depends on what has happened.

 Profits issue: account of profits = monetary damages


 When director doesn’t make any profits = equitable compensation
 Want to stop certain actions = injunction
ASIC v Adler (2002)

1. Duty of Care, Skill and Diligence:


a. An equitable duty, not fiduciary. Therefore, primary remedy is equitable
compensation (damages): PBS v Wheeler.
b. Directors also owe a common law duty of care, the remedy for which is damages:
Daniels v Anderson
c. Remedy is designed to compensate the company for the loss caused by the breach of
duty.
2. Duty of Loyalty:
a. Proper purposes / conflict of interest (no profit):
i. injunctive relief;
ii. declaratory relief;
iii. rescission;
iv. resist specific performance.
b. Proper purposes / conflict of interest (profit):
i. account of profits;
ii. constructive trust
iii. equitable compensation

6.6.2 Corporations Act

Basically an issue of standing: who can prosecute for statutory equivalent and what can they get?
Remedies for breach of civil penalty provisions (basically every breach we have covered)

 s 180(1): Duty of care, skill & diligence


 s 181: Duty to act in good faith
 s 182: Prohibition on improper use of position
 s 183: Prohibition on improper use of inside information
 s 588G: Director’s duty to prevent insolvent trading
 s 209(2): Contravention of related party prohibition
 s 344(1): Directors’ failure to comply with provisions dealing with accounts.
Who has standing to bring civil penalty proceedings?

 ASIC can apply for a range of orders: s1317J(1)


 The company can apply for a compensation order: s1317J(2)
 The company can intervene in an ASIC application for a declaration of contravention or
pecuniary penalty order: s1317J(3)

90
Consider who you are advising

 ASIC: interested in statutory breaches and breaches of Corporations Act


 Board of Directors: interested in breaches of general law also
Declaration of contravention: s 1317E

 If ASIC suspects director has breached a duty, can apply under s1317J.
 Court must make a declaration of contravention if satisfied of contravention of a civil penalty
provision (CPP) s1317E.
 › NB: a declaration of contravention is a prerequisite to granting of a Disqualification Order
or Pecuniary Penalty Order or both.
Pecuniary penalty order: s 1317G
If a declaration is made that a person has contravened a CPP under s1317E, then

 ASIC may apply to the court under s1317G for an order that a person who has contravened a
CPP pay to ASIC a pecuniary penalty of up to $200,000.
 But Court must be satisfied that the contravention:
o materially prejudices the interests of the company or its members; or
o materially prejudices the corporation’s ability to pay its creditors; or
o is serious.
Compensation to the company: s 1317H

 A Compensation Order can be made whether or not a Declaration of Contravention has been
made under s1317E.
 An order that a person who has contravened a CPP compensate the company for damage
resulting from the contravention: ss1317H and 588J.
 Damage to the company is also deemed to include any profit made by any person resulting
from the contravention or offence: s1317H(2).
 Court must still be satisfied that a contravention of a CPP has occurred and damage resulted.
Disqualification: s 206C

 If a declaration is made that a person has contravened a CPP under s1317E, then
 ASIC can apply to the court under s206C for an order disqualifying the person from
managing corporations for a period that the court deems appropriate
 Court must be satisfied that the disqualification is justified (depends on remorse, track record,
people been affected by your conduct, etc)
Application by the corporation: s 1317J(2)

 Only a Compensation Order may be made: s1317J(2)


 BUT company may intervene (get standing) in an application for a Declaration or
Compensation Order by ASIC: s1317J(3).

91
6.7 Some Liability Escape Hatches for Directors

Be aware that a number of escape hatches may exist, whereby Ds may ultimately be saved from the
consequences of breach of fiduciary duty. These escape hatches include:

6.7.1 Ratification by the shareholders in general meeting

Woolworths v Kelly

 Requires full disclosure of nature and extent of interest


 But it won’t be effective if it amounts to fraud on the minority and probably can’t waive
breach of statutory duty.
For ratification to be effective, it must not be brought about by unfair or improper means (Cook v
Deeks)
The GM must be fully informed about the relevant conduct before the resolution is passed. Failure to
do so will render the ratifying resolution ineffective: Winthrop Investments v Winns
Authority that GM can’t cure a statutory equivalents (can only waive common law breaches): Forge

6.7.2 Ratification by the Board?

Usually ineffective unless authorised by the constitution or replaceable rules – but query Qld Mines

6.7.3 Relief granted by the court

ss1317S (civil penalty provisions), 1318 (civil proceedings) – where D acted honestly and, with
regard to all the circumstances ought to be excused McLellan v Caroll

6.7.4 Constitution Modification

Exemption (s199A(1)) – it’s prohibited, but attenuation of duty is ok: Whitehouse v Carlton;

6.7.5 Indemnification

Policy argument: balancing exercise between interests of shareholders and interests of Ds.

 Personal liability is necessary to motivate Ds to make decisions in the best interests of


shareholders. But to a certain extent, their job is to be entrepreneurial. They’re not going to
take business risk unless they can be indemnified or exonerated in certain circumstances.
 Can’t, for example, completely exonerate Ds for reckless dishonest behaviour. Can’t promise
to pay Ds penalty debts if ASIC pursues them. But can indemnify in advance. Company can
pay indemnity insurance premiums. D can then use insurance policy to defend themselves.
 Indemnification for some breaches is prohibited, others ok; (s199A(2))
 Indemnification for legal costs (s199A(3)) – only denied if the D found liable;
 Payment of insurance premiums (s199B) – generally OK, unless wilful breach or breach of ss
182-183.

92
7. Members’ Rights and Remedies
Members rights and remedies separates good exam responses: target remedies, don’t scattergun.
Who is my client? What do they want (damages; injunction; release?)
When do members have standing to bring a derivative action against the company?

7.1 Equitable limitations on the voting power of majorities


Constitution is freely alterable, which changes rights and obligations. Some members will experience
discrimination. When is the prejudice sufficient to bring a derivative action against the company?
The Rule in Foss v Harbottle (proper plaintiff rule): to the extent that the company is a separate
legal entity and where a wrong is done to the company, the company is the plaintiff who has standing
to bring an action to seek remedy of that wrong (i.e. breach of directors duties);

 Generally stops individual shareholders bringing actions.


 Problems arise when BOD don’t want to bring an action (self-interest, lazy)
Exceptions: standing for individuals is available:
1. Ultra vires or illegal acts
2. Absence of authorisation by a special majority of the company in GM
3. Fraud in the minority
4. Breaches of member’s personal rights**  due to statute, this is really the only relevant one
a. Usually because this deals with share allotments, changes to holdings and value,
which are personal rights
b. Right to bring derivative action at common law has been abolished by statutory right
5. Where justice so requires  unique to Australia, flexible application, means whatever court
wants it to mean

7.1.1 Types of resolutions

75% majority to change constitution. How has general meeting exercised power? Is it bona fide for
the benefit of the company? Limitations on how they can exercise power or discretion.
However, minority bears onus of proof for remedies (except Gambatto), because want to limit
individual standing  want to insulate board.
Fraud on the Minority: “The powers of the majority must be exercised bona fide for the benefit of the
company”: Allen v Gold Reefs of West Africa Ltd
Main Type of Resolutions Where Fraud on the Minority has been bar to ratification:

 Misappropriation of corporate property and rights (corporate opportunity, secret profits)


 Release of directors’ duties (ratification of general meeting – Woolworths v Kelly, but
caveats)
 Alteration of articles (constitution freely alterable, but how
far until a remedy is available?)
 Expropriation of a member’s shares (Gambotto’s case: High
court split in two, changes that involve expropriation of shares
and then other changes that don’t)

93
A. Misappropriation of corporate property and rights

 Misappropriation of an opportunity that the company had been actively pursuing = actual
misappropriation amounting to fraud on the minority: Cook v Deeks
 75% shareholders drafted resolution saying company was not interested in opportunity so
they could pursue in a side company
 At all material times, the original company was still interested in the opportunity.
 Would need to be a disinterested vote
o Cf: When directors receive an incidental profit, rather than stealing per se: Regal
Hastings
 Parent company didn’t have enough capital to pay landlord, who was not
comfortable dealing with shell company. Asked to pay capital or personal
guarantee from director.
 Was this an opportunity directors acquired through exercise of fiduciary
office?
 Only because of fiduciary office, special opportunities and knowledge?
 Did they make a profit? court focused on no profit limb
 Did they get consent of general meeting?
 This is where it failed. But court noted they could have.
o Regal Hastings was classified as incidental profits. Company in Cook v Deeks –
company was materially interested in the opportunity.
o In Cook v Deeks, there was breach of no conflict and breach of proper purposes rule
by directors - impossible for directors to argue they were acting for the benefit of the
company according to the formulation of fraud on the minority in Allen v Gold Reefs
(exercise of power was bona fide and for the benefit of the company as a whole).
Look at surrounding circumstances = just because a GM resolution says it’s passed, doesn’t mean it
can’t be challenged. Is it capable of being cured by shareholder ratification? E.g won’t be effective:

 Fraud on minority
 Insolvency
 Any lack of information, concealment: Winthrop v Winns
o Including circulars, notice of general meeting

B. Release of directors’ duties

 Traditionally, depended on the quality of the directors’ acts complained of.


 Courts more lenient when there is an absence of self-interest.
 Breaches where general meeting ratification or waiver would seemingly be permitted
included:
 Incidental Profit : Regal Hastings v Gulliver
 Failure to disclose an interest in a contract with the company: North-West. Transportation Co
Ltd v. Beatty
 Duty to act in good faith and for proper purposes (transactional package that had the effect of
blocking a purported takeover – no self-interest): Hogg v Cramphorne.
Full and complete knowledge (purpose and nature of breach) is required for effective shareholder
ratification. (Winthrop)
Broad ability for GM to waive breaches: prospectively, retrospectively. (Winthrop)
Can waive bona fide and mala fide breaches. (Winthrop)

94
* Winthrop Investments Ltd v Winns Ltd [1975]

Winns board engaged in defensive conduct in response to an unwelcome takeover bid, by making a diluting share issue to Burns Philp.
Winns shareholders approved the directors’ conduct. Agreed assumption was that directors didn’t enter into the transaction bona fide in the
best interests of the company, but rather to frustrate the takeover. Assumed that this was a mala fide (i.e. bad faith) breach i.e. intentionally
not for proper purposes.

Held: Exercise of power by the directors is voidable, but not void - possible for GM to relieve the breach of duty, by waiving the breach.
Purpose of the resolution must be clearly stated and the nature of the directors’ breach clearly disclosed by the directors seeking absolution
(i.e. similar to stringent standards in relation to notices and circulars).

What happens where the directors whose actions are challenged themselves control the General
Meeting?
Silent on this issue in Winthrop v Winns
If majority shareholders had the same improper purpose as the directors that would amount to fraud
on the minority such that the breach could not be cured: Residues Treatment and Trading Co v
Southern Resources (also applicable in Cook v Deeks – present to themselves)

C. Alterations of the company’s articles that prejudice shareholder rights

A classic area for challenges under the doctrine of fraud on the minority, since it was an obvious area
of shareholder power to pass resolutions.
Traditionally, the test for fraud on the minority as set out in Allen v Gold Reefs of West Africa: “The
powers of the majority must be exercised bona fide for the benefit of the company” was read broadly
and greatly favoured the majority: See Peters’ American Delicacy v Heath (1939)..
Company cannot deprive itself of the power to alter its articles. (Peters American Delicacy)
Shareholders are able to vote for their own personal advantage (Peters American Delicacy)
If actions prejudice rights - not automatically mean fraud on the minority. (Peters American
Delicacy)
If resolution was passed fraudulently or oppressively or was so extravagant that no reasonable person
could believe that it was for the benefit of the company, it should be held to be invalid. (Peters
American Delicacy)
* Peters American Delicacy Co Ltd v Heath (1939)

Constitution contained 2 inconsistent modes of issuing of bonus shares computed on amount of capital paid up on the shares (Art 108); and
computed on the number of shares held (Art 120). It was basically a stuff up by the legal drafters. Constitution was altered to delete article
120, so that thereafter the shares could only be distributed on the basis of the amount paid-up on the shares. The partly-paid shareholders
challenged the resolution.

Held: The fact that an alteration prejudices or diminishes some of the rights of the shareholders is not in itself a ground for attacking the
validity of the alteration.

A resolution to alter the articles that is made bona fide for the benefit of the company as a whole and, not exercised fraudulently and for the
purpose of oppressing the minority, will be valid. Shareholders need not always have only the benefit of the company in view, particularly
when the question which arises relates to the relative rights of different classes of shareholders.

Test for fraud on the minority is whether, in the honest opinion of the shareholders, the alteration to
the article was passed “bona fide for the benefit of the company as a whole”. (Greenhalgh)
The “company as a whole” doesn’t mean the company as a commercial entity, but the shareholders as
a general body. (Greenhalgh)

Look at the resolution and ask whether it is liable to discriminate between majority and minority  or
does it apply equally? (Greenhalgh)

95
Greenhalgh v Arderne Cinemas Ltd [1951]

Mr Greenhalgh was a minority shareholder in Arderne Cinemas and wished to prevent majority shareholder, Mr Mallard, selling control.
Constitution contained a pre-emption clause, which prohibited the sale of any member’s shares to an outsider, so long as another member
was prepared to purchase them. Company changed its articles, allowing existing shareholders to offer any shares to person/members outside
the company, provided it was approved by ordinary resolution. Mr Mallard, the majority shareholder, wished to transfer his shares for 6
shillings each to Mr Sol Sheckman in return for £5000 and his resignation from the board.

Held: Lord Evershed MR (which whom Asquith and Jenkins LLJ concurred) held that the £5000 payment was not a fraud on the minority.
None of the majority voters were voting for a private gain. The alteration of the articles was perfectly legitimate, because it was done
properly.

Discrimination on the surface will be presumed to not be for a proper purpose unless proved
otherwise. (Clyde)
AFT v Clyde Industries Ltd (1959)

The case involved a resolution directed at members who held shares as a trustee under unit trusts.

The resolution stated that henceforth, such shareholders could not vote unless they had received the direction of the majority of unit holders.
If discrimination exists, without anything to explain it, the inference can be drawn that the resolution has not been passed for a valid
company purpose.

D. Acquisition of shares – “Gambotto’s Case”

The power to amend the constitution to expropriate the shares of the minority can be taken only if:
i. It is exercisable for a proper purpose; and
ii. Its exercise will not operate oppressively in relation to minority shareholders (fair price, fair
dealing)
* Gambotto v WCP Ltd (1995)

Majority shareholders in WCP were wholly-owned subsidiaries of Industrial Equity Limited and held 99.7% of the issued capital
(compulsorily acquisition was not possible under the Corporations Law). A notice of general meeting included a proposed amendment to the
articles, the effect of which was to enable any member who was “entitled for the purposes of the Corporations Law to 90% or more of the
issued shares” to compulsorily acquire the remaining issued shares. In May 1992, the meeting took place and the resolution was approved.
The appellants did not attend.

The appellants contended that the purported amendment was invalid as: The amendment was oppressive and thus beyond the scope and
purpose of the power of alteration of the articles conferred by s176 of the Corporations Law (now s136(2) of the Corporations Act); and The
amendment imposed restrictions on the right to transfer shares within the meaning of s180(3) of the Corporations Law (now s140(2) of the
Corporations Act).

Held: Amendment was fraud on the minority. Was not for proper purpose.

“an expropriation may be justified where it is reasonably apprehended that the continued shareholding of the minority is detrimental to the
company, its undertaking or the conduct of its affairs – resulting in detriment to the interests of the existing shareholders generally – and
expropriation is a reasonable means of eliminating or mitigating that detriment”

Limb 1: “proper purpose”:

 It is only right that exceptional circumstances should be required to justify an amendment to


the articles authorising the compulsory expropriation by the majority of the minority’s
interests in a company
 Majority held $4 million tax saving was not a proper purpose. Rather would need to avoid a
significant detriment to the company. But see McHugh J below who thought it was proper
purpose.

96
Limb 2: “will not operate oppressively”: substantial and procedural fairness
“Procedural fairness” said to reflect 2 matters :
i. full disclosure of all material information leading to the alteration [mandatory]; and
ii. an independent expert’s valuation of the shares to be expropriated [optional or presumptive].
a. centres upon quality and extent of information provided to shareholders
“Substantive fairness” is largely concerned with the price at which the shares are expropriated (i.e. an
economic view of fairness).

 Market price is relevant insofar as expropriation below market price is prima facie unfair.
However, market price is not determinative.
 Surrounding facts and circumstances: A variety of other factors must be considered when
assessing fairness including assets of the company; market value; dividends; the nature of the
company; and its likely future
McHugh’s judgment in Gambotto
Proper purpose: tax benefit was fine
Oppressive:

 “fair price”: questionable as to whether market price represented a fair price. He stated
“judges cannot delegate to the market the duties of courts to fix a fair price for shares.”
 “fair dealing” involves how the expropriation was structured, initiated, negotiated, and
disclosed. It was oppressive not to give them full information.
o While the price stipulated "may well have been a fair price for the shares... almost no
attempt was made to make the full disclosure that is required in this class of case".
Majority: “proper purpose” McHugh: “proper purpose”

 To avoid detriment  Did not restrict proper purpose to


 Goal of majority shareholders – to avoidance of detriment.
reduce the company’s taxation liability  It was also a proper purpose to pursue
- did not satisfy this test of validity. significant goal.
 Expropriation for a legitimate business
object was OK provided fair to
expropriated party.
 This test would permit a company to
pursue a tax advantage.
Tests for amendment of constitution at general law (Gambotto)
Constitutional Amendment NOT Constitutional Amendment INVOLVING
INVOLVING EXPROPRIATION EXPROPRIATION

 Prima facie valid unless ultra vires,  Prima facie invalid unless:
beyond any purpose contemplated by o Proper purpose (not to
the company’s articles or oppressive. detriment, but c.f. McHugh)
 Onus: on minority to establish fraud o Not operate oppressively (fair
on the minority price, fair dealing)
 Onus: on majority

97
7.2 Shareholder suits

Need to prove underlying breach first. These are then very broad remedies. Two options, but onerous
procedure to get them. Oppression is an easier route with broader remedies.

7.2.1. Statutory injunctive relief under s 1324

Option 1
s 1324: Injunction: ‘any person who interests have been or are affected by conduct contravening the
act’  interpreted narrowly to be true to Foss v Harbottle, not broad as intended.

7.2.2. The statutory derivative suit (ss236-42)

Option 2
Action on behalf of all members of company

 Need to prove breach underneath first


 To extent party is successful, derivative suit  proceeds go back to co
 Then prove everything to get standing and leave  oppression easier route
s239: ratification by GM not an absolute bar

 However the court may take the ratification into account if it was
o 1. Well-informed
o 2. Acting for proper purposes when they voted to ratify the conduct
 Based on doctrine of waiver: can’t waive without full and complete knowledge, if you’re
bullied and consideration of purposes.
s236(3): abolishes common law remedy
s236: Standing

 A member of the company (or a related body corp);


 A former member of the company (or rbc); (e.g. if you were kicked out)
 A person entitled to be registered as a member of the company (or rbc); (BOD refuses to
register you)
 An officer or former officer of the company (broad definition of officer: anyone who has
ability to affect financial standing of the company)
s237: Elements

 Court must permit a derivative action to proceed when:


o It is probable that the company will not commence the action
o The applicant is acting in good faith
o The action is in the best interests of the company
o There is a serious question to be tried
o The applicant gave the company 14 days’ notice of the intention to apply for leave to
bring the action OR it is appropriate for the court to grant leave anyway
 Can presume in absence of communication that company will not commence action
s 242 Costs orders: A lot at stake: any orders it considers appropriate” about the costs any of the
parties to a statutory derivative suit

98
7.3 Statutory protection for minorities

7.3.1 Compulsory liquidation remedies

A. The just and equitable ground

s 461(1)(k)
Failure of substratum
Re Tivoli Freeholds: “general intention and common understanding of members”

 paramount objects impossible to achieve; or


 embarked on a course completely outside those objects
o NOT to do with corporate capacity. But relates to objects clause.
o Look at surrounding facts and circumstances, incl objects clause.
* Re Tivoli Freeholds Ltd [1972]

A public, unlisted company, had been incorporated with the main object listed in its memorandum as public entertainment. New controllers
involved it in lending surplus funds and profitable ‘corporate raiding’  aggressive finance company. This activity as not contemplated in
the company’s main objects. There was a large minority (42% of issued capital) and 93% of all minority shareholders were opposed to this
policy and challenged the company’s actions on a number of bases including: (i) ultra vires; (ii) winding up on the just and equitable ground;
and (iii) oppression.

Held: company wound up on ground of failure of substratum: complete and far-reaching departure from what was originally commonly
intended and understood by members of the company.’

“It has been recognised that it may be just and equitable to wind a company up if the company engages in acts which are entirely outside
what can fairly be regarded as having been within the general intention and common understanding of the members when they became
members.” per Menhennit J

Quasi-partnership companies
Ebrahimi: non dividend shares; removed as director (only income)

 Small, closely, held company  while it was incorporated, for all intents and purposes it
operated like a partnership
o Personal relationship (long-standing relationships)
o Participate in running the business
o Transfer restrictions  can’t sell shares (here no dividends attached to the shares).
 Ebrahimi was locked into company with no financial rewards and no participation in
management.
* Ebrahimi v Westbourne Galleries Ltd [1973]

Facts: Nazar and Ebrahimi formed private co to sell carpets (previously done as equal partners). They each subscribed 500 shares and made
directors. Later, N’s son was made director and N and E each gave him 100 shares. Profits were wholly distributed to D’s as remuneration.
No dividends were ever paid. 11 years after incorporation, ordinary resolution was passed in HM by the Nazars to remove E as D. E
petitioned for winding of co on ‘just and equitable’ ground. Order was made, set aside by Court of Appeal. E appealed.

Held: The right course was to dissolve the association by winding up. Equity provides breach – just and equitable ground is supplement to
law of corporations.

Equitable considerations typically requires one (and probably more) of the following elements:

- Association formed on a personal basis with mutual confidence (converted partnership satisfies this completely).

- Agreement or understanding that all or some of the shareholders participate in the running of business.

- Transfer restrictions – so if confidence is lost, or one member is removed from management, he cannot take out his stake and go elsewhere.

Deadlock

99
B. Directors acting in their own interests

s461(e) To the extent directors are acting in affairs of company in self-interest and preferring their
interests over one or more sections of members, it may be sufficient for a winding up order.

 Re Cumberland Holdings: Broadly interpreted


o Affairs of company broad definition
o No need for unanimous agreement of all directors
o Don’t need to show preference of own interests over interests of all shareholder –
sections or a significant body is sufficient
 Re Weedmans
o Misleading and deceptive statements are objectively unfair and evidence of directors
preferring their own interests.
* Re Cumberland Holdings Ltd (1976) 1 ACLR 361 [Redmond [8.170]]

s 461(e) is traditionally broadly interpreted:

- There is no need to establish the unanimous agreement of all directors.


- ‘Affairs of the company’ is to be interpreted very broadly.
- Court will look through the corporate veil to determine whether the directors have acted in their own interests.
- It is not necessary to show that the directors have preferred their own interests over the interests of all shareholders – simply must show
that they have preferred their own interests over the interests of one or more sections of the members.
o It is enough that the conduct is unfair to a significant body of members.
o Not every shareholder might be on board, but it is possible to wind up anyway

* Re Weedmans Ltd [1974] Qd R 377 [Redmond [8.175]

Sportscraft secured voting control of Weedmans and appointed its nominees to constitute a majority of the Weedmans board. Controlling
nominees then issued a substantial block of shares to a Sportscraft subsidiary, Sportsgirl, in abuse of their fiduciary powers. They
subsequently took steps to facilitate a takeover bid for the outstanding equity in Weedmans. Notwithstanding that the bid price was "grossly
less than a true or fair price," the nominee directors recommended that shareholders accept the offer. They announced that they intended to
do so for their shares which they held (however held merely as nominees for Sportscraft).

Court held winding up order cold be made

The Court accepted that the recommendation for acceptance of the bid, even at a gross undervalue, did not, of itself, involve breach of the
nominees' duty. However, the deceitful and misleading statements made to shareholders by the nominee directors as to their own
shareholdings and other matters were held to offend standards of commercial morality and to react unfairly and unjustly against other
members.

C. Relationship between winding up and oppression provisions

s 467(4) Compulsory liquidation remedies are last resort:


s 467(4): ‘..unless it is of the opinion that some other remedy is available’

 Think about: what remedy is your client seeking?


 Do they want to be reinstated?
 Do they want to wind up the company and run far away?

100
7.3.2 Oppression

All roads lead to s 232: broad standing, broad remedies.


First: ask what your client is seeking (trying to get rid of board; wind up company, share buyout, etc)
Standing very broad: Member; Former member; Offences against you in another capacity; Bring an
action as a creditor; Company A might be causing substantial damage to Company B – shareholder of
Company B has standing to bring an action for oppression against A.

s 232: COURT CAN MAKE AN ORDER IF:


(a) conduct of company’s affairs or (b) actual or proposed act or omission by or on behalf of a
company or (c) resolution or proposed resolution
is either
(d) contrary to interests of members as a whole OR
(e) oppressive, unfairly prejudicial or unfairly discriminatory

Cases

 Thomas: mere prejudice or discrimination is insufficient


 Wayde: conduct outside ordinary standards of reasonableness and fair dealing
o c.f. expectation of trust or involvement in management Ebrahimi
o No need to establish irregularity or breach of legal rights
 Jenkins v Enterprise Gold Mines: The presence of conflicts of interest and the inability to
explain how the conflicts are to be resolved will result in the inference of unfair conduct.
o Also  oppressor cannot use voting power to cure oppression (this is just further
oppression)
 Campbell v Backoffice: Wrongful exclusion from management of a company can constitute
oppression under s 232.
o Does not need to be one single, major instance of oppression  can be one of a
number of things together can constitute oppression.
o Don’t need unlawful conduct or mala fide intention.
Oppression in Corporate Groups

 Re Spargos: In a corporate group, oppression remedy grounded where actions devoid of any
benefit to company to which they were appointed.
 REMEDIES: Court can order appointment of new board (among other things  winding up
not appropriate, company there yet)
s 233 REMEDIES (can do pretty much anything)
(a) winding up
(b) constitution be modified or repealed;
(c) regulating the conduct of the company's affairs in the future;
(d) share buy-out
(e) purchase of shares to reduce share capital

101
A. Early interpretations

Deficiencies in courts’ interpretation of original oppression provision


Oppression is a fairly modern statutory remedy (introduced in 1983)
Before then, general law had lots of deficiencies, statute intended to fix this.

 Never really worked at general law for a number of reasons


o Pre-1983 general law oppression remedy: could only bring an action as a member for
offences that affected you in capacity as a member (e.g. membership rights –
Hickman)
o This is why Ebrahimi could not bring an action for oppression at general law 
affected directorship rights, not membership rights.
 Arguably, that was the exact situation: breakdown of trust and confidence,
squeezing him out with no remedy  that is the exact poster child situation
for why oppression remedy is required
 Original definitions of oppression at general law  very restrictive
o Very very hard to establish:
 Narrow definition of oppression – “burdensome, harsh and wrongful AND
lacking in the degree of probity that other members are entitled to expect”;
 “Wrongful” = legal impropriety or invasion of legal rights
 Minority also had to prove the elements
 Wrongful conduct must be continuing at the time of the petition i.e.
oppression did not cover isolated acts, past acts or threatened conduct
(though it could cover inactivity and omissions).
o Therefore, the courts focused on the quality of the alleged oppressive conduct, not on
the effects of that conduct on the petitioner. (see Brennan J in Wayde)

B. Modern grounds for relief

See above
Standing
Treadtel International Pty Ltd v Cocco [2016] NSWCA 360

Facts:

Held:

Grounds for complaint


Mere discrimination insufficient to ground a remedy under s 233.
Need to show oppression: unfair detriment (consider expectations of shareholders: history of company
showed always had modest dividends and Thomas not involved in management.
* Thomas v H W Thomas Ltd (1984)

Facts: The case concerned a small proprietary company where a minority shareholder was dissatisfied with the way the business was being
conducted. Managing director was fiscally conservative and modest dividends being declared. The petitioner was locked into the company,
since the other shareholders refused to buy him out and there was no active market for his shares.

Held: No oppression. Court held that there was nothing unfair about making the applicants abide by the directors’ decisions as to
appropriate distributions. Mere disagreement with management policies is not enough to ground relief and small dividends without anything
more to show that the board’s conduct was not bona fide is not enough to ground relief.

Here, Thomas was far removed from management

102
Test for oppression (Wayde)

 More than mere prejudice; needs to be unfair detriment. Conduct outside ordinary standards
of reasonableness and fair dealing.
 Also s 180(1) compliance: good faith, proper purpose, best interest of the league as a whole.
 Look at the power and discretion and how they were exercised.
* Wayde v New South Wales Rugby League Ltd (1994) 180 CLR 459; (1985) [know this case]

Facts: Constitution of NSW Rugby League Contained a provision that allowed league to determine which teams would be permitted to play
in the competition. The constitution also provided that the directors of the league were to exercise their powers in the best interest of the
game of rugby league. The Ds determined that the # of teams in the competition was uneconomical (with several clubs in poor financial
condition) and detrimental to the players (b/c there were too many games). League called for applications. Western Suburbs put in an
application for the next year, rejected. Wayde, on behalf of Wests, applied for order that League’s action was a breach of the predecessor of
s 232(e).

Held: League’s decision to exclude wests was made in good faith and did not constute oppression. Decision was w/in power of Ds under the
co’s constitution and was done for benefit of the competition.

Brennan J
1. No need to establish any irregularity or breach of legal rights;
2. Mere prejudice or discrimination is not sufficient to est a breach of s 232, particularly where constitution expressly allows Ds to prejudice
or discriminate against members for the benefit of the co. It needs to be unfair prejudice or discrimination
3. The court will assess the conduct according to ‘ordinary standards of reasonableness and fair dealing’. This is an objective assessment and
requires the court to determine whether a reasonable D would think conduct unfair.

How does oppression apply to corporate groups?


In a corporate group, oppression remedy grounded where actions devoid of any benefit to company to
which they were appointed. Court can order appt of new board (among other things). (Re Spargos).
* Re Spargos Mining NL (1990)

Facts: Spargos Mining NL was one of a number of companies in the Independent Resources Group. Companies within the group were
controlled directly or indirectly by Independent Resources Limited. Shortly after the takeover, the board became dominated by
representatives of the IRL board. At the time of the takeover, Spargos was a company with considerable prospects. As described by Judge
Murray, “it was at least on the threshold of becoming a major gold producer, operating its own mine and having other interests of some
value.” After being subsumed within the IRL group, the company’s situation deteriorated due to a policy whereby Spargos was used as a
cash box to fund, directly or indirectly, various activities undertaken by other companies within the group. These activities were described as
being “almost entirely devoid of any commercial benefit to Spargos and its shareholders, rather being directed to benefit other companies in
the group.” Spargo’s financial situation began to progressively decline.

Held: Under the equivalent of s232, the court found this amounted to oppressive or unfair conduct against the members of the company and
ordered (amongst other things) the appointment of a new board.

Jenkins v Enterprise Gold Mines NL (1992) 10 ACLC 136; (1992)

Facts: Jenkins was a minority shareholder in Enterprise Gold Mines, a public company, whose majority shareholders were drawn from the
IRL group of companies. The directors of Enterprise were appointed by the IRL shareholders, and also held various positions in other IRL
companies. The directors of Enterprise engaged in transactions with other IRL companies that resulted in substantial losses to Enterprise.
Jenkins sued for oppression on the basis that the directors had breached their duties to Enterprise by preferring the interests of other IRL
companies. Jenkins sought the appointment of a receiver over Enterprise.

Held: Once oppression or unfair prejudice is established, the court is obliged to rectify the matter via an appropriate remedy. The holding of
a general meeting of shareholders for the purpose of remedying oppression does not work where the oppressor effectively controls the
voting power of such meeting. The meeting itself worked further oppression.

Orders
Campbell v Backoffice Investments Pty Ltd [2009]

A claim of oppression through wrongful exclusion from management in the context of a 50/50 held company.

High Court confirmed that even prior to the amendments of the s232, wrongful exclusion from management could constitute a form of
oppression. (i.e. modern remedy for Ebrahimi). Oppressive conduct (aside from its oppressive nature) does not need to be unlawful conduct.
Conduct in breach of contractual obligations can also be classified as oppressive conduct.

There can be oppressive conduct even when a person thinks he or she is acting rightly.

103
7.4 Protection of class rights

s246B(1): where the constitution sets out a procedure for varying or cancelling class rights (e.g.
special resolution passed by class itself)
s246B(2): if company doesn’t have an express provision in their constitution, it will be implied.
Cumbrian Newspapers Distinction between shareholder rights annexed to particular shares, compared
to rights annexed to a clump of shares. However, no reason that shareholder rights cannot be class
right just because they’re not annexed to particular shares.

 YES: Rights or benefits annexed to particular shares;


 NO: Rights or benefits conferred on an individual otherwise than in the capacity of
shareholder (Eley’s Case);
 MAYBE: Shareholder rights that are not attached to particular shares E.g: Bushell v Faith
(weighted voting); Rayfield v Hands [1960] Ch 1 (put-option with directors).
White v Bristol Aeroplane: Distinction between variation of class rights and enjoyment of class
rights.

 Primary examples of affecting enjoyment of class rights only: diluting share classes, splitting
classes with new rights
 Entirely reversed under s 246C:
 s246C(6): deems an issue of preference shares to vary the rights of existing preference
shareholders
Personal rights:

 Personal rights conferred through CA and general law.


 Don’t need a derivative action
 Shareholders have a personal right to ensure that the voting power of their shares is not
reduced or diminished by an improper share issue
* Cumbrian Newspapers Group Ltd v Cumberland & Westmorland Herald Newspaper and Printing Co Ltd [1987]

Facts: In 1968, the plaintiff company acquired 10.7% of the issued ordinary shares in the defendant company. The arrangement was part of
a partial merger by way of a defensive takeover strategy, with the plaintiff acting as a white knight for the defendant. Defendant adopted 3
new articles giving the plaintiff: (1) Pre-emption rights over ordinary shares in the defendant. (2) Rights with respect to unissued capital. (3)
Right to appoint a director while holding at least 10% of shares. 18 years later, the defendant company wanted to cancel the articles
conferring the benefits. The plaintiff argued that the rights were class rights and therefore protected under the provision in the company’s
constitution requiring class consent for alteration of class rights.

Held: Distinction between shareholder rights annexed to particular shares, compared to rights annexed to a clump of shares. However, no
reason that shareholder rights cannot be class right just because they’re not annexed to particular shares.

* White v Bristol Aeroplane Co [1953]

Facts: There was an allotment of bonus shares; both ordinary and preference, to ordinary shareholders. The company’s constitution had
been drafted very broadly so that it didn’t just refer to “variation or cancellation” of the rights. The provision said that class consent was
required whenever class rights were “affected, varied, dealt with, abrogated in any manner.

The court, however, still held that the preference shareholders’ rights had not been “affected, varied etc.”. The court thought that a sensible
distinction could be made between affecting rights compared with affecting the enjoyment of rights. Only the latter had occurred here.

104

You might also like